Bên cạnh đó các tính chất hình học như độ cong, độ xoắn và các tính chất topo của không thời gian đang đem lại rất nhiều quan niệm mới mở ra những chân trời ứng dụng mới không những cho [r]
(1)TRÍ TƯỞNG TƯỢNG quan trọng TRI THỨC
ALBERT EINSTEIN
GIẢ THUYẾT KEPLER VÀ BÀI TOÁN XẾP CAM - Dương Đức Lâm
CHỨNG MINH CÔNG THỨC EULER CHO ĐA DIỆN BẰNG VẬT LÝ - Đàm Thanh Sơn BÀI TOÁN CHỨNG MINH TRUNG ĐIỂM VÀ CÁC MỞ RỘNG - Trần Quang Hùng
XẤP XỈ DIOPHANTINE -
DÀN, CẦU NỐI ĐẾN VỚI ĐỘNG HỌC THUẦN NHẤT - Lý Ngọc Tuệ
(2)CHỦ BIÊN:
Trần Nam Dũng BIÊN TẬP VIÊN:Võ Quốc Bá Cẩn Ngô Quang Dương Trần Quang Hùng Nguyễn Văn Huyện Dương Đức Lâm Lê Phúc Lữ Nguyễn Tất Thu
Đặng Nguyễn Đức Tiến
(3)tập Epsilon xin trân trọng ủng hộ
Số 12 mà bạn cầm tay chưa phải số cuối Epsilon Đó số cuối năm 2016, năm thứ hai Epsilon Chúng số báo cuối phiên 2015-2017 Epsilon vào ngày 13 tháng năm 2017
(4)Lý Ngọc Tuệ
Xấp xỉ Diophantine - Dàn, cầu nối đến với hệ động học 15
Nguyễn Ái Việt
Hình học Thời gian 22
Đàm Thanh Sơn
Chứng minh công thức Euler cho đa diện vật lý 35
Dương Đức Lâm
Giả thuyết Kepler toán xếp cam 38
Neal Koblitz (người dịch: Hảo Linh)
Thi trắc nghiệm gây nhiều tranh cãi Mỹ 46
Trần Quang Hùng
Bài toán chứng minh trung điểm mở rộng 50
Nguyễn Ngọc Giang
Sáng tạo toán học phương pháp vật lý học 62
Đặng Nguyễn Đức Tiến
Tốn học giải trí với Tư sáng tạo 71
Nguyễn Tài Chung
Sử dụng tổng tích phân để tính giới hạn dãy số 77
Nguyễn Tất Thu
Hàm Phần Nguyên 106
Trịnh Đào Chiến
Từ đa thức Chebyshev đến bất đẳng thức Berstein - Markov 124
Lương Văn Khải - Đỗ Trần Nguyên Huy
Giải bất đẳng thức đoạn phương pháp biến đổi số 141
Ban Biên tập
(5)GIỚI THIỆU
Cùng với phát triển vượt bậc kỹ thuật, việc chỉnh sửa ảnh ngày thực dễ dàng vài thao tác đơn giản máy tính hay điện thoại Điều giúp cho ảnh đẹp hơn, thể tốt chủ đề mà người chụp ảnh muốn lưu lại dẫn đến hệ khơng thể chối bỏ: khái niệm “thấy tin” không cịn đứng vững thơng qua ảnh! Với cơng nghệ nay, ảnh giả mạo xuất khắp nơi, lĩnh vực Vậy làm để kiểm chứng ảnh thật hay giả? Và tốn học có liên quan đến việc này? Epsilon gửi đến bạn đọc giải đáp cho câu hỏi qua viết kỳ đăng số số
Trong viết này, gửi đến bạn đọc nhìn tổng quát ảnh giả mạo liên quan với đời sống thơng qua phóng ảo, tổng hợp chủ yếu từ câu hỏi mà người viết nhận từ bạn bè đồng nghiệp Để tiện trình bày, chúng tơi tạo hai nhân vật ảo phóng viên (PV) người trả lời người làm nghề giám định ảnh (GĐA) với đoạn hỏi đáp ảo họ
Thế ảnh giả mạo để phát chúng? Phóng viên (PV): Chào người giám định ảnh, chủ đề hơm nói ảnh giả mạo, trước tiên muốn hỏi ảnh giả mạo?
Người giám định ảnh (GĐA): Cảm ơn câu hỏi anh Trả lời cách đơn giản ảnh giả mạo ảnh khơng phản ảnh thật ảnh chụp Tuy vậy, để giải thích chi tiết cần phải trình bày dài dịng chút Trước tiên tơi muốn giới thiệu khái niệm ảnh gốc ảnh chỉnh sửa Ảnh gốc ảnh lấy trực tiếp từ máy ảnh chưa qua thao tác can thiệp Phần lớn máy ảnh hỗ trợ người dùng trích xuất ảnh gốc Ngắn gọn hơn, anh chụp ảnh xong, ảnh thẻ nhớ anh chưa làm gì, ảnh gốc! Sau trích ảnh khỏi máy ảnh, thao tác tác động lên ảnh ảnh xem chỉnh sửa
(6)PV: Xin lỗi tạm ngắt lời, nén ảnh Jpeg bị tính chỉnh sửa? Điện thoại của chụp ảnh lưu dạng này, ảnh chụp xong có phải ảnh chỉnh sửa?
GĐA: Phải phân biệt nén nén lần thứ nén từ nguồn
Các máy ảnh trung cấp trở lên số điện thoại cho phép lưu ảnh gốc dạng khơng nén, gọi ảnh raw Nói nôm na cảm biến sau nhận ánh sáng đo lượng hố ghi xuống Nếu ảnh raw sau chép từ thẻ nhớ nén (có thể theo Jpeg, Jpeg 2000 hay kiểu nén khác) phần mềm máy tính hay điện thoại, ảnh trở thành ảnh chỉnh sửa
Ở máy ảnh hay điện thoại lưu trực tiếp ảnh dạng nén, xem ảnh gốc Nhưng ảnh nén nén lại phần mềm khác (tức nén lần) chúng trở thành ảnh chỉnh sửa Hay nói cách khác, ảnh nén xem ảnh gốc nén tối đa lần việc nén phải thực phần mềm máy ảnh Cũng điều này, ảnh nén từ lần trở lên chắn ảnh chỉnh sửa, ngược lại, ảnh không nén chưa ảnh gốc! Vì vậy, nghề chúng tơi, việc phân tích xem ảnh bị nén lần quan trọng!
PV: Cảm ơn anh, hai kiểu chỉnh sửa lại?
GĐA: Tơi tiếp đến nhóm 2, nhóm chỉnh sửa thơng thường với mục đích làm đẹp ảnh (theo thẩm mỹ người chỉnh) Ví dụ ảnh chụp tối quá, anh tăng độ sáng lên, anh muốn cân sáng lại cho ảnh, nội dung ảnh Các phần mềm hay ứng dụng chỉnh sửa ảnh, đa số nhóm
Cuối nhóm nhóm chỉnh sửa nhiều nhất, làm thay đổi nội dung ảnh Thường thấy anh muốn xố số đối tượng anh khơng muốn xuất Hoặc anh muốn ghép thêm chi tiết Chỉnh sửa nhóm thơng thường làm ảnh khác rõ với thị giác người (nếu anh có ảnh để so sánh)
Quay lại với câu hỏi chính, với đa số người dùng phổ thông, ảnh giả mạo, gọi tắt ảnh giả, ảnh nhóm nhóm 3, nhóm thường đánh giá nhóm nghiêm trọng Tơi muốn nhấn mạnh việc anh chấp nhận ảnh chỉnh sửa, gọi tắt ảnh sửa, tới mức tuỳ theo lĩnh vực Gần nhóm đâu chấp nhận, với thị giác người, ảnh gốc ảnh sửa lúc gần khơng khác Chỉnh sửa nhóm ngược lại, đa số khơng có chấp nhận ảnh cắt ghép Tuy có ngoại lệ, ví dụ số thi nhiếp ảnh chấp nhận ảnh gốc, nên kể việc chỉnh sửa nhóm khơng chấp nhận, việc kiểm chứng địi hỏi nhiều cơng sức Một số thi khác chấp nhận chỉnh sửa mức khuôn khổ số thao tác định, ví dụ nước hồ phù sa mà anh đổi thành vắt không Nhóm 3, khơng chấp nhận nhiều lĩnh vực, quảng cáo lại áp dụng phổ biến
(7)(8)(9)thế này, Exif anh có tên phần mềm sửa ảnh (ví dụ Adobe Photoshop) ảnh bị xem ảnh sửa, khác được! Hoặc thông tin Exif ảnh cho thấy tỉ lệ ảnh 4:3 mà tỉ lệ anh có 16:9 điều cho thấy ảnh sửa, khơng qn Đây cách kiểm tra đơn giản hiệu cao Tuy vậy, cần lưu ý thêm thơng tin can thiệp hiệu chỉnh, nên người sửa ảnh biết cách chỉnh sửa qua mắt kiểu kiểm tra
Cách thứ hai để kết luận dựa việc bắt lỗi Việc sửa ảnh dễ với công cụ nay, để tạo ảnh chỉnh sửa "như thật" địi hỏi nhiều cơng sức Do vậy, đa số ảnh giả thường hay mắc lỗi Những lỗi thường gặp lỗi ánh sáng tỉ lệ vật lý người quen làm việc với ảnh chiều để ghép ảnh cho tốt thường phải có mơ hình chiều Cũng tương tự cách thứ 1, cần phát lỗi khẳng định ảnh sửa Cách thức cuối giám định ảnh Thật giám định ảnh bao gồm cách trên, muốn tách riêng để nhấn mạnh điều mà có kỹ thuật giám định làm Tồn q trình tạo ảnh để lại dấu vết định Dấu vết thơng tinh ảnh Exif, lỗi việc chỉnh sửa ảnh, nhiễu ảnh từ cảm biến, ảnh hưởng quang học từ ống kính, dấu vết giá trị lượng hố q trình nén ảnh, dấu vết lọc ảnh, tỉ lệ vật lý, phản xạ vào dấu vết này, (những người giám định ảnh) tái dựng (về mặt lý thuyết) tồn trình tạo ảnh, từ loại máy ảnh nào, số chụp sao, nén ảnh kiểu gì, thao tác áp dụng lên ảnh cuối Một ảnh ghép khơng có lỗi nào, khơng thể ghép mà không để lại dấu vết Do vậy, từ dấu vết để lại, đưa kết luận kèm với độ xác thực ảnh có phải ảnh sửa hay khơng, hay chí ảnh chụp máy ảnh cụ thể Trong số tới Epsilon, giới thiệu với bạn đọc vài cách thức giám định ảnh bản!
PV: Như tơi tóm tắt tìm chứng bất kỳ, ảnh bị xem là ảnh sửa Nhưng tơi có ảnh xem thật ảnh giả kỹ thuật giám định chưa phát dấu vết sửa ảnh? Hay nói cách khác, ảnh giả nên khơng thể tin cậy vào ảnh nữa?
(10)Hình 3: Đây dạng khác ảnh Ảnh phịng tuyệt đẹp khơng phải ảnh giả, ảnh thật Đây ảnh hồn tồn tạo dựng máy tính Đây vấn đề hóc búa bên cạnh việc phát ảnh giả Tác phẩm tạo nghệ sĩ Mateusz Wielgus
PV: Tôi gặp nhiều nhiếp ảnh gia hay chuyên viên chỉnh sửa ảnh phát biểu: “Vì tơi làm nghề này, nên nhìn vào ảnh chỉnh sửa biết ngay,” điều có hay khơng?
GĐA: Đây ngộ nhận thường gặp người làm nghề liên quan tới ảnh, đặc biệt ảnh số Thực tế phủ nhận kinh nghiệm chụp hay chỉnh sửa ảnh giúp ích số tình huống, để thật xác định ảnh sửa bắt buộc phải có kiến thức giám định ảnh Nói nơm na, điều giống kiếm hiệp, cao thủ phán đoán “chiêu thức” xem qua thi thể nạn nhân, để đến kết luận, phần lớn trường hợp bắt buộc phải có giám định pháp y! Kiến thức giám định ảnh, thực tế, liên quan mật thiết chặt chẽ với toán học nhiều với kiến thức nhiếp ảnh
Như trả lời cách tìm ảnh sửa cách gần làm (đọc Exif), cách thứ hai (tìm lỗi) cách mà kinh nghiệm nhiếp ảnh đem lại lợi so với người khơng có kinh nghiệm Tuy vậy, với ảnh giả đầu tư, hay gọi "giả y thật" hai cách kiểm tra khơng hiệu quả, cần phải áp dụng giám định ảnh
(11)hoặc “đây ảnh thật”, mà có dạng: “ảnh có kiểm định đạt độ tin cậy X% dựa kỹ thuật giám định A, Y% dựa kỹ thuật giám định B, ” vào mà xem xét đưa phán kết hợp với chứng lập luận khác
Thực tế dùng ảnh làm chứng khó khăn người thường nghĩ nhiều Tôi lấy ví dụ anh có ảnh kẻ thực vụ án mạng anh dùng để tố cáo nghi phạm Trước tiên, ảnh anh gửi cho (giám định viên) để giám định Nhưng để giám định, cịn có phận làm thao tác đảm bảo tính trọn vẹn liệu, tức họ phải đảm bảo ảnh mà chúng tơi khảo sát ảnh mà anh gửi tố cáo Việc thực thông qua hàm băm, ví dụ gần họ phổ biến sử dụng SHA Sau khảo sát, đưa kết luận nói phần đầu trả lời câu hỏi này, tức phần trăm với kỹ thuật kiểm định Giả sử xác định ảnh anh đưa ảnh thật chưa qua chỉnh sửa, sau phận khác tiếp tục phân tích xem trường mà anh ghi nhận có phải trường mà anh tố cáo hay không, người mà anh ghi hình có phải nghi phạm hay khơng Tất thơng tin tập hợp lại trước tồ vào họ đưa kết luận
Điều đặc biệt khó khăn ảnh anh bị nén nhiều lần hay ví dụ anh lấy từ đoạn video youtube thơng tin nhiều ảnh anh lúc chắn ảnh sửa
PV: Như muốn nguỵ tạo chứng ảnh giả, nên dùng ảnh có độ phân giải cao, nén lần?
GĐA: Nếu anh có ảnh giả có độ phân giải cao, lưu lại nhiều dấu vết dễ truy q trình chỉnh sửa ảnh, cịn anh có ảnh giả có độ phân giải thấp làm trình giám định khó khăn giá trị sử dụng làm chứng lại thấp Do vậy, ngụy tạo chứng luôn việc làm sai trái bất lợi trường hợp!
(12)PV: Tơi khơng thấy tốn học có vai trị Liệu anh giải thích rõ vì sao giám định ảnh lại liên quan đến toán học nhiều nhiếp ảnh anh đề cập?
GĐA: Như đề cập câu trả lời trước, giám định ảnh cần phải xây dựng dựa chứng kiểm định Tốn học đóng vai trị trọng yếu việc xây dựng độ đo để kiểm định giả thiết đưa Bên cạnh đó, tốn học giúp mơ hình hố q trình ghi nhận ảnh xấp xỉ loại nhiễu Đây thành tố quan trọng giám định ảnh!
Như nói trên, việc nén ảnh từ lần trở lên làm cho ảnh trở nên chắn ảnh sửa, để phát việc nén ảnh lần kiến thức nhiếp ảnh khơng làm được, toán học làm Một cách làm dựa luật Benford phân tích phân bố chữ số biến đổi Cosine rời rạc ảnh Anh xem Epsilon số 5, chúng tơi có nói đến ứng dụng Trong số tới, chúng tơi trình bày số phương pháp để phát ảnh cắt ghép, để xác định ảnh chụp camera Tất dựa toán học!
PV: Anh nêu ví dụ cụ thể việc áp dụng toán học mà sử dụng ảnh nén hay không? Ý tơi ví dụ dễ hình dung hơn.
GĐA: Tơi giới thiệu ví dụ tiếng ảnh chụp Lee Harvey Oswald
Ngày 22/11/1963, tổng thống Mỹ đương thời John F Kennedy bị ám sát, thủ sau xác định Lee Harvey Oswald, xạ thủ bắn tỉa Một chứng quan trọng ảnh chụp lại cảnh Oswald sân sau cầm loại súng trường xác định loại dùng để ám sát Kennedy
Tại thời điểm bị bắt, Oswald tuyên bố ảnh giả mạo nhiều chuyên gia chi tiết giả mạo, bao gồm chi tiết sau:
1 Ánh sáng bóng ảnh trái thực tế Cụ thể họ cho vùng bóng mũi Oswald vùng bóng tồn thân (trên nền) có chiều trái ngược
2 Đặc điểm khuôn mặt Oswald khơng phù hợp với hình ảnh khác Cụ thể cằm Oswald đời nhỏ so với ảnh chụp, nên người ảnh khơng phải Oswald
3 Kích thước súng trường không phù hợp với độ dài biết đến loại súng Cụ thể tỉ lệ độ dài súng so với chiều cao Oswald ảnh không hợp lý (tỉ lệ theo độ dài súng chia cho chiều cao Oswald 0.6493, tỉ lệ ảnh 0.5824);
4 Tư đứng Oswald bất hợp lý, thăng đứng Mặc dù vậy, vào chứng khác, Oswald bị buộc tội Nhưng liệu có phải ảnh giả?
(13)Hình 4: Lee Harvey Oswald mơ hình phân tích 3D Từ phải sang trái: Ảnh chụp Lee Harvey Oswald vào năm 1963; mơ hình 3D tái dựng dựa số sinh học Oswald kích thước vật dụng khác cho thấy chiều dài súng, gương mặt bóng Oswald hình xác; mơ hình 3D có bổ sung thêm khối lượng phần thể Oswald cho thấy dáng đứng thật thăng (Nguồn ảnh: Đại học Dartmouth, tháng 10 năm 2015.)
(14)GĐA: Nếu bạn muốn thử nghiệm, tơi giới thiệu số dịch vụ sau internet (vẫn hoạt động Epsilon số 12 đăng):
http://29a.ch/photo-forensics/ http://www.imageforensic.org/ http://fotoforensics.com/ http://www.getghiro.org/
Các công cụ địi hỏi phải có kiến thức tảng giám định ảnh định, bạn đọc thử nghiệm, đặc biệt phân tích Exif Với bạn đọc u thích tốn học, chúng tơi xin hẹn vào số tới, chúng tơi trình bày chi tiết cách dùng tốn để phát ảnh sửa nguồn gốc ảnh
(15)1 Giới thiệu
Trong phần trước loạt xấp xỉ Diophantine, trả lời cho câu hỏi khả xấp xỉ số (véc tơ, ma trận) thực số (véc tơ, ma trận) hữu tỉ với kết kinh điển: Định lý Dirichlet, Định lý Khintchine, dày đặc tập số (véc tơ, ma trận) xấp xỉ Các công cụ mà sử dụng để chứng minh định lý phần trước bao gồm liên phân số3 (cf [8,10], quy tắc Dirichlet, hình học số4 (cf [9]), trò
chơi Schmidt/trò chơi siêu phẳng tuyệt đối5 (cf [11]) Trong phần này, xin giới
thiệu hướng phát triển mạnh mẽ khoảng 20 gần lý thuyết xấp xỉ Diophantine: thông qua mối liên hệ với động học lý thuyết ergodic6.
Mối liên hệ số học động học có lẽ lần lưu ý đến Raghu-nathan vào năm 1970, giả thuyết Oppenheim [15] số học:
Giả thuyết 1(Giả thuyết Oppenheim). Chon ≥3 GọiQlà dạng tồn phương khơng xác
định với hệ số thực bội số dạng với hệ số hữu tỉ Với mọiε > 3, tồn
tại véc tơ~xkhác cho0< Q(x, x)< ε.
tương đương với tính chất sau không gian dàn:
Mọi quỹ đạo tương đối compact củaSO(2,1)trongSL(3,R)/SL(3,Z)là compact.
Quan sát Raghunathan dẫn đến lời giải cho giả thuyết Oppenheim Margulis [12] vào năm 1987 Mối liên hệ tương tự sử dụng chứng minh Einsiedler, Katok, Lindenstrauss [4] tập số không thỏa giả thuyết Littlewood có chiều Hausdorff (một kết mang lại giải Fields cho Lindenstrauss vào năm 2010)
Vào năm 1985, mối liên hệ trực tiếp xấp xỉ Diophantine động học không gian dàn nhận cụ thể Dani [3] sau:
3continued fractions 4geometry of numbers
(16)SL2(R)(SLn+1(R),SLm+n(R)).
Sử dụng mối liên hệ này, Dani áp dụng kết Schmidt xấp xỉ Diophantine để chứng minh kết động học tập điểm có quỹ đạo bị chặn tác động nhóm đường chéo tham số Định lý tập thắng trò chơi Schmidt
Tương ứng Dani sau Kleinbock Margulis [5] sử dụng để chứng minh giả thuyết Baker-Sprindˇzuk phần giao đa tạp giải tích9 với tập số xấp xỉ tốt có độ đo
bằng Sau đấy, Kleinbock Margulis mở rộng tương ứng Dani cho ma trậnψ-xấp
xỉ được, áp dụng kết động học để đưa chứng minh hoàn toàn khác cho định lý Khintchine-Groshev Phương pháp sau mở rộng áp dụng để chứng minh định luật 0-1 cho đa tạp giải tích thực [1,2] vàS-arithmetic10[7,13,14]
với trường sở làQ
Trong phần 3, phát biểu lại Tương ứng Dani cách cụ thể chứng minh tương ứng Trước phần 2, khảo sát qua số điều không gian dàn động học phần sau
2 Không gian dàn trong Rn
Một dàn không gian véc tơ Rn là khái niệm tổng quát hóa tập số nguyênZ⊆ R
lênRn Xét bên tập số ngun có số tính chất đáng ý sau:
(1) Zlà nhóm củaR (2) Zlà tập rời rạc trongR
(3) Tồn miền bản11: Có song ánhR/Z
→[0,1) : x7→x mod
(4) Miền bản[0,1)củaZtrongRcó độ dài hữu hạn (= 1) Và độ dài củaR/Zđược định
nghĩa độ dài miền
Các tính chất sử dụng để định nghĩa dàn nhóm LieGbất kỳ sau: Định nghĩa 2. Một tập conΓ⊂Gđược gọi dàn như:
(1) Γlà nhóm rời rạc trongRn 7unimodular lattice
8diagonal one-parameter subgroup 9analytic manifold
(17)(iii) 1
−1 y :x, y ∈Z dàn trongR
Tất dàn trongRcó thể mô tả dễ dàng sau:
Bài tập 4. Chứng minh rằngΓlà dàn trongRkhi khiΓ =aZvớia6=
Quan sát mở rộng lên không gian véc tơ cách thay sốa 6= 0bằng
ma trận khả nghịch:
Định lý 5. (1) Với dànΓ⊆Rn, tồn ma trậng ∈GL
n(R)sao cho:
Γ =gZn
(2) Xét dànZntrongRnvàg
∈GLn(R)là ma trận khả nghịch Khi đấy,gZn =Znkhi
và khig ∈SLn(Z).
Bài tập 6. Chứng minh nhưΓ dàn trongRn, thì Γcó chứa n véc tơ độc lập
tuyến tính
Bài tập 7. Chứng minh Định lý
Lưu ý 8. DànΓtrongRnđược gọi dànđơn modula13nếu nhưλn(Rn/Γ) = 1, vớiλnlà
độ đo Lebesgue trênRn Khi đấy, ma trậngtrong Định lý thuộc tập ma trận tuyến tính
đặc biệtSLn(R) Tập dàn đơn modula trongRnđược ký hiệu bởiΩn
Định lý cho phép đồng nhấtΩnvớiSLn(R)/SLn(Z)qua phép gán:
Γ =gZn 7→gSLn(Z)
Lưu ý tập ma trận tuyến tính đặc biệtSLn(R)là nhóm Lie, vàSLn(Z)cũng
dàn trongSLn(R) Cách đồng đem lại độ đo, khoảng cách Riemann, không gian tiếp
tuyến, nhiều tính chất khác từSLn(R)/SLn(Z)choΩn Cụ thể hơn,Ωntrở thành không
gian đồng
(18)(~x1, , ~xn)choΓvà tập sinh(~y1, , ~yn)choΓ0 cho:
max{k~xi−~yik: 1≤i≤n}
đủ nhỏ, khoảng cách Riemann giữaΓvàΓ0 tương đương với giá trị
Lưu ý 9. Khi n = 1, có dàn đơn modula R Z Với n ≥ 2, miền dàn SLn(Z)trong SLn(R) có độ đo hữu hạn, khơng
tương đối compact dàn trongRn Nói cách khác, khin
≥2, tậpΩnk0 phải
tập compact
Vậy dàn đơn modula trongRncó thể "tiến đến vô cùng" nào? Cho dãy dàn
{Γi}∞i=1 Gọi~xilà véc tơ khác ngắn củaΓi Nếu như:
lim
i→∞k~xik= 0,
thì giới hạn dànΓisẽ bị bậc tự do, k0 phải dàn
Rn:
lim
i→∞Γi ∈/ Ωn Xét hàm:δ: Ωn →R>0,
Γ7→δ(Γ) := min{k~xk:~x∈Γr{0}}
Hàmδcó thể xem "khoảng cách đến∞của dànΓ Và ta nói làΓi ∈Ωntiến đến vơ
nếu
lim
i δ(Γi) =
Chúng ta gọi tập S ⊆ Ωn bị chặn tồn tập K ⊆ Ωn compact cho
S ⊆K Các tập bị chặn củaΩncó thể mơ tả tiêu chuẩn sau Mahler14:
Định lý 10(Tiêu chuẩn compact Mahler). TậpS ⊆Ωnlà bị chặn khi:
inf
Γ∈Sδ(Γ)>0
Nói cách nơm na, tập củaΩnbị chặn khoảng cách từ tập đến vô đủ
xa
(19)~x− ~pq
> |q|1+c
n
Ký hiệu tập số xấp xỉ BA, tập véc tơ xấp xỉ BAn Tương ứng
Dani xây dựng sau Với số thựcx∈R, ký hiệu ma trận: ux :=
1 x
Hiển nhiênux ∈SL2(R) Gán choxmột dàn đơn modulaΛxnhư sau:
Λx:=uxZ2 ∈Ω2
Với mỗit ∈Z, đặtgtlà ma trận đường chéo trongSL2(R)định nghĩa bởi:
gt :=
et e−t
Ta dễ dàng thấy với mọitvà với mọix: gtΛx =gtuxZ2 ∈Ω2
Tương ứng Dani cho tậpBAcó thể phát biểu sau:
Định lý 11. Số thựcx∈Rlà số xấp xỉ luồng chiều16{g
tΛx :t≥0}
trongΩ2bị chặn.
Chứng minh. (⇒): Giả sử nhưx∈BA, vớic >0sao cho với mọip, q ∈Z, q6= 0:
x− pq
> qc2 15badly approximable
(20)gtux
−p q = et e−t
x −p q =
et(qx−p) e−tq
= maxet|qx−p|, e−t|q|
≥pet|qx−p| ·e−t|q|
=p|qx−p| · |q|
>√c
Vì vậy,
δ gtuxZ2 :t≥0 ≥√c >0,
và theo tiêu chuẩn compact Mahler,gtuxZ2 :t≥0 bị chặn trongΩ2
(⇐): Theo chiều ngược lại, giả sử như:
δ gtuxZ2 :t≥0 =c >0
Giả sử nhưp, q ∈Z, q6= 0sao cho:
|q| · |qx−p|<1
Đặt
t=−1
2log
x−pq
>0
Khi đấy, ta có được:
et· |qx−p|=e−t|q|=
et(qx−p) e−tq
= gtux
−p q
≥δ gtuxZ2 :t≥0 =c
Từ ta suy ra:
|q| · |qx−p|=et|qx−p| ·e−t|q| ≥c2 > c2/2>0
Và vậy,x∈BA
Tương ứng Dani mở rộng choBAnnhư sau:
Định lý 12. Véc tơ~x∈Rnlà véc tơ xấp xỉ luồng chiều{g
tΛ~x :t≥0}
trongΩn+1bị chặn, với:
Λ~x :=
In ~x
1
Zn+1,
và
gt:=
entI n
e−t
Bài tập 13. Chứng minh Định lý 12
Bài tập 14. Phát biểu Tương ứng Dani cho tập ma trận xấp xỉ A ∈ BAn,m: tồn
c >0sao cho với mọi~p∈Zn, ~q∈Zm, ~q6= 0:
(21)proximation, J Reine Angew Math.359(1985), pp 55–89
[4] M Einsiedler, A Katok, E Lindenstrauss,Invariant measures and the set of exceptions to Littlewoods conjecture, Ann Math.164(2006), pp 513–560
[5] D Kleinbock G Margulis, Flows on homogeneous spaces and Diophantine approxi-mation on manifolds, Ann Math.148(1998), pp 339–360
[6] D Kleinbock G Margulis, Logarithm laws for flows on homogeneous spaces, Invent Math.138(1999), pp 451–494
[7] D Kleinbock G Tomanov, Flows onS-arithmetic homogeneous spaces and
applica-tion to metric Diophantine approximaapplica-tion, Max Plank Institute for Mathematics preprints (2003), no 65, pp 1–45
[8] Lý Ngọc Tuệ,Xấp xỉ Diophantine trênRvà Liên phân số, Epsilon4(2015)
[9] ——, Xấp xỉ Diophantine trên Rn - Quy tắc Dirichlet Hình học số, Epsilon 5
(2015)
[10] ——,Xấp xỉ Diophantine với độ đo - Định lý Khintchine, Epsilon6(2015)
[11] ——, Xấp xỉ Diophantine trênRn - Véc tơ xấp xỉ Trò chơi siêu phẳng tuyệt đối,
Epsilon8(2016)
[12] G Margulis, Formes quadratiques indefinies et flots unipotents sur les espaces ho-mogènes, C R Acad Sci Paris, Série I304(1987), pp.249–253
[13] A Mohammadi A Salehi Golsefidy, S-arithmetic Khintchine-type theorem, Geom
Func Anal.19(2009), pp 1147–1170
[14] ——,Simultaneous Diophantine approximation on non-degeneratep-adic analytic
mani-folds, Israel J Math.188(2012), pp.231–258
[15] A Oppenheim,The minima of indefinite quaterary quadratic forms, Proc Natl Acad Sci U.S.A bf 15 (1929), pp 724–727
(22)HÌNH HỌC VÀ THỜI GIAN
Nguyễn Ái Việt
(Viện Công Nghệ Thông Tin, Đại Học Quốc gia Hà Nội)
TÓM TẮT
Hình học phẳng hình học khơng gian chương trình tốn phổ thơng hình học Euclide Thêm chiều thời gian vào hình học có hình học khơng thời gian hay cịn gọi hình học Minkowski với nhận thức giới xung quanh Một phát quan trọng hình học khơng thời gian có ảnh hưởng tới phát triển nhân loại công thức lượng Einstein Cơng nghệ định vị tồn cầu GPS khơng thể xác khơng tính đến quan niệm khoảng cách thời gian Hình học khơng thời gian đời hình học Euclide khơng thể giải thích quan sát thực nghiệm vận tốc ánh sáng Einstein, Lorentz, Poincaré Minkowski tìm hình học khơng thời gian tìm mơ hình tốn học để giải thích thí nghiệm Michelson-Morley Ứng dụng toán học vào thực tiễn trước tiên phải chọn mơ hình tốn học phù hợp không ràng buộc quan niệm vào lý thuyết toán học
Nguyên lý cộng vận tốc
Giả sử có tàu chạy với vận tốc v, tàu có hành khách với vận tốcu0 Người
quan sát sân ga thấy hành khách chuyển động với vận tốculà bao nhiêu?
Bài toán đơn giản học sinh phổ thông, thử thận trọng kiểm tra bước lập luận Nhiều ý tưởng khoa học vĩ đại đời xem xét ví dụ đơn giản Nếu đánh dấu điểmX tàu, khoảng thời
giant, người quan sát sân ga thấy điểmX dịch chuyển đoạn đườngx =vt So với
điểmX, hành khách đoạn đường làx0 = u0t Trong hình học Euclide, khoảng cách mà hành khách di chuyển so với sân ga làs = s1 +s2 = (v+u0)t Như vận tốc của hành khách làu=v+u0 Đó lànguyên lý cộng vận tốc(Hình 1)
Nguyên lý cộng vận tốc ứng dụng rộng rãi đời sống tỏ xác với vận tốc đời thường, nhỏ so với vận tốc ánh sángc= 300.000km/c Toán học đẹp đẽ chỗ
(23)Hình 1: Nguyên lý cộng vận tốc
Hình 2: Vận tốc ánh sáng không đổi nguồn sáng chuyển động
lập tức, ông nghĩ tới việc áp dụng lý thuyết để mở rộng lý thuyết tương đối Einstein Do sử dụng công thức sai mà Cartan cho "hiển nhiên", ông đến hệ mâu thuẫn với thực tiễn Điều làm việc ứng dụng lý thuyết Cartan vào thực tế bị chậm lại gần nửa kỷ
Chúng ta thử xét thêm toán khác Giả sử tàu chuyển động với vận tốc v có
một nguồn sáng Biết ánh sáng truyền với vận tốc xấp xỉc= 300.000km/s Nếu áp dụng
nguyên lý cộng vận tốc, người quan sát sân ga thấy ánh sáng chuyển động với vận tốc
c+v (Hình 2)
Mặc dù lập luận nêu trên, dựa nguyên lý cộng vận tốc, thấy kết luận khác xa với thực tế
Thí nghiệm Michelson-Morley
(24)Hình 3: Lực Coriolis chứng tỏ Trái Đất không đứng yên nguồn sáng
Theo kết thí nghiệm Michelson-Morley, ngun lý cộng vận tốc khơng thể áp dụng cho ánh sáng Cụ thể, ví dụ nêu trên, người quan sát đứng sân ga phải thấy vận tốc ánh sáng bằngcgiống hành khách tàu Điều có mâu thuẫn hay khơng? Thí nghiệm
hay lập luận ngun lý cộng vận tốc có sai sót? Trong thực tế, thí nghiệm Michelson-Morley lặp lại nhiều lần, loại bỏ sai số dẫn đến kết luận nhau, lầm lẫn
Chúng ta cịn phải biện luận khả Thí nghiệm Michelson-Morley thực phịng thí nghiệm đặt Trái Đất Vì thế, Trái Đất đứng yên tuyệt đối thuyết địa tâm Nhà Thờ Trung cổ, thí nghiệm Michelson-Morley cho thấy ánh sáng truyền theo phương với vận tốc không đổi Tuy vậy, có chứng khác để chắn Trái Đất không đứng yên thực nhiều chuyển động quay khác Khi vật chuyển động quay, có lực Coriolis tác động lên vật chuyển động bề mặt Trong thực tế người ta quan sát lực hai bán cầu Trái Đất theo hai hướng khác Hình
Chúng ta tìm lời giải thích khác cho thí nghiệm Michelson-Morley
Phép biến đổi Lorentz
Như vậy, cần phải có cơng thức cộng vận tốc áp dụng cho trường hợp nguồn sáng chuyển động Công thức phải đảm bảo vận tốc ánh sáng người quan sát sân ga giống người quan sát tàu bằngcvừa có
thể bao gồm nguyên lý cộng vận tốc cũ mức độ xác
Để làm điều này, xét lại giả thiết "ngầm định" lập luận nêu cộng vận tốc Thậm chí, phải thay đổi quan niệm khoảng cách, thời gian, hai nguyên lý cộng vận tốc
(25)Dễ dàng thử được,công thức biến đổi Lorentz (1) thỏa mãn hệ thức
x2−c2t2 =x02−c2t02 (2)
Hệ thức (2) có ý nghĩa quan trọng: Đại lượngx2−c2t2 luôn số không đổi không phụ
thuộc người quan sát họ chuyển động so với
Trong trường hợp số không đổi bằng0, đối tượng quan sát chuyển động với vận tốc ánh
sáng hai người quan sát
u=x/t=u0 =x0/t0 =c (3)
Đây trường hợp nguồn sáng chuyển động khơng làm thay đổi vận tốc ánh sáng thí nghiệm Michelson-Morley
Chúng ta tìm cơng thức cộng vận tốc từ công thức biến đổi Lorentz (1) sau:
x0/t0 = −v+x/t
1−v/c2x/t, (4)
hoặc cách tường minh
u0 = −v+u
1−v/c2u , u=
u0+v
1 +v/c2u0 (5) Đối với vật chuyển động với vận tốc nhỏ nhiều so với vận tốc ánh sáng v/c2 ≈ 0,
chúng ta có nguyên lý cộng vận tốc cũu=v+u0.
Như vậy, thời gian quãng đường hai người quan sát đo liên hệ với phép biến đổi Lorentz (1), có nguyên lý cộng vận tốc thỏa mãn yêu cầu thực nghiệm
(26)Biến đổi tọa độ khơng gian
Trong hình học Euclide, quen thuộc với việc sử dụng phép biến đổi tọa độ không gian Để đơn giản xem xét không gian hai chiều, tính chất tốn học quan trọng mà quan tâm không thay đổi không gian nhiều chiều
Một điểm P cho trước mô tả tọa độ(x1, x2)trong hệ tọa độ thứ và(x01, x02)
trong hệ tọa độ thứ hai quay gócθso với hệ tọa độ thứ Hình
Sử dụng kiến thức hình học lượng giác phổ thơng, tìm cơng thức biến đổi quay hệ tọa độ sau
x01 = cosθx1+ sinθx2, x02 = sinθx1−cosθx2 (6)
Hiển nhiên khoảng cách đoạn OP không thay đổi ta quay hệ tọa độ Do
OP2 =x21+x22 =x012 +x022 (7)
Không thế, khoảng cách hai điểm bất kỳP vàQcũng không thay đổi với phép quay
hệ tọa độ theo trục hình học Euclide
Theo ngơn ngữ toán học, khoảng cách bất biến với phép quay Trong khơng gian Euclide chiều có phép quay, khơng gian chiều có phép quay độc lập theo ba trục khơng gian vng góc với Tổng qt, khơng gian n chiều cón(n−1)/2phép quay
độc lập
Như vậy, hiểu giáo trình tốn cao cấp, định nghĩa hình học Euclide bao gồm phép biến đổi quay hệ tọa độ tính chất bất biến khoảng cách Trong sách giáo khoa phổ thơng, thuộc tính bất biến "ngầm định" xem hiển nhiên, không
(27)Hệ quy chiếu
Khi có thêm chiều thời gian hình học, có khái niệmhệ quy chiếu, bao gồm tọa độ không gian thời gian Thời gian số thực, có biến thực khác Tuy nhiên, thấy hình học khơng thời gian có tính chất khác khơng gian Euclide chiều tính bất biến khoảng cách
Trong hình học khơng thời gian, bất biến khoảng cách bị thay bất biếnkhoảng không thời gianđược định nghĩa sau
ds2 = (y1−x1)2+ (y2−x2)2+ (y3−x3)2−c2(τ −t)2, (8)
đối với hai kiện xảy tọa độ không gian(x1, x2, x3),(y1, y2, y3)và thời điểm
lần lượt làtvàτ
Điều làm khoảng cách thay đổi hình học không thời gian không gian tính đồng đẳng hướng, mà chuyển động với vận tốc lớn Điều chưa biết vật lý cổ điển, hình học Euclide coi hiển nhiên với thực tiễn Khoảng cách thời gian đo thay đổi người quan sát chuyển động với vận tốc lớn cho phép vận tốc ánh sáng khơng phụ thuộc vào hệ quy chiếu Đó ý nghĩa sâu xa bất biến khoảng không thời gian
Trong không gian ba chiều có phép biến đổi Lorentz độc lập tương tự công thức (1), ứng với tọa độ không gian khác Như vậy, có phép quay tọa độ độc lập khơng gian phép biến đổi Lorentz độc lập
Poincaré người nhận phép biến đổi Lorentz có tính chất giống phép quay Điều khác biệt thay hàm lượng giác thông thường công thức (6) hàm lượng giác hyperbole Như mặt phẳng(ct, x)chúng ta có phép quay hyperbole
ct0 = coshθct−sinhθx, x0 =−sinhθct+ coshθx (9)
Chúng ta tìm hiểu ý nghĩa hàm lượng giác hyperbole liên hệ với hàm lượng giác bình thường sau: Nếu cơng thức Moivre
(28)Do đó,
coshζ = e
x+e−x
2 , sinhζ =
ex−e−x
2 (12)
Hàm lượng giác hyberbole có tính chất sau
cosh2ζ−sinh2ζ = (13)
Sử dụng tính chất kiểm tra khoảng không thời gian x2 −c2t2 bất biến
với phép quay hyperbole (9) Mặt khác, kiểm tra phép biến đổi Lorentz cơng thức (1) phép quay hyperbole với tham số quayθxác định qua vận tốc tương
đối hai hệ quy chiếu
tanhθ = sinhθ coshθ =
v
c (14)
Trong vật lý, phép quay Lorentz gọi phép némdo biến đổi hệ quy chiếu thành hệ quy chiếu chuyển động với vận tốcvso với
Khơng gian Minkowski
Năm 1907, nhà tốn học Hermann Minkowski, vốn thày dạy toán Einstein, nhận thấy tất công thức lý thuyết tương đối hẹp Einstein, Lorentz Poincaré xây dựng viết lại đẹp đẽ khơng gian chiều với tọa độ sau(x1, x2, x3, x0 =ct) Khoảng
không thời gian bất biến
ds2 =dx21+dx22+dx23−dx20 = (15)
trong đódxµ, µ= 1,2,3,0là chênh lệch tọa độ không thời gian hai kiện
So với không gian Euclide chiều R4, hình học khơng thời gian có thay đổi dấu trừ
trong định nghĩa khoảng bất biến Như thấy, áp dụng vào thực tiễn, dấu trừ định nghĩa khoảng bất biến đảo lộn nhiều nhận thức hàng ngày Có cách quan niệm khác: Nếu thời gian số ảo, có bất biến hình học Euclide Nói rộng ra: thời gian chiều không gian ảo ngược lại không gian chiều thời gian ảo Như số ảo tồn giới thực Khiên cưỡng với số thực phi thực tế Học Tốn để có tư cởi mở linh hoạt khơng phải để tự trói vào kiến thức quen thuộc lạc hậu
(29)biến thiên theo thời gian học lượng tử tảng cho phương trỡnh Schrăodinger ni ting mụ t cỏc hin tng lng tử giới vi mơ Điều hàng chục năm sau Minkowski nhà vật lý hiểu rõ
Một vẻ đẹp hình học Minkowski việc mô tả lý thuyết điện từ Maxwell cách thống Như biết, lý thuyết điện từ nhà vật lý người Scottland James Clerk Maxwell phát vào năm 1865 Lý thuyết bao gồm tất định luật điện từ Lý thuyết tiên đoán tồn sóng điện từ có ứng dụng thực tế rộng rãi ngày Ánh sáng loại sóng điện từ đặc biệt Trong lý thuyết Maxwell có hai đại lượng làtừ trườngB~ = (Bx, By, Bz)vàđiện trường E~ = (Ex, Ey, Ez) Câu hỏi đặt ra
là làm mô tả điện trường từ trường hình học Minkowski, vector hình học phải có chiều?
Khác với trường hợp năng-xung lượng, mô tả điện trường từ trường không đơn giản thêm vào thành phần thứ Trong thực tế, khơng có đại lượng vật lý ứng với thành phần thứ tư điện trường từ trường
Người ta tìm vector chiều hình học MinkowskiA~ = (A1, A2, A3, A0) gọi là
vector thế điện từ Khi điện trường từ trường biểu diễn qua vector điện từ sau
~
E = (F01, F02, F03), ~B = (F12, F23, F31)
Fµν =−Fνµ =∂µAν −∂νAµ, (16)
trong đóµ, ν = 1,2,3,0và∂µlà đạo hàm theo biến thứµ Các phương trình Maxwell, vốn
khó nhớ viết thành dạng đơn giản đẹp đẽ tương tự phương trình sóng điện tửAµnhư sau
( ∂
∂x2 + ∂2 ∂y2 +
∂2 ∂z2 −
1 c2
∂2
∂t2)Aµ = (17)
Điều đáng nói hình học Minkowski, điện trường từ trường thống với thông qua vector điện từ trường chiều
(30)được năm, không kịp chứng kiến đời cơng trình khoa học vĩ đại nhân loại học trò khám phá
Ngày cách mơ tả đại lượng vật lý thông qua vector chiều Minkowski trở nên phổ biến rộng rãi vật lý Nhờ đó, Dirac tìm phương trình mô tả electron positron hạt vật chất Cũng nhờ đó, Yang Mills phát phương trình Yang-Mills mơ tả tương tác vật chất
Hình học khơng thời gian thực tế
Công thức lượng Einstein
Việc thống lượng xung lượng không gian Minkowski có hệ vơ quan trọng Tương tự bất biến khoảng không thời gian, năng-xung lượng liên quan tới đại lượng bất biến khối lượng thông qua công thức
p2 =p21+p22+p23−E2 =−m2c4 (18)
Chúng ta khơng tìm cách dẫn cơng thức theo cách mà Einstein nhà toán học vật lý đồng thời với ông làm Các cách dẫn chứa đựng nhiều quan điểm khác nhau, kể sai sót, cịn tranh luận ngày [2]
Tuy nhiên, người quan tâm tới sở tốn học chặt chẽ cơng thức xung lượng (18) thấy khối lượng bất biến Casimir lý thuyết biểu diễn nhóm Poincaré E.P.Wigner [3] Chính thế, khối lượng đặc trưng
Chúng ta viết lại công thức xung lượng hệ tọa độ Descartes(x, y, z)dưới dạng E =qm2c4+p2
x+p2y +p2z (19)
Khi xung lượng bằng0, có cơng thức lượng tiếng Einstein
E =mc2 (20)
Công thức có ý nghĩa vơ quan trọng:Do khối lượng vật chất tương đương với lượng, khối lượng giải phóng lượng vơ lớn Đó sở lượng nguyên tử
(31)Hình 5: Bom nguyên tử Hiroshima Nagasaki
Giải phóng lượng hai trường hợp khối lượng bị mát theo công thức khối lượng Einstein
Công thức khối lượng thành tựu vĩ đại để giải vấn đề lượng cho lồi người, tiếc thay, cơng thức bị lạm dụng để làm loại vũ khí hủy diệt chưa thấy lịch sử Đó vết nhơ lịch sử lồi người Điều cho thấy, để vào ứng dụng thực tế, nhà khoa học cần có tảng đạo đức vững bên cạnh kiến thức khoa học uyên thâm
Quan niệm thời gian khoảng cách
Trong hình học khơng thời gian, khái niệm khơng gian thời gian hòa trộn, quan niệm khoảng cách thời gian tuyệt đối mà thừa nhận chân lý hiển nhiên không cịn
Trước hết tính đồng thời Hai kiện gọi đồng thời xảy thời điểm Do thời gian hệ quy chiếu khác khác nhau, nên hai kiện coi đồng thời hệ quy chiếu khơng cịn đồng thời hệ quy chiếu khác Điều tương tự có hai kiện đồng thời xảy thành phố Hồ Chí Minh Hà Nội, người chuyến máy bay từ Hà Nội thành phố Hồ Chí Minh ngược lại kiện xảy trước kiện
(32)Tuy nhiên, nh˙ng ng˜Ìi quan tâm tĨi cÏ s tốn hÂc ch∞t ch≥ cıa cơng th˘c n´ng xung l˜Ịng (18) có th∫ thßy khËi l˜Ịng bßt bi∏n Casimir trong l˛ thuy∏t bi∫u diπn nhóm Poincaré cıa E.P.Wigner [3] Chính th∏, khËi l˜Ịng ∞c tr˜ng
Chúng ta vi∏t l§i cơng th˘c n´ng xung lềng trong ta ẻ Descartes(x, y, z) dểi d§ng
E = qm2c4+p2
x +p2y +p2z (19)
Khi xung lềng băng 0, chỳng ta cú cụng th˘c
n´ng l˜Ịng nÍi ti∏ng cıa Einstein
E =mc2 (20)
Cơng th˘c có mỴt ˛ nghỉa vơ quan trÂng:Do khËi l˜Ịng cıa v™t chßt t˜Ïng ˜Ïng vĨi n´ng l˜Ịng, khËi l˜Ịng mßt i s≥ gi£i phóng ra mỴt n´ng l˜Ịng vơ lĨn ó cÏ s cıa n´ng l˜Ịng ngun t˚.
N´m 1938, ng˜Ìi ta ã phát hiªn n´ng l˜Ịng ˜Ịc gi£i phóng rã h§t nhân ngun t˚ uran b‡ b≠n phá bi h§t neutron Bên c§nh ó, trong cỏc phÊn ng tng hềp cỏc hĐt nhõn nhà thnh hĐt nhõn nng, cng cú mẻt lềng nng lềng lển gòp bẻi ềc giÊi phúng.
GiÊi phúng nng l˜Ịng c£ hai tr˜Ìng hỊp ∑u khËi l˜Ịng b‡ mßt mát theo cơng th˘c khËi l˜Ịng cıa Einstein.
Cụng thc khậi lềng l thnh táu vổ Đi l≥ chø ∫ gi£i quy∏t vßn ∑ n´ng l˜Ịng cho lồi ng˜Ìi, nh˜ng ti∏c thay, cơng th˘c b‡ lĐm dng lm cỏc loĐi v khớ hy diêt ch˜a t¯ng thßy l‡ch s˚.
ó v∏t nhÏ l‡ch s˚ lồi ng˜Ìi i∑u ó cho thßy, ∫ i vào ˘ng dˆng th¸c t∏, nhà khoa hc cản cú nn tÊng Đo c vng chc bờn cĐnh kin thc khoa hc uyờn thõm.
Quan niêm mĨi v∑ thÌi gian kho£ng cách Trong hình hÂc khơng thÌi gian, khái niªm khơng gian thèi gian ềc hũa trẻn, cỏc quan niêm v khoÊng cách thÌi gian tuyªt Ëi mà chúng ta th¯a nh™n nh˜ chân l˛ hi∫n nhiên khơng cịn
úng n˙a.
Tr˜Ĩc h∏t tính Áng thÌi Hai sá kiên ềc gi l ng thèi nu xÊy tĐi cựng mẻt thèi im. Do thèi gian hª quy chi∏u khác là khác nhau, nên hai sá kiên ềc coi l ng thèi
Hỡnh 5.Bom ngun t˚ t§i Hiroshima Nagasaki
Hình 6.Tính ng thèi ph thuẻc vo quy chiu trong mẻt hª quy chi∏u s≥ khơng cịn Áng thÌi trong quy chiu khỏc iu ú cng tẽng tá nh cú hai sá kiên ng thèi xÊy tĐi thnh phË HÁ Chí Minh Hà NỴi, nh˜ng Ëi vĨi ng˜Ìi chuy∏n máy bay t¯ Hà NỴi i thành phậ H Chớ Minh hoc ngềc lĐi thỡ mẻt sá kiên s xÊy trểc sá kiên kia.
Trong Hình 6, có mỴt tia sáng chi∏u t¯ gi˙a toa tàu ang chuy∫n Ỵng Ng˜Ìi quan sát ˘ng trên tu s thòy tia sỏng chiu n ảu tu v ci tàu Áng thÌi Trong ó, ng˜Ìi ˘ng sân ga s≥ thßy tia sáng ∏n ci tàu sĨm hÏn quãng ˜Ìng ánh sáng ph£i i ng≠n hÏn.
BĐn c cú th tá nghổ ròt nhiu tình hng l˛ thú tính Áng thÌi b‡ vi phĐm.
Do thèi gian cỏc quy chiu khác là khác phép bi∏n Íi Lorentz, khơng nh˙ng tính Áng thÌi b‡ vi ph§m mà thÌi trơi i hª quy chi∏u có th∫ dài ho∞c ng≠n hÏn thÌi gian trơi i hª quy chi∏u Ng˜Ìi ta có mỴt k‡ch b£n gi£ t˜ng v∑ hai anh em sinh ơi, sËng hai hª quy chi∏u khác nhau, ng˜Ìi s≥ già hÏn ng˜Ìi kia gp lĐi.
Chớnh vỡ th cõu chuyên T¯ Th˘c g∞p tiên, khi tr v∑ quê nhà, nh˙ng ngèi thõn u ó qua èi, vđn cú phản thác t∏ hình hÂc khơng thÌi gian.
Hình 6: Tính đồng thời phụ thuộc vào hệ quy chiếu
Bạn đọc tự nghĩ nhiều tình lý thú tính đồng thời bị vi phạm
Do thời gian hệ quy chiếu khác khác phép biến đổi Lorentz, tính đồng thời bị vi phạm mà thời trơi hệ quy chiếu dài ngắn thời gian trôi hệ quy chiếu Người ta có kịch giả tưởng hai anh em sinh đôi, sống hai hệ quy chiếu khác nhau, người già người gặp lại
Chính câu chuyện Từ Thức gặp tiên, trở quê nhà, người thân qua đời, có phần thực tế hình học khơng thời gian Nếu cõi tiên Từ Thức hệ quy chiếu chuyển động, ông thấy thời gian ngắn so với hệ quy chiếu gắn với quê hương, nơi có người thân ông sinh sống
Độ dài thay đổi hệ quy chiếu khác Người quan sát thấy vật chuyển động ngắn lại nhờ phép biến đổi Lorentz Điều đáng ý thời gian bị đảo lộn hệ quy chiếu, hình học khơng thời gian, tính nhân khơng bị đảo lộn Nếu kiện A hệ kiện B, hệ quy chiếu kiện A hệ kiện B Do đó, hệ quy chiếu, người quan sát phải luôn thấy cha sinh trước
Công nghệ GPS thuyết tương đối
(33)Hình 7: Cơng nghệ định vị vệ tinh
giây ngày đêm Do quỹ đạo vệ tinh cách mặt đất chừng 20.000km, lực hấp dẫn vệ tinh nhỏ mặt đất lần Các hiệu ứng hấp dẫn lại làm đồng hồ mặt đất chạy nhanh 45 micro giây ngày đêm Như tổng số, đồng hồ mặt đất chạy nhanh đồng hồ vệ tinh 38 giây ngày đêm
Để đo vị trí vật mặt đất cơng nghệ GPS xác tới 15m, sai số đo thời gian phải 50 nanơ giây Nếu khơng tính tới hiệu ứng thuyết tương đối, ngày đêm sai số tích lũy khoảng 10km, số lớn làm sai lệch việc đo khoảng cách GPS
Ngày nay, hiệu chỉnh thời gian với hình học khơng thời gian hình học Riemann bao gồm hiệu ứng hấp dẫn quan trọng công nghệ GPS
Mở rộng hình học khơng thời gian
Năm 1915, Einstein ứng dụng hình học Riemann vào vật lý cách mở rộng hình học khơng thời gian bỏ qua điều kiện bất biến khoảng khơng thời gian Hình học mơ tả hình thành vũ trụ tương tác hấp dẫn từ khoảng cách xa hàng triệu năm ánh sáng Tuy nhiên, chưa phải giới hạn cuối mở rộng Ngày nay, hình học Riemann mở rộng thêm chiều không gian chiều thời gian bắt đầu nghiên cứu Bên cạnh tính chất hình học độ cong, độ xoắn tính chất topo khơng thời gian đem lại nhiều quan niệm mở chân trời ứng dụng cho khoảng cách lớn Thiên hà mà khoảng cách vô bé, nơi quan niệm không thời gian phải thay đổi nhiều
(34)[1] Nguyễn Ái Việt,Cấu trúc không thời gian: Tập 1.Thuyết tương đối hẹp đối xứng không thời gian(sẽ xuất bản)
[2] E.Hecht, American Journal of Physics,79(6) (2011) 591–600
(35)GIỚI THIỆU
Bài viết trích từ viết https://damtson.wordpress.com/2016/10/ 22/euler-formula/của tác giả
Giải Nobel vật lý năm trao cho ba nhà vật lý, Thouless, Haldane Kosterlitz, đóng góp liên quan đến chuyển pha trạng thái tôpô Nhân dịp dùng vật lý để chứng minh công thức tiếng, liên quan đến tôpô - công thức Euler cho đa diện Công thức nói với đa diện bất kỳ, số đỉnhV;số mặtF số cạnhE
thoả mãn
V CF E D2:
Ví dụ với hình lập phương ta cóV D8; F D6; E D12;và8C6 12D2:Bạn kiểm
tra với vài hình đa diện để thấy công thức
Để chứng minh công thức này, ta lắp mạch điện theo hình đa diện, thay cạnh đa diện điện trở Không quan trọng giá trị điện trở bao nhiêu, miễn tất điện trở khác không Để cho đơn giản ta cho điện trở là1!:Sau ta chọn
(36)Khi ta nối mạch điện vậy, tất nhiên điện chạy mạch cách định Ta đặt nhiều câu hỏi với mạch điện Ví dụ ta hỏi điện trở mạch Câu hỏi hỏi sau: Giả sử tổng dòng điện chạy qua mạch là1Amper, dòng điện chạy qua
từng điện trở bao nhiêu? (Tất nhiên trả lời câu hỏi tìm điện trở mạch)
Để trả lời câu hỏi trên, ta lập hệ phương trình cho phép ta tìm dịng điện chảy qua điện trở Giả sửAB cạnh, ta ký hiệuIAB dịng điện chạy từ đỉnhAđến đỉnhB: Ta cóIAB D IBA;và có tổng cộngE đại lượng Ta lập hệ phương trình để tìm giá trị dịng điện
Có hai loại phương trình, xuất phát từ hai định luật Kirchhoff Loại sau Giả sử
Alà đỉnh, vàB; C; D; : : :là đỉnh kềA:Ta có phương trình: IABCIAC CIADC D0hoặc1hoặc 1:
Vế phải là0nếu đỉnhAkhông phải hai đỉnh nối vào nguồn điện, là1nếuAđược
nối vào cực dương 1nếuAnối vào cực âm Đơn giản phương trình nói dịng điện chạy
vào đỉnh phải dòng chạy từ
Ta có tổng cộng phương trình này? Đếm thấy tổng cộng V phương
trình, thực chúng khơng độc lập với Có thể thấy điều cách lấy tổng tất phương trình Ta đồng thức D 0; vế trái với IAB
cũng cóIBA:Vế phải tất nhiên tổng là1C 1/cộng nhiều số0;cũng không Như cóV 1phương trình độc lập
Nhưng phương trình khơng phải tất phương trình ta phải viết Có loạt phương trình khác (phương trình loại hai) Ta giả sửABCDlà mặt (ta cho tứ giác
ở logic với đa giác) Ta có phương trình
(37)phải số ẩn
(38)GIẢ THUYẾT KEPLER VÀ BÀI TOÁN XẾP CAM
Dương Đức Lâm
University of Sussex, United Kingdom
GIỚI THIỆU
Johannes Kepler nhà toán học, nhà thiên văn học tiếng người Đức, người có đóng góp tảng góp phần tạo nên cách mạng khoa học kỉ 17 Ngoài định luật chuyển động hành tinh nhiều người biết đến, ơng cịn có nhiều đóng góp khác cho tốn học, số giả thuyết Kepler, toán cổ xưa toán học Bài viết đề cập số vấn đề xoay quanh giả thuyết Kepler mở rộng nó, tốn xếp cam nhiều chiều, tiến đạt thời gian gần
1 Giả thuyết Kepler vài nét lịch sử
Rất nhiều tốn khó lâu đời toán học phát biểu đơn giản Những ví dụ điển hình tốn bốn màu (được Francis Guthrie phát biểu lần đầu vào năm 1852), định lí cuối Fermat (được ghi lại năm 1637 bên lề sách Diophantus) hay nhiều giả thuyết liên quan đến số nguyên tố Một ví dụ tiếng khơng kém, chí cịn lâu đời hơn, giả thuyết Kepler, Johannes Kepler đưa vào năm 1611
Câu chuyện bắt đầu vào năm cuối thập kỉ 90 kỉ 16 [4] Sir Walter Raleigh, thủy thủ quý tộc người Anh, chuẩn bị cho chuyến thám hiểm, hỏi người bạn trợ lý nhà tốn học Thomas Harriot, làm để tính số lượng đạn cối chứa giỏ Harriot đưa lời giải khơng khó khăn Tuy nhiên, vốn nhà tốn học, ơng đưa câu hỏi tổng quát rằng, phải xếp để lượng đạn chiếm phần khơng gian
Sau chuyến đi, Harriot viết thư trao đổi với Kepler, đồng nghiệp ơng lúc Praha (Cộng hịa Czech) Sau thời gian nghiên cứu toán, Kepler xuất sách nhỏ tựa đề "Strena Seu de Nive Sexangula1" (1611), mơ tả hai cách xếp hình
cầu khơng gian ba chiều: xếp theo kiểu kim tự tháp (hay kiểu lập phương tâm diện), xếp theo kiểu lục giác (xem Hình 1) Quan trọng hơn, ơng nói rằng, hai cách xếp chặt có thể, nhiên lại khơng đưa chứng minh
(39)Hình 1: Hai cách xếp cam chặt (Nguồn ảnh: Wikipedia and Plus Maths.) Với cách xếp này, độ chiếm không gian cam
1
3√2 '74%
Giả thuyết Kepler nói rằng, tất cách xếp khác có mật độ chiếm khơng gian khơng lớn số
Có thực tế rằng, cách xếp trái quầy bán hoa hay cách xếp, chồng chất hàng hoá, đóng gói vật dụng có dạng hình cầu nói chung tuân theo quy tắc này2 (xem Hình 2) Bởi mà giả thiết Kepler gọi tốn xếp cam hay tốn
đóng gói hình cầu (sphere packing problem)
2 Thomas Hales hành trình tìm kiếm lời giải
Khác với số toán tiếng khác, chẳng hạn toán bốn màu, mà nhiều năm liền chủ yếu nhận quan tâm tìm kiếm lời giải từ nhà toán học "amateur", giả thuyết Kepler nhiều nhà tốn học hàng đầu ý, có Newton Gauss [5] Nó Hilbert đưa vào danh sách 23 toán tiếng đại hội toán học giới năm 1900 nằm vị trí số 18
2Điều minh hoạ cho quy luật tối ưu tự nhiên: tượng tự nhiên ln hướng đến trạng
(40)Hình 2: Cách xếp hoa quầy hàng tuân theo giả thuyết Kepler (Nguồn ảnh: New Scientist.)
Năm 1990, nhà toán học Wu-Yi Hsiang Đại học Berkeley, California tuyên bố chứng minh giả thuyết kết sau xuất tạp chí International Journal of Mathematics (1993) Tuy nhiên, nhà tốn học, có Thomas Hales Đại học Michigan, sớm tìm lỗ hổng chứng minh Hsiang không chấp nhận kết
Những năm sau đó, Hales xây dựng chương trình nhằm chứng minh giả thuyết Kepler với trợ giúp máy tính Đến năm 1998, Hales thơng báo hồn thành chứng minh gửi thảo cho tạp chí Annals of Mathematics, cộng đồng toán học thừa nhận rộng rãi tạp chí uy tín tốn, cơng bố cơng trình tốt Trong thơng báo sau Hales Notices of AMS, hội đồng bình duyệt gồm 12 người thành lập để kiểm tra tính đắn chứng minh, điều có tiền lệ việc bình duyệt tạp chí, thơng thường việc bình duyệt báo có hai người, người bình duyệt thường giấu tên
Tuy nhiên, với riêng trường hợp điều dễ hiểu chứng minh Hales dài tận 250 trang, chưa kể gigabytes liệu tính tốn kèm theo Nhất sau có tranh cãi xảy với chứng minh chưa hoàn thiện Hsiang, họ phải cẩn thận Quá trình bình duyệt kéo dài tới tận năm, sau họ phải bỏ "hết lượng" Tạp chí Annals of Mathematics cuối nhận đăng kèm theo thích rằng, bình duyệt viên khơng thể kiểm tra tính đắn chứng minh, chắn khoảng 99% Một điều chưa có tiền lệ! Và tính thời gian từ lúc ban biên tập nhận thảo (9/1998) đến lúc nhận đăng (8/2005) lên tới tận năm, điều thấy!
Khơng lịng với chậm trễ lẫn kết luận ban biên tập, Hales tiếp tục xây dựng chương trình gọi "Flyspeck"3 để kiểm tra tất bước logic chứng minh máy
tính Đây chương trình hợp tác tồn cầu, có phần thực Viện Toán học, Hà Nội (xem [3])
Sau 10 năm nỗ lực khơng biết mệt mỏi, chương trình đến lúc hái trái Vào đầu tháng 8/2014, trước thềm Đại hội Toán học Thế giới diễn Seul, Hales thơng báo thức Flyspeck hoàn thành Giả thuyết Kepler chứng minh cách hình thức
(41)Hình 3: Thomas Hales (Nguồn ảnh: Balin Thirling.)
(42)Bài toán tương ứng với giả thuyết Kepler, hay tốn xếp cam, khơng giannchiều
dành nhiều quan tâm Gọi ρn mật độ lớn cách xếp cam không
giannchiều Theo giả thuyết Kepler ρ3 =
1
3√2 '74%
Dưới đây, ta tìm hiểu chút trường hợp khin6=
3.1. n= 1 hoặc 2
Trong không gian hay hai chiều, toán giải tương đối dễ dàng Trường hợp chiều khơng có để bàn ("không gian" đường thẳng, "quả cam" đoạn thẳng),ρ1 = Trong trường hợp hai chiều ("không gian" ứng với mặt phẳng, "quả cam"
ứng với hình trịn), xét hai cách xếp Hình (a) (b) Có thể tính tốn khơng khó khăn mật độ chiếm khơng gian hình trịn hình (a) π
4 '79% Cịn hình (b), mật độ
này lớn
ρ2 = π
2√3 '91%
Hình 4: Bài tốn xếp cam hai chiều
Thật vậy, điều chứng minh dựa vào ý tưởng "chia để trị", biến mặt phẳng thành miền địa phương, miền, chứng minh mật độ chiếm khơng gian hình trịn khơng vượt qρ2 Để đơn giản, xét miền địa phương với ba hình trịn tiếp xúc
như Hình
Trong miền tam giác tạo thành từ tâm ba hình trịn, có phần trống khơng phủ Nếu ta mở rộng hình trịn cách tăng bán kính chúng lên khoảngc= √2
3
thấy, khơng khó khăn, hình trịn phủ kín phần trống chúng
Bằng cách mở rộng hình tròn khoảngcmột cách tương tự cho miền bất kì,
(43)Hình
3.2. n > 3
Bây giờ, câu chuyện khác hẳn số chiều tăng lên, có phần phức tạp Vào ngày 14 tháng đầu năm nay, nhà toán học trẻ người Ukraina, Maryna Viazovska, gây xơn xao cộng đồng tốn học đưa lên arXiv chứng minh độc đáo giải toán xếp cam trường hợpn= Kết mà Viazovska đưa cho trường hợp
ρ8 = π
24.4! '25.37%
Terence Tao, Đại học California, Los Angeles, nói chứng dài 23 trang Viazovska: "Hai số điều tơi thích chứng minh là: (a) sử dụng kĩ thuật của giải tích Fourier để nhận số tốt nhất, (b) nhân tử Fourier tối ưu xây dựng nhờ sử dụng dạng modular, điều mà ngờ lại có liên quan tới tốn xếp cam." Cịn Peter Sarnak, Đại học Princeton viện IAS, nhấn mạnh: "Chứng minh đơn giản một cách đáng kinh ngạc, tuyệt vời có thể!"
(44)trong thành tựu đáng ý Hình học tổ hợp năm Và lời Thomas Hales nói, điều mà chờ đợi
4 Những chân trời rộng mở lời kết
Giả thuyết Kepler giải, số kết riêng trường hợp chiều cao tìm Vậy điều lại?
Lời giải Thomas Hales cho giả thuyết Kepler, dù chứng minh đúng, chưa làm hài lịng nhà tốn học Bởi điều mà Paul Erdos hay tin tưởng, tốn có lời giải đẹp đẽ chứa sách Chúa Trời Vậy lời giải đẹp đẽ (và không nhờ trợ giúp máy tính) giả thuyết Kepler liệu có tồn tại? Nếu tồn tại, gì?
Ngoài ra, quay lại với cách xếp cam giả thuyết Kepler, để đạt cách xếp tối ưu nhất, phải xếp chúng cách cẩn thận theo cách Nhưng giả sử, vội ném cam vào hộp cách nhanh Lúc cam rơi ngẫu nhiên vào hộp Vậy mật độ chiếm thể tích hộp cam bao nhiêu? Đây tốn đóng gói ngẫu nhiên Các kết xoay quanh tốn cịn nhiều chuyện đáng bàn
Bây giả sử ta không xếp cam nữa, mà xếp đồ vật có dạng hình khối khác Lúc mật độ chiếm thể tích chúng bao nhiêu? Đương nhiên, kết phụ thuộc vào hình dạng vật xếp Trong [1], tác giả đạt số trường hợp riêng cho trường hợp vật xếp có dạng ellipsoid Tuy nhiên, kết khiêm tốn
Rõ ràng, với việc tìm kiếm lời giải cho tốn xếp cam chiều cao cịn lại (hiện giải chon = 1,2,3,8và 24), xoay quanh tốn cịn nhiều điều thú vị chờ
chúng ta khám phá Hy vọng, sau thành tựu gây nức lòng Viazovska, sớm biết thêm nhiều điều bí ẩn xung quanh toán
Tài liệu
[1] A Donev, I Cisse, D Sachs, E Variano, F H Stillinger, R Connelly, S Tarquato and P.M Chikin, Improving the density of jammed disordered packings using ellipsoids,Science, 303 (2004), 990–993
[2] Frank Morgan, Kepler’s Conjecture and Hales’s Proof, A Book review, Notices of AMS, Vol 52, Issue 1, 2005
(45)[9] http://blog.kleinproject.org/?p=742
(46)THI TRẮC NGHIỆM GÂY NHIỀU TRANH CÃI Ở MỸ
Neal Koblitz
(Đại học Washington)
Người dịch: Hảo Linh LỜI DẪN
Dù có nhiều ý kiến nặng ký (trong có ý kiến Hội tốn học Việt Nam) phản đối việc áp dụng hình thức thi trắc nghiệm hầu hết môn thi tốt nghiệp THPT, Bộ Giáo dục Đào tạo định triển khai phương án “theo lộ trình” Tạp chí Tia sáng có đăng viết giáo sư Neal Koblitz, nhà tốn học Mỹ có nhiều dun nợ với Việt Nam nêu ý kiến vấn đề sở kinh nghiệm ông Mỹ, nơi trắc nghiệm thường sử dụng kỳ thi đánh giá lực cấp độ Được đồng ý dịch giả Tịa soạn, chúng tơi đăng lại viết để bạn đọc Epsilon tham khảo thêm
Một hạn chế đề thi trắc nghiệm thất bại việc chuẩn bị cho học sinh đối diện với phương thức giải vấn đề mà họ bắt gặp lớp toán, khoa học nghề nghiệp tương lai
(47)định kỳ Lý việc chấm điểm so sánh tương quan dễ dàng thực máy tính Tuy nhiên, việc sử dụng kết kiểm tra để đánh giá giáo viên trường học thay học sinh Một trường học giáo viên có nhiều học sinh học mức trung bình khơng tăng điểm qua năm bị “phạt” nhiều cách số trường hợp, trường học bị đóng cửa
Việc sử dụng thi trắc nghiệm gây nhiều tranh cãi Mỹ bị phản ứng mạnh mẽ hiệp hội giáo viên chuyên gia giáo dục Ở trường hàng đầu, phụ huynh giáo viên tin họ người đánh giá tốt việc học học sinh không cần phải nhờ trợ giúp kiểm tra trắc nghiệm Kết là, họ từ chối triển khai kì thi trắc nghiệm Chính sách đánh giá giáo viên trường học dựa kết học sinh làm đề kiểm tra trắc nghiệm bắt đầu từ15năm trước suốt nhiệm kỳ Tổng thống
Bush kéo dài suốt nhiệm kỳ Tổng thống Obama Chính sách tạo áp lực lớn lên giáo viên việc “dạy để thi” ngăn cản việc dạy sâu, dạy theo bối cảnh Trong nhiều trường hợp, cịn dẫn đến hành vi sai trái gian lận (giáo viên ban giám hiệu sửa đáp án học sinh nhằm tăng điểm cho em) Hầu hết nhà khoa học giáo dục tin sách thất bại chất lượng trường phổ thông Mỹ không cải thiện mà chí cịn giảm sút suốt thời kì Trong sách bán chạy xuất vào năm 2010; chuyên gia giáo dục Diane Ravitch, cán giáo dục hàng
đầu thời kỳ đương nhiệm Tổng thống Bush ủng hộ kỳ thi trắc nghiệm, thay đổi quan điểm Bây bà người dẫn đầu nhóm phản đối việc sử dụng kỳ thi trắc nghiệm ngày phổ biến
2 Không dựa kết thi trắc nghiệm để tuyển sinh đại học
Ở Mỹ, kiểm tra đánh giá lực (Scholastic Aptitude Test – SAT1) sử dụng một
phần trình tuyển sinh đại học Ban đầu SAT hoàn toàn kiểm tra trắc nghiệm khách quan Nhưng cách khoảng7 10năm, nhiều trường ngưng sử dụng kết SAT cho
việc tuyển sinh họ cảm thấy thi trắc nghiệm khơng phải cách hữu ích để lựa chọn sinh viên Từ đó, cơng ty đề SAT, Công ty dịch vụ kiểm tra đánh giá giáo dục ETS
1SAT kỳ thi dùng để đánh giá lực học tập học sinh, bao gồm ba phần: Đọc hiểu (critical reading),
(48)Một điều quan trọng cần nhấn mạnh, trường đại học Mỹ không dựa kết thi cử để tuyển sinh Các cán tuyển sinh trường đại học tin thông qua việc sử dụng nhiều công cụ, họ có cách đánh giá ứng viên tốt dựa vào kết kỳ thi đầu vào
Có thể nói, khó để thiết kế thi trắc nghiệm công hữu ích Bản thân ETS đầu tư số tiền lớn năm để thiết kế đề thi trắc nghiệm Khi thi trắc nghiệm thiết kế tốt, có hai lợi thế.1/nó tốn kinh phí để chữa chấm điểm
vì thực máy, và.2/nó sử dụng để loại trừ học sinh
có lực yếu Có nghĩa là, học sinh có kết tệ thi trắc nghiệm khó ứng cử viên cho trường đại học hàng đầu Tuy nhiên, kỳ thi trắc nghiệm lựa chọn sinh viên xuất sắc
Một hạn chế đề thi trắc nghiệm thất bại việc chuẩn bị cho học sinh đối diện với phương thức giải vấn đề mà họ bắt gặp lớp toán, khoa học nghề nghiệp tương lai Trên thực tế, nhà toán học nhà khoa học không giải vấn đề đáp án ngắn Ngược lại, vấn đề đòi hỏi đáp án dài, cần nhiều bước giải quyết, sinh viên phải đưa lời giải đầy đủ họ chuẩn bị tốt cho nghề nghiệp lĩnh vực khoa học công nghệ
Ở cấp học cao hệ thống giáo dục, thi trắc nghiệm sử dụng Một đại học tốt không dựa vào đề thi trắc nghiệm để định tuyển sinh Tuy nhiên, đề thi trắc nghiệm có ích trình độ cao, ví dụ đề thi sau đại học GRE (Graduate Record Exam) Toán học – để xác định loại sinh viên có học lực yếu
Bởi vậy, Chính phủ Việt Nam cần phải giảm thiểu chi phí chấm điểm thi đầu vào đại học có cách hợp lý để làm điều đó, thỏa hiệp khơng ảnh hưởng xấu đến trình tuyển chọn Đề thi tuyển sinh có hai phần: Một phần trắc nghiệm phần tự luận Những học sinh có kết thấp mức điểm sàn phần thi thứ không đỗ không cần chấm phần thi thứ hai họ Điều có nghĩa là, phần thi xác định học sinh yếu phần thứ hai để xác định học sinh xuất sắc Điều tiết kiệm thời gian chi phí mà khơng phải đánh đổi chất lượng trình tuyển sinh
3 Con người chính, máy móc thứ yếu
(49)Neal Koblitz giáo sư Toán học Đại học Washington giáo sư danh dự Viện nghiên cứu Mật mã Ứng dụng Đại học Waterloo Neal Koblitz cha đẻ mật mã đường cong elliptic siêu elliptic - hai đóng góp quan trọng ngành mật mã đại Năm1985;ơng vợ mình, Ann Hibner Koblitz sáng lập
ra giải thưởng Kovalevskaia để tôn vinh nhà khoa học nữ nước phát triển Ông tới Việt Nam giảng dạy nhiều lần trường đại học viện nghiên cứu Toán học
(50)BÀI TOÁN CHỨNG MINH TRUNG ĐIỂM VÀ CÁC MỞ RỘNG
Trần Quang Hùng (THPT chuyên KHTN, Hà Nội)
TÓM TẮT
Bài viết mở rộng phát triển tốn hình học hay chứng minh trung điểm sử dụng cơng cụ hình học túy hàng điểm điều hịa
Bài tốn đẹp tiếng lần xuất [1] Sau thời gian ngắn [2,3] đề cập tới có nhiều lời giải khác dùng phương pháp xạ ảnh túy hình học Gần [4] nhắc lại tốn với mở rộng hay Bài tốn sau
Bài tốn 1. Cho tam giácABCnội tiếp đường trịn(O).ADlà đường kính của(O) Tiếp tuyến
tạiDcủa(O)cắtBC tạiT OT cắtCA, AB tạiE, F Chứng minh O trung điểm EF
Tôi đưa lời giải túy hình học cho toán
A
B
C O
M
D
T E
F
(51)E, F Chứng minh rằngP trung điểm củaEF
Mở rộng đơn giản, lời giải hồn tồn tương tự lời giải tốn gốc
A
B C
O
D P
M
Q
T E
F
Hình
Lời giải. Gọi M trung điểm BC, AD đường kính (O) nên dễ thấy ∠P M T =
∠P QT = 90◦ Từ đó ∠QM C = ∠QP T = ∠AP F và ∠P AF = ∠M CQ Vậy hai tam giác AP F CM Qđồng dạng Tương tự tam giácAP E BM Qđồng dạng Từ P EP F =
P E AP
AP P F =
M Q M B
M C
M Q = VậyP trung điểmEF Ta có điều phải chứng minh
Nhận xét.KhiP trùngO thìQD tiếp tuyến của(O)ta toán ban đầu Với cách giải
đó ta thu tiếp tốn mở rộng sau
Bài toán 3. Cho tam giác ABC nội tiếp đường tròn (O) với D điểm thuộc cung BC
không chứaA Tiếp tuyến tạiDcủa(O)cắtBC T Đường tròn ngoại tiếp tam giác ODT
(52)B C O
D
T P
F
E M
Hình
Lời giải. GọiM trung điểmBC dễ thấy M thuộc đường trịn đường kính OT
đường trịn ngoại tiếp tam giác ODT Từ ta có∠DM C = ∠DP T =∠AP F ∠P AF =
∠M CD Vậy hai tam giácAP F vàCM Dđồng dạng Tương tự tam giácAP EvàBM Dđồng
dạng Từ P E P F =
P E AP
AP P F =
M D M B
M C
M D = VậyP trung điểmEF Ta có điều phải chứng
minh
Tổng qt ta có tốn sau
Bài tốn 4. Cho tam giácABC nội tiếp đường trịn(O)và điểm P AP cắt(O)tạiD
(53)B C
D
Q T
E
Hình
Lời giải. Ta có∠QDC = ∠DP T = ∠AP F ∠P AF = ∠QCD Vậy hai tam giácQCD
và P AF đồng dạng Tương tự hai tam giácQBDvàP AE đồng dạng Từ P EP F = P EAP.APP F =
QD QB
QC QD =
QC
QB Ta có điều phải chứng minh
Nhận xét.KhiQlà trung điểmBCvàP thuộcOQta thu toán Rõ ràng toán sau
này tổng quát, nhiên lại thấy lời giải đơn giản Chúng ta xét tiếp toán sau tác giả đề nghị báo THTT, xin giới thiệu lời giải gốc đề nghị báo
Bài toán 5. Cho tam giác ABC nội tiếp đường tròn (O) với P điểm cung BC
không chứa A (O) Tiếp tuyến P (O) cắt BC T AP cắt đường thẳng qua T
vuông góc OAtại Q M trung điểm AQ T M cắt AB, AC X, Y Chứng minh
(54)B C O
E
T P
Q M
X
Y
R
D
K
L U
V
Hình
Lời giải thứ nhất. GọiADlà đường kính của(O)vàADcắtT QtạiE QuaAkẻ đường thẳng
song songT M cắtT QtạiR DoM trung điểmAQnênT trung điểmQR GọiAB, AC
giao ET K, L AD đường kính (O) nên ∠ACD = 90◦ = ∠DEL, tứ giác DCLE nội tiếp Từ đó∠CLK = ∠CDA = ∠CBA nên tứ giácBCLK nội tiếp DoP T
tiếp tuyến của(O), ta có∠P QT = ∠K+∠P AB = ∠ACB+∠P CB = ∠ACP = 180◦ −
∠ABP = 180◦−∠AP T =∠T P Q Từ tam giácT P Q cân tạiT Kết hợp tứ giácBCKL
nội tiếp ý T P tiếp tuyến (O) ta có T Q2 = T P2 = T B.T C = T L.T K.
Mặt khác T trung điểm QR nên theo hệ thức Newton ta có (LK, QR) = −1 A(LK, QR) =−1 NhưngAQ kT M từ theo liên hệ tỷ số đơn tỷ số képM trung điểm XY Ta có điều phải chứng minh
(55)B C
P N
E
T
Q Y
D
Hình
Lời giải thứ hai. GọiADlà đường kính của(O)vàADcắtT QtạiE QuaAkẻ đường thẳng
song songT M cắtT QtạiR Dễ thấy tứ giácP DEQnội tiếp Từ đó∠Q=∠ADP = 180◦−
∠ABP = 180◦−∠AP T =∠T P Qdo tam giácP T Qcân màM trung điểmAQnênT
trung điểmQR, từ tam giácT P Rcân Ta có tứ giácAP ERnội tiếp nên∠OAP =∠T RP
Vậy hai tam giác cânOAP vàT RP đồng dạng Suy hai tam giácP OT vàP ARđồng dạng
tương ứng Từ đó∠P OT = ∠P AR = ∠P M T Vậy suy M thuộc đường trịn đường kính OT Từ dễ có OM ⊥ XY Gọi N trung điểm BC ta có ∠P N C = ∠P M T =
∠AM X ∠P CN = ∠M AX Vậy hai tam giácAM X CN P đồng dạng Tương tự tam
giácAM Y BN P đồng dạng Vậy M XM Y = M XAM.AMM Y = N PN C.N BN P = Từ đóM trung điểm XY Ta có điều phải chứng minh
Nhận xét. NếuE trùng D ta thu toán Từ lời giải thứ hai, thu
trường hợp riêng đẹp sau toán
Bài toán 6. Cho tam giác ABC nội tiếp đường tròn(O) M trung điểmBC.AM cắt(O)
(56)B
C O
M
P
T
Q E K
L
U V
D
Hình
Lời giải thứ nhất. Tương tự lời giải thứ Gọi ADlà đường kính của(O) Đường thẳng
quaT vng góc vớiADcắtAD, AP tạiE, Q GọiP DcắtQE tạiR Dễ thấy tứ giácP DEQ
nội tiếp Từ đó∠Q = ∠ADP = 180◦−∠ABP = 180◦ −∠AP T = ∠T P Q tam giác P T Q cân mà tam giácQP Rvuông tạiP nênT trung điểm QR, từ tam giácT P Rcân
Ta có tứ giácAP ERnội tiếp nên∠OAP =∠T RP Vậy hai tam giác cânOAP vàT RP đồng
dạng Suy hai tam giác P OT P AR đồng dạng tương ứng Từ đó∠P M T = ∠P OT =
∠P AR Suy raAR k M T vậyM trung điểmAQ Áp dụng toán vào tam giácAKL
suy raM trung diểmU V Ta có điều phải chứng minh
Chúng ta đưa lời giải đơn giản dựa lời giải toán sau
A
B
C O
M
P
T
K
L U
V N
(57)Chúng xin đưa lời giải túy hình học sử dụng ý tưởng từ toán Ta cần bổ đề sau
Bổ đề 7.1. Cho tứ giác ABCD nội tiếp đường tròn (O) AB cắt CD E Các điểm P, Q
thuộcBD, ACsao choAP, BQcùng vng góc vớiOE ThìE, P, Qthẳng hàng
O A
B
C D
I
E
Q P
K
L
Hình
Chứng minh. QuaE kẻ đường thẳng vng góc vớiOE cắtCA, BDtạiK, L Theo tốn
con bướm E trung điểmKL Từ ta có APBQ = APEL.EKBQ = ABEB.EAAB = EAEB Theo định lý
Thales đảo suy raE, P, Qthẳng hàng
(58)D
C O
E F
T M
N
Q
K L
H
G
Hình 10
Lời giải thứ nhất. Dễ thấy tứ giácABEF hình chữ nhật GọiM N cắtAF, BE tạiG, H Olà trung điểmGH Ta chứng minh rằngM G=N H, GọiK, LthuộcDE, F C
choF K, ELcùng vng góc vớiOT Theo bổ đề dễ có K, Lthẳng hàng suy F KEL = T ET F =
EG F H =
HA
GB Ta lại có∠M AH =∠SEF và∠AHM =∠T HF = 90◦−∠HT F =∠SF E Từ
đó ta có hai tam giácAHM EF K đồng dạng Tương tự hai tam giácBGN vàF ELđồng
dạng Từ HM
GN =
HM HA
HA GB
GB GN =
F K F E
EL F K
EF
EL = Từ đóM G=N H suy raOlà trung điểm
GH trung điểmM N Ta có điều phải chứng minh,
(59)D
C
E F
T Q
Hình 11
Lời giải thứ hai. GọiOD cắtN E tạiP Áp dụng định lý Pascal đảo cho
D B E F P C
do
O, N, T thẳng hàng nênP thuộc(O) Từ tứ giácAP EDlà hình chữ nhật suy raO trung
điểmM N
Nhận xét.Khi hai đỉnhAvàBtrùng ta thu lại toán Sau xin đưa
mở rộng thú vị khác cho toán 7, bạn làm luyện tập
Bài toán 8. Cho tứ giác ABCD nội tiếp đường tròn(O) P trung trực AB P A, P B
cắt (O) tạiE, F khác A, B Các điểm M, N thuộcBC, AD cho EM k AD F N kBC.M N cắtAE, BF tạiK, L Chứng minh đẳng giác củaOtrong tam giácP KL
cách đềuM N
Dựa vào ý tưởng tốn 7, chúng tơi đề xuất mở rộng toán sau
Bài tốn 9. Cho tứ giácABCDnội tiếp đường trịn(O).M trung điểmCD.AM, BM cắt (O)tạiE, F khácA, B.EF cắtCD tạiT Một đường thẳng quaT cắt(O)tạiK, L.AK, BL
(60)D
C O
M
F E
T
K
L
U V
S
Hình 12
Lời giải. Gọi KM cắt EV S Áp dụng định lý Pascal đảo cho
S F L A K E
với M, V, T
thẳng hàng suy raSthuộc(O) Từ áp dụng định lý bướm cho tứ giácAKES suy raM
là trung điểmU V Ta có điều phải chứng minh
Bài tốn nhiều ứng dụng thú vị chẳng hạn toán sau, bạn coi luyện tập
Bài toán 10. Cho tam giácABC vuông tạiA.Dlà điểm choCD ⊥BC.M trung điểm BC.DM cắtABtạiE.F thuộcADsao choBF kCE Chứng minh rằngBF ⊥CF
Bài tốn 11. Cho tam giácABC nội tiếp đường trịn(O)có đường caoAD.E, F hình chiếu
củaDlênCA, AB.EF cắtBCtạiG.AGcắt(O)tạiLkhácA.K đối xứngAquaBC Đường
tròn ngoại tiếp tam giácAKLcắtCA, AB tạiM, N khácA
a) GọiP trung điểmM N Chứng minh rằng∠P AC =∠DAB
b) GọiQ, Rlà trung điểmBC, EF Chứng minh rằngP, Q, Rthẳng hàng
Bài toán 12. Cho tam giácABC nội tiếp đường trịn(O)vớiADlà đường kính của(O) Tiếp
tuyến tạiDcủa(O)cắtBCtạiT Tiếp tuyến tạiB, Ccủa(O)cắt tạiS Chứng minh
đường thẳng quaO vng góc vớiOT chia đơiAS
Bài tốn 13. Cho tứ giácABCDnội tiếp đường trịn(O)vớiAE, BF đường kính của(O) EF cắtCDtạiT Tiếp tuyến tạiC, D của(O)cắt tạiS Tiếp tuyến tạiA, B của(O)cắt
(61)[4] Trần Minh Ngọc
(62)SÁNG TẠO TOÁN HỌC
BẰNG PHƯƠNG PHÁP VẬT LÝ HỌC
Nguyễn Ngọc Giang
Chúng ta bắt gặp nhiều viết sáng tạo toán phương pháp toán học Để sáng tạo tốn người ta có nhiều cách thức khái quát hóa, đặc biệt hóa, tương tự hóa, tìm tốn thực tế, tìm nhiều cách giải, tìm tốn đảo, Tuy nhiên, tốn học có mối liên hệ mật thiết với số ngành vật lý học tin học nên toán học chịu chi phối không nhỏ ngành khoa học Có tốn tốn học người ta chưa tìm lời giải tốn học cho mà tìm lời giải Tin học Vật lý Điển toán bốn màu tin học hay vấn đề trọng số Steiner vật lý học Trong viết xin đưa hướng tư tương đối thú vị mẻ sáng tạo Toán học phương pháp Vật lý
Bài toán (Bài toán Héron) Trong mặt phẳng, cho trước đường thẳngd và hai điểmA; B
ở phía đường thẳng Trên đường thẳng cho tìm điểm C sao cho tổng
các khoảng cách từ điểmC đó đến hai điểmA; Blà nhỏ nhất.
So với toán khác, tốn Héron có đặc trưng mà khơng nhiều tốn có Đó tốn Héron có nhiều thể thực tế Chẳng hạn toán sách giáo khoa toán sau ví dụ tiêu biểu:
Bài tốn (Câubbài39trang88;Tốn8tập một) Bạn Tú vị tríA;cần đến bờ sôngd
để lấy nước đến vị tríB:Con đường ngắn mà bạn Tú nên đường nào? Bài tốn (Trang12;Hình học11nâng cao) Người ta tổ chức chạy thi bãi biển
với điều kiện sau: Các vận động viên xuất phát từ địa điểmAvà đích đến địa điểmB;nhưng
trước đếnB phải nhúng vào nước biển (ta giả thiết mép nước biển đường
thẳng) Tìm vị trí bờ biển để tổng khoảng cách từ địa điểmAđến vị trí bờ biển đếnB là
ngắn nhất?
Bài toán Học sinh cắm trại vị tríA;nhưng quy định nấu bếp vị trí B (cùng phía so
với bờ sông) Người nấu bếp phải chọn bến lấy nước vị trí để đường từ trại đến bến rồi đến bếp ngắn (cho bờ sơng thẳng)?
Bài tốn Có hai địa điểmAvàB nằm phía đường xe lửa qua
hai địa điểm Người ta muốn xây dựng nhà ga cho tổng khoảng cách từ địa điểmA
đến nhà ga từ nhà ga đến địa điểmBlà ngắn nhất?
Bài tốn Có hai kho hàng phía đối đường quốc lộ Trên đường quốc lộ người ta
(63)Bài toán Héron tốn quan trọng chương trình giáo dục mơn tốn nước ta Tầm quan trọng thể chỗ, xuất sách giáo khoa toán Trung học sở (lớp8) sách giáo khoa tốn Trung học phổ thơng (lớp11) Chính khai
thác sâu tốn giúp ích nhiều cho thầy giáo, bậc phụ huynh em học sinh có nhìn tương đối đầy đủ
Chúng ta đến với lời giải toán Héron cách tìm hiểu ngun lí Fermat Ngun lí Fermat thường phát biểu đơn giản theo cách sau: Ánh sáng truyền theo đường mà thời gian truyền nhỏ nhất Chính xác hơn, nguyên lí Fermat phát biểu dạng tổng quát là: Quang lộ từ điểm tới điểm khác phải cực trị: Nghĩa hoặc nhỏ nhất, lớn nhất, dừng (độ dài đường truyền nhau)”
Một cách phát biểu dạng khác: “Thời gian truyền ánh sáng từ điểm tới một điểm khác phải cực trị : Nghĩa nhỏ nhất, lớn nhất, không đổi (như nhau đường truyền)”
Như vậy, dựa vào ngun lí Fermat, ta giải tốn động học có dạng: “Tìm quỹ đạo chuyển động vật để thời gian chuyển động nhỏ ?”
Phương pháp giải tốn động học có dạng nguyên lí Fermat hiểu: Chuyển động vật xem truyền ánh sáng
Các miền mà chất điểm chuyển động đóng vai trị mơi trường truyền ánh sáng Khi đó, ranh giới miền (môi trường) xảy tượng phản xạ khúc xạ ánh sáng Áp dụng định luật Quang hình học, nhận lời giải cần tìm tốn
Từ kết lí thuyết ta có cách giải vật lý cho toán Héron sau
Cách Chuyển động theo đường gấp khúc ACB xem ánh sáng truyền từ A tới
“gương phẳng”d (đường thẳngd đóng vai trị gương phẳng), sau phản xạ gương, tia
(64)Theo định luật phản xạ ánh sáng, ta nhận lời giải toán C trùng vớiC0:
Lúc này∠AC0N D∠NC0B:
Cách giải cho thấy chất vật lý học tốn tốn học Tuy nhiên, khơng phải cách giải vật lý Ta giải toán phương pháp vật lý khác sau
Cách Ta xét vịngC khơng trọng lượng trượt khơng ma sát theo trụcd nằm ngang
Ở đầuC có nối hai sợi Mỗi sợi vịng qua rịng rọc (tương ứng ởAvà ởB)
đầu sợi có treo khối nặng có khối lượng tương ứng làC1 vàC2(hình vẽ)
(Ta thừa nhận điều kiện đơn giản hóa thơng thường: Trục tuyệt đối cứng, sợi tuyệt đối mềm không giãn, ta không kể tới ma sát, trọng lượng sợi phản lực chỗ gập góc chúng, kích thước rịng rọc vịng) Chúng ta cần phải tìm vị trí vịngC thanhd cho hệ thống học trạng thái cân
Thật vậy, hai khối nặng phải treo lơ lửng thấp tốt (nghĩa hệ phải cực tiểu) Do suy tổngAC1 C1 C BC2 C2 phải cực đại Vì chiều dài sợi
dây khơng đổi nên tổngAC C1 C BC C2phải cực tiểu
Ở vị trí cân hệ, lực tác động lên vịngC khơng.C chịu lực căng
!
T1và!T2 (j!T1j D j!T2j- khối nặng kéo lực có độ lớn nhau, lực căng
các khối nặng không giảm ma sát ròng rọc mà truyền toàn vẹn) phản lực!N
(65)từ đó˛ D ˇ:Ta nhận kết cách giải1:
Ở ta cần giải thích thêm ta lại đem chiếu!N C !T1 C !T2 D !0 lên đường thẳng
d (!N ? d) thu hệ thứcj!T1jcos˛ D j!T2jcosˇ:
Lý giải bạn đọc áp dụng tương tự cho toán 9hay toán15dưới
Do lực căng khơng thể kéo vịng theo phương thẳng đứng được, trục d xun qua vịng tuyệt đối cứng (phản lực!N trục có cường độ tùy ý) Thêm vào lực thành phần
nằm ngang hai lực đó, có hướng đối lập, phải triệt tiêu lẫn nhau, phải cường độ Hay ta có hệ thứcj!T1jcos˛ D j!T2jcosˇ:Điều trùng với lập luận
Khái quát hóa toán ta toán sau
Bài toán Trong mặt phẳng, cho trước đường thẳngd và hai điểmA; B ở phía
đối với đường thẳng Trên đường thẳng cho tìm điểmC sao chomAC CM BC đạt
giá trị nhỏ vớimvàM là số dương cho trước?
Bài tốn giải phương pháp tương tự cách giải2bài toán1:
Cũng giống lập luận cách toán (thay C1 m; C2 M), từ điều kiện cân
bằng hệ!N C !T1 C !T2 D !0 ta đem chiếu lên đường thẳng d cho đẳng thức
m cos˛ D M cosˇ, tức vịng C vị trí cho sợi cột vào tạo với d
thỏa hệ thức cos˛
cosˇ D M
m (hình vẽ)
(66)Rõ ràng theo toán thìm D v11; M D v12 nên điều kiện cực tiểu thỏa sin˛ sinˇ D
v1
v2
:Từ
đây dễ dàng rút thời gian cực tiểu cần tìm
Bây tìm số tốn tương tự tốn Héron Ta có tốn
Bài tốn 11 Trên mặt phẳng cho trước3điểmA; B vàC:Hãy tìm điểm thứ tưXsao cho tổng
khoảng cách từ đến3điểm đạt giá trị nhỏ nhất?
Lời giải Chúng ta đưa lời giải học cho toán trường hợp điểm cho
trướcA; BvàC tạo hình tam giác mà tất góc nhỏ hơn120ı Ta xét ba rịng rọc
quay xung quanh trục (cái đinh) đóng tường thẳng đứng điểmA; B C (hình vẽ) Ba sợi chỉXAD1; XBD2; XC D3 luồn qua ba ròng rọc tương ứng ởA; Bvà C:
Ba sợi nối đầu chungX đầu sợi ta treo khối nặng tương
ứngD1; D2vàD3 Các khối nặngD1; D2vàD3nặng Ta phải tìm vị trí cân
Ba khối nặng phải treo lơ lửng thấp tốt nghĩa tổng khoảng cách chúng từ mực ngang cho (mặt đất) phải nhỏ (tức hệ phải cực tiểu) Do tổngAD1 C BD2 C C D3 phải cực đại Vì chiều dài sợi khơng đổi nên
AX CBX CCX phải cực tiểu
Mặt khác trạng thái cân tổng lực tác dụng lên điểmX khơng Có ba lực có độ
lớn tác động lênX lực căng sợi chỉT1; T2vàT3(các khối nặng
kéo sợi lực có độ lớn nhau):j!T1j D j!T2j D j!T3jvà!T1 C !T2 C !T3 D !0
Rõ ràng, đối xứng, lực phải xiên góc lực lực kia, góc hai số ba sợi dây nối chung ởXbằng120ı:(Tam giác ba lực tạo thành đều,
các góc ngồi bằng120ı)
Nhận xét Trong trường hợp điểmA; B vàC tạo thành tam giác mà góc giả sử gócA > 120ıthì điểmXsẽ trùng với đỉnhAnày Bạn suy nghĩ xem sao?
(67)ứngWnvàonsợi dây tạo thành hệ thống lực Khi hệ đứng yên (cân bằng), vị trí điểmP
là điểmF nghĩa rộng (minh họa hình vẽ khinD5)
Ta gọiC số
C D XWi.P Ai C li/ D
X
.Wi P Ai C Wili/;
trong đóli khoảng cách tương ứng từnđiểm đếnnvật nặng
Từ
C D XWi.P Ai C li/ D
X
.Wi P Ai C Wili/;
ta có X
Wi P Ai D C
X
Wili D C
X
.Wi.h hi// D C
X
Wih C
X
Wihi;
trong đóhlà khoảng cách từ mặt phẳng đến mặt đất (phẳng)
NhưngP
Wihcũng số,
X
Wi P Ai D số C
X
Wihi;
tức hệ thống lực cân bằng, tổng củanvật nặngP
Wihi cần đạt giá trị nhỏ
Vì vậyP
Wi P Ai đạt tới giá trị nhỏ nhất, hayl đạt giá trị nhỏ
(68)sao cho tam giácABC không tù Các phần sản phẩm nhà máy cung cấp cho địa điểm A; B; C làm; n; p:Cần phải xây dựng nhà máy đâu chi phí vận chuyển nhất?
(Ở bỏ qua chi phí xây dựng đườngAX; BX vàCX nối địa điểmA; B; C
với nhà máyX và coi q trình chi phí vận chuyển tỉ lệ thuận với tích khối
lượng hàng nhân với độ dài quãng đường).
Ở Sách giáo khoa hình học11nâng cao, có tốn cực trị hình học tương tự với toán Héron Bài toán 14 (Bài toán 2;trang 7; Hình học 11 nâng cao) Hai thơn nằm hai vị trí Avà B
cách sông (xem hai bờ sông hai đường thẳng song song (hình vẽ) Người ta dự định xây cầuMN bắc qua sông (cố nhiên cầu phải vuông góc với bờ sơng) và
làm hai đoạn đường thẳng từAđếnM và từB đếnN:Hãy xác định vị trí cầuMN sao
choAM CBN ngắn nhất?
Bài toán giải việc sử dụng phương pháp hình học túy Tuy nhiên sử dụng phương pháp học để giải
Chúng ta giải toán 14 cách phát biểu toán tổng quát sau
Bài tốn 15 Trên bờ sơng khác có5khu dân cưA1; A2; A3; B1; B2 (A1; A2; A3
trên bờ sông,B1; B2ở bờ bên (hình vẽ)) Cần phải xây dựng cầuCDsao cho tổng
các chiều dài đườngA1C; A2C; A3C: B1D; B2Dtừ khu dân cư đến cầu tối thiểu (các
bờ sơng song song, cầu vng góc với bờ sơng)?
Để giải toán ta xét hệ thống sau Một cứngCDcó thể trượt khơng ma sát
trên đường ray ln ln vng góc với đường ray Ta xét năm ròng rọc quay xung quanh trục (cái đinh) đóng tường thẳng đứng điểm A1; A2; A3 B1; B2 (xem
(69)Do!T1 C !T2 C T!3 C !T4 C !T5 C N!1 C N!2 D !0 vàN!1; N!2 ? d1; d2, ta đem chiếu
lên đường thẳng d song song trùng vớid1 hoặcd2cho ta
j!T1jcos˛1 C jT!2jcos˛2 C j!T3jcos˛3 C j!T4jcosˇ1 C j!T5jcosˇ2 D0;
hay
cos˛1 C cos˛2 C cos˛3 C cosˇ1 C cosˇ2 D0: (1)
Các góc tính theo chiều ngược chiều kim đồng hồ ˛1 ; ˇj 2; i D
1; 2; 3; j D1; 2(xem hình vẽ) Nhận xét Ta có số nhận xét:
1 Đẳng thức (1) không cho ta cách dựng đoạnCD compa thước kẻ,
thường hữu ích cho việc giải toán thực tế tương tự
2 Bài toán 15 toán mà theo số tài liệu người ta chưa tìm lời giải túy tốn học cho Qua ta thấy rõ ưu điểm phương pháp vật lý học ứng dụng số toán toán học
3 Ở tốn 15 bên bờ sơng có khu dân cư ta thu toán 14 Từ đẳng thức (1) cho ta cos˛ C cosˇ D 0; từ đóˇ D C ˛ (hình vẽ), tức
đoạnAC vàBDphải song song Trong trường hợp đoạnCDcó thể dựng
(70)Chúng ta có khám phá thú vị xoay quanh toán Héron Các cách giải vật lý khác nhau, tốn khái qt hóa, tương tự hóa, tốn thực tế đem đến cho nhiều điều bổ ích Qua việc sáng tạo toán học phương pháp vật lý học cịn nhận thấy điều, có toán toán học mà giải phương pháp toán học khó khơng thể giải giải phương pháp vật lý lại tương đối đơn giản Hy vọng viết mang đến cho bạn nhiều cảm nhận mẻ Bài viết cần trao đổi thêm? Mong chia sẻ bạn đọc
Sau số tập luyện tập
Bài toán 16 Cho hai điểmA vàB;đường thẳng d phân cách Avà B và hai vận tốc u vàv:
Tìm thời gian nhỏ cần để chuyền từAtớiB;giả định chuyển từ Atớid thì vận
tốc làuvà từd tớiB thì vận tốc làv?
Bài tốn 17 Có hai kho chứa xăng hình trịn phía đường quốc lộ Người ta
muốn xây dựng trạm cung ứng phân phối xăng bên đường quốc lộ nối với hai bồn xăng trên cho đường ống nối tới hai bồn xăng ngắn nhất? Chúc bạn thành công.
Mọi ý kiến đóng góp gửi tác giả theo địa chỉ: Số nhà 229=85 Thích Quảng Đức,
phường4;quận Phú Nhuận, thành phố Hồ Chí Minh.
Di động: 0908576218
Email:nguyenngocgiang.net@gmail.com
Tài liệu
[1] Nguyễn Bá Đô, Đặng Hùng Thắng, Hoàng Văn Trung (2005), Một số vấn đề toán học chưa giải được, Nhà xuất Giáo dục
(71)GIỚI THIỆU Làm tạo nên hình tam giác với que diêm?
"Hãy nghĩ khác đi, nghĩ khác thói quen, bạn tìm cách giải quyết!"
"Hãy nghĩ khác đi", tiêu chí quan trọng tư sáng tạo, hay gọi tư ngoại biên "Hãy nghĩ khác đi", bạn tìm cách giải cho hàng loạt toán chuyên mục toán học giải trí kỳ với chủ đề tư sáng tạo
Tư sáng tạo
Tư sáng tạo, hay tư ngoại biên (lateral thinking) thuật ngữ đặt Edward de Bono (1933 -) vào năm 1967 để cách suy nghĩ để giải cách giáng tiếp sáng tạo vấn đề khó giải suy luận logic thông thường Theo de Bono, sử dụng tư logic dạng tư "thẳng đứng", tìm cách trực tiếp để giải vấn đề, sử dụng tư ngoại biên việc suy nghĩ "bên lề", để tạo ý tưởng thay đổi cách thức nhìn nhận vấn đề Và vậy, thay "đâm đầu" giải trực diện, tư ngoại biên tìm cách "vượt" qua vấn đề cách tiếp cận hoàn toàn khác lạ Tư ngoại biên, tên nghe xa lạ thật xuất phổ biến câu đố vui dân gian Người viết nhớ câu đố dạng như: "Vì mùa đơng số lồi chim lại bay phương bắc?" câu trả lời thay lý thời tiết, tập quán số loài chim câu đáp án lại "vì chim phải bay hổng lẽ đi!"
(72)Trong phần này, tổng hợp gửi đến bạn đọc 20 câu hỏi tuyển dụng, chủ yếu từ nguồn http://puzzlefry.com Tất cầu hỏi giữ nguyên nội dung toán học, thay đổi mơ tả tốn, chủ yếu để làm ngắn câu hỏi "Hãy nghĩ khác đi," bạn tìm lời giải cho câu hỏi Để giúp bạn "giải toả xúc" nghĩ không cách giải, đăng lời giải gợi ý bên câu hỏi, để tránh làm hứng thú cho bạn đọc muốn tự giải, xoay ngược lại chiều phần giải đáp
Câu hỏi 1 Sử dụng đồng hồ cát, phút, phút, làm để đo xác 9 phút? Yêu cầu thời gian đo không vượt phút.
Lời giải gợi ý:
Bắtđầu bởi2
đồnghồ cátcùng lúc.Sau
4phút, đồhồ
4phút đãc
hạy xong, đồnghồ
7phút
đã đồnghồ yhết, chạ phútđã hồ7 phút,đồng Sau7 4phút đồnghồ Lậtngược 4phút yđược chạ 4phút đãc
hạy 3phút Lậtngược
đồnghồ 4phút Sau8
phút,đồ hồ4 phútđã chạ yx
ong,đồng tâmđồng quan (khôngcần 7phút đồnghồ Lậtngược 1phút yđược chạ phútđã hồ7 hồ4
phút ong yx chạ 7phút đồnghồ 9phút, nữa).Sau
Câu hỏi 2 Có 10 túi tiền, túi có 10 đồng tiền, có túi chứa tồn tiền giả nặng 9gr đồng so với tiền thật đồng nặng 10gr Với cân số, dùng lần cân để tìm túi chứa tiền giả
Lời giải gợi ý:
Chỉcần 1lần cân.X
emthêm ởEpsilon
số7
Câu hỏi 3 Một vị vua bạo ngược tặng 1000 thùng rượu quý, tin mật báo cho hay có thùng rượu bị hạ độc Chất độc cực độc cần uống giọt đủ để chết người, tuy nhiên độc tính tác dụng tháng sau uống Nhà vua bạo ngược thùng nào bị bỏ độc định dùng phạm nhân thử rượu Nhà vua muốn tối đa 10 tù nhân, muốn sau tháng xác định thùng rượu bị bỏ độc Phải làm nào?
Lời giải gợi ý:
Đánhsố cácthùng theohệ
nhịphân Khiđó
tùnhân thứ1
uốngcác bìnhcó
chữ sốđầu
tiênbằng hỉcần nênc 1000 > =1024 10 Vì bằng1 thứ2 hữsố cóc cácbình 2uống nhânthứ 1,tù
10 hạđộc rượubị đượcthùng ácđịnh sẽx thửrượu tùnhân
(73)Đây làmột bàitoán
kinhđiển Chỉcần
3lần cân.Mời
bạnđọc xem lạilời giảic
hitiết ởEpsilon
Câu hỏi 6 Có hộp: hộp có banh màu trắng, hộp có banh màu đen, hộp có banh trắng, banh đen Mỗi hộp có ghi nhãn bên chứa banh màu gì, rủi thay 3 nhãn dán sai Bạn chọn hộp, lấy banh ngẫu nhiên hộp Hỏi phải làm lần để xác định hộp chứa banh gì?
Lời giải gợi ý:
Chỉcần 1lần Chọnhộp
cóg ánnhãn Trắng-Đen
Câu hỏi 7 Một bánh bị cắt bớt phần Làm với nhát cắt chia phần cịn lại thành phần nhau?
Lời giải gợi ý:
Cắtbánh theoc
hiềucao
Câu hỏi 8 Một ốc treo mạn tàu Hiện ốc treo 1m cao mực nước. Nước triều dâng, dâng cao 15 cm Hỏi sau mực nước chạm vào ốc.
Lời giải gợi ý:
Khơngbao giờ,vì
tàunổi trênnước
(74)Nhàđầu tưmua vào 200* 50= 10,000$.Giá cổphiếu tăng50% nêntrở thành10,000 *(1
+ tưbị nhàđầu 9,000.Do 0.4)= (1- 15,000* nêncịn đi40% đógiảm 15,000$.Sau 0.5)=
Câu hỏi 10 Ba sinh viên du lịch thuê chung phòng với giá 30$ (mỗi người trả 10$) Người quản lý sau kiểm lại thấy họ khuyến nên giá có 25$ mà nên gọi nhân viên trả lại 5$ cho sinh viên Nhân viên tự thấy 5$ khó chia lấy đi 2$ đưa lại cho sinh viên 3$ Các sinh viên vui vẻ họ phải trả 27$ (mỗi người 9$) cho tiền phòng Tuy vậy, họ trả 27$ nhân viên giữ lại 1$ tổng cộng là 29$ Hỏi $ lại biến đâu?
Lời giải gợi ý:
Đây làmột câuđố kinhđiển vìcác hđặt vấn đềlàm rốingười đọc.Khơng có$ nàomất đicả
vì là27$ nhânviên cho thêm2$ có25$ tiềnphòng viêntrả 3sinh
Câu hỏi 11 Ở quốc gia nọ, tất gia đình muốn có trai Vì vậy, họ tiếp tục sinh có trai dừng Hỏi kỳ vọng tỉ lệ bé trai bé gái quốc gia là bao nhiêu?
Lời giải gợi ý:
Rấtlý thú,thoạt tiêncó vẻ nhưtỉ lệbé traisẽ caohơn hẳnbé gái, nhưngtính rathì tỉlệ
tiến
Câu hỏi 12 Một tử tù đưa hộp giống hệt với 50 banh trắng 50 quả banh đen Tử tù phép bỏ banh vơ hộp theo ý Sau cai ngục bịt mắt anh ta lại yêu cầu lấy ngẫu nhiên banh từ hộp Nếu banh chọn có màu trắng, tử tù thả tự do, có màu đen, bị xử tử Tìm chiến thuật để xác suất sống cao nhất.
Lời giải gợi ý:
Đặtmột banhtrắng vào mộthộp tồnbộ vào hộpcịn lại.Khi đóx ácsuất sốnglà 1/2+ 1/2 ∗ 49/99 ≈ 75%
Câu hỏi 13 Stacy có 12 vớ trắng 12 vớ đen tủ Hỏi khơng nhìn vào tủ, Stacy phải lấy vớ để tạo thành đơi màu?
Lời giải gợi ý:
(75)Có.Xác suấtbất kỳđể 2số n sốđầu tiênlà nguyên tốcùng nhaulà khoảng 6/π ≈ 61%
, lời sẽcó ậy nhưv lệcược ớitỉ nênv
Vì n sốđầu tiênsẽ có n cặpsố cóthể, với tỉlệ cược1:2, cầnsố cặpkhông nguyên
tố ≥ cùngnhau
n /3 thìđã cólời Tacó sốcặp 2số cùngc hiahết cho n /4 ,cùng chia hếtc
ho sốcặp ậy dov /36 =1 /6 n à3) 2v hocả hếtc chia 6(cùng cho hiahết cùngc /9 n cùngc hiahết cho 2hoặc cùngc hiahết cho 3là n /4+ n /9 − n /36= n /3 Dov ậy, cần
Câu hỏi 15 Có kiến, đỉnh tam giác Bắt đầu kiến bắt đầu di chuyển theo chiều ngẫu nhiên bò dọc theo cạnh tam giác với tốc độ di chuyển bằng Hỏi xác suất chúng không gặp bao nhiêu?
Lời giải gợi ý:
Giữ1 conkiến bấtkỳ xem xác suấtcủa 2con cịnlại cócùng chiều với nóha ykhơng
Xác tư mộtphần nênlà chiều, đicùng chùng rakhi ảy nhaux gặp conkhông cả3 xuấtđể
Câu hỏi 16 Có cọc cắm hồ Một nửa cọc nằm lớp bùn dưới đáy hồ, phần ba cọc nằm nước phần nhô lên khỏi mặt hồ feet Hỏi chiều cao cọc.
Lời giải gợi ý:
Phầnnằm dướihồ là1/2 +1/3 =5/6 cọc,nên phầnnhơ lêndo là1/6 củacọc, từđó
suy feet là42 củacọc hiềucao rac
Câu hỏi 17 Làm để lấy từ 19 để lại 20?
Lời giải gợi ý:
SốLa Mã.19 =XIX, lấy đi1 làXX =20
Câu hỏi 18 Nếu bạn có bánh pizza có độ dày bằnga và bán kính bằngz thì thể tích
(76)Câu hỏi 19 Vào lúc 15 góc kim kim phút bao nhiêu?
Lời giải gợi ý:
Thoạtnhìn tưởng0,
nhưngtính ralà
7.5độ
Câu hỏi 20 Làm trịn gần đến cm, có đất lỗ hình hộp chữ nhật kích thước3×4×5mét?
Lời giải gợi ý:
Khơngcó đất,vì
đây làmột lỗ!
Phần kết
Nếu bạn cịn tìm cách giải cho câu đố phần giới thiệu, bạn lần thử nghĩ khác đi, thay tìm xếp que diêm mặt phẳng, bạn thử nghĩ thêm chiều không gian thứ Vâng, vậy, tứ diện đều, với mặt tam giác cạnh tạo que diêm Và liệu có phải cách giải nhất? Khơng phải, chúng tơi cịn có cách giải khác nữa, khác! Bạn có tìm khơng? Nếu khơng, dịng bên chúng tơi cho bạn thêm lời giải Hãy đọc chúng qua gương
Dùng3 que đầy xếp thànhsố
bốn,v à3 que saux ếpthành hìnhtam
giác
Có2 cách ởđâ y,viết số4
dạngLa MãIV
(77)LỜI DẪN
Đây phần kết thúc viết đăng số9của tạp chí 2.4 Lơgarit hóa để biến tích thành tổng
Sử dụng công thức loga.xy/ D logax Clogay, ta biến tích thành tổng Chú ý để
logax cú ngha thỡ0 < a Ô vx > Khi lơgarit hóa ta thường chọn số e để việc tính
tốn đơn giản
Bài tốn 12 Tính
lim
n!C1
n q
.n2C1/ n2C4/ n2C.2n/2
n4 :
Lời giải ĐặtanD
n q
.n2C1/ n2C4/ n2C.2n/2
n4 ; 8nD1; 2; : : :Khi
anD
n2C1
n2C4
n2C.2n/2n1
n
D
"
n4n 1C
1 n 2! 1C 2 n 2! 1C 2n n 2!# n n D " 1C n 2! 1C n 2! 1C 2n n 2!# n
lnanDln
" 1C 1 n 2! 1C 2 n 2! 1C 2n n 2!# n D n "
ln 1C
1
n
2!
Cln 1C
2
n
2!
C Cln 1C
2n
n
2!#
(78)Do
lim
n!C1.lnan/D
2
Z
0
ln.1Cx2/dx:
Đặt
u Dln.1Cx2/
dv Ddx )
8 < :
duD 2x
1Cx2
v Dx:
Khi
2
Z
0
ln.1Cx2/dx D xln.1Cx2/ˇˇ
2
2
Z
0
x2dx
1Cx2 D2ln5
2
Z
0
1
1Cx2
dx
D2ln5
2
Z
0
dxC2
2
Z
0 dx
1Cx2 D2ln5 4C2arctan2:
Do hàm sốex liên tục nên
lim
n!C1anDn!C1lim e
lnan Den!C1lim lnan/ De2ln5 4C2arctan2 '4; 192: Từ thu kết tốn
Bài tốn 13 Tính lim
n!C1Pn, với
PnD
2C
5n 2C
18 5n
2C 15n 12
5n
3n
;8nD1; 2; : : :
Lời giải Ta cóPn > 0và
lnPnD
n
ln
2C
5n
Cln
2C 18
5n
C Cln
2C15n 12
5n D n n X
iD1
ln
2C 15i 12
5n
:
Xét hàm sốf x/Dln.2Cx/trên đoạnŒ0; 3 Rõ ràngf x/liên tục trênŒ0; 3nên khả tích
trên đoạn
Xét phép phân hoạch đoạnŒ0; 3bởi điểm chia
x0 D0; x1 D 31
n ; x2 D
32
n ; : : : ; xn D
3.n 1/
n ; xnD
3n
(79)lim
n!C1.lnPn/D
Z
0
f x/dxD
Z
0
ln.2Cx/dx:
Đặt
uDln.2Cx/
dv Ddx )
8 < :
duD dx
2Cx
v Dx:
Khi
3
Z
0
ln.2Cx/dx D Œxln.2Cx/j30
Z
0 xdx
2Cx
D3ln5
Z
0
1
2Cx
dx D3ln5 Œx 2ln.2Cx/j30
D3ln5 Œ.3 2ln5/C2ln2D5ln5 2ln2 3:
Suy lim
n!C1PnDn!C1lim e
lnPn Den!C1lim Pn De5ln5 2ln2
Bài tốn 14 Tính giới hạn đây
a/ lim
n!C1
n
Y
kD1
.k n/.nCk/1 lnn b/ lim
n!C1
n
X
kD1
k n:
Hướng dẫn.a/ĐặtunD
n
Q
kD1
.k n/.nCk/1lnn, đóun> 0 Ta có
lnunD
n
X
kD1
ln.k n/.nCk/1 lnn D
n
X
kD1
ln.k n/
.kCn/lnn D
n
X
kD1
ln
k
n n
2
.kCn/lnn
D
n
X
kD1
lnk
n
.kCn/lnnC
n
X
kD1
2lnn
(80)lim
n!C1ˇn Dn!C1lim
n
X
kD1
kCn Dn!C1lim
0 B @ n n X
kD1 k
n C1
1 C AD2
1
Z
0 dx
xC1 D2ln2:
2.5 Sử dụng tổng tích phân kết hợp với nguyên lí kẹp
Nhiều giới hạn cần tính khơng phải tổng tích phân hàm số Lúc ta cần sử dụng bất đẳng thức để đánh giá tổng cho theo tổng tích phân hàm số đó, sau sử dụng ngun lí kẹp để suy kết
Định lý (Nguyên lí kẹp) Nếu ba dãy số.xn/,.yn/,.zn/thỏa mãn điều kiện
yn 6xn6zn;8n>n0;
và lim
n!C1ynDn!C1lim znDLthìn!C1lim xn DL.
Bài toán 15 Giả sửk là số nguyên dương, ˛ là số thực Hãy tìm giới hạn dãy số an/, với
anD
1k ˛C
2k ˛C C
nk ˛
nkC1 ; 8nD1; 2; : : :
Hướng dẫn.Với số thựcx, ta cóx < Œx6x, nên ˛ 1k C2kC Cnk
nkC1
1
nk < an6
˛ 1kC2k C Cnk
nkC1 ; 8nD1; 2; : : : (1)
Tương tự toán 2.5 phần trước, ta có lim
n!C1
1kC2k C Cnk
nkC1 D
1
Z
0
xkdxD x
kC1
kC1
ˇ ˇ ˇ ˇ D
kC1:
Vậy từ (1), sử dụng nguyên lí kẹp, suy lim
n!C1anD
˛
kC1
Bài tốn 16 Tính lim
n!C1
n
X
kD1
nCk2
n3Ck3.
Lời giải Đặt
unD
n
X
kD1
nCk2
n3Ck3 D
1 n
n
X
kD1
nCk2
n2C k
3 n D n n X
(81)Ta có
jun vnj D
n n
X
kD1 n 1C k n
n; 8nD1; 2; : : : ;
từ sử dụng nguyên lí kẹp ta lim
n!C1jun vnj D0 Như
ˇ ˇ ˇ ˇ
un
3ln2
ˇ ˇ ˇ ˇ6j
un vnj C
ˇ ˇ ˇ ˇ
vn
3ln2
ˇ ˇ ˇ ˇ
; 8nD1; 2; : : :
Từ sử dụng nguyên lí kẹp ta lim
n!C1un D
1
3ln2
Bài tốn 17 Tính
lim
n!C1
sin n
n2C12 Csin
n
n2C22 C Csin n
n2Cn2
:
Lời giải Ta chứng minh
x x
3
6 6sinx6x; 8x >0:
Do
n
n2Ck2
n3
6.n2Ck2/3 6sin
n
n2Ck2
n
n2Ck2;
suy
n
X
kD1 n
n2Ck2
n
X
kD1
n3
6.n2Ck2/3
n
X
kD1
sin n
n2Ck2
n
X
kD1 n
n2Ck2: (2)
Ta có
lim
n!C1
n
X
kD1 n
n2Ck2 Dn!C1lim
0 B B B @ n n X
kD1 1C k n C C C A D Z dx
1Cx2 D
(82)
kD1
Từ sử dụng nguyên lí kẹp ta lim
n!C1
n
X
kD1
n3
6.n2Ck2/3 D0:
Như từ (2) sử dụng nguyên lí kẹp ta lim
n!C1
n
X
kD1
sin n
n2Ck2 D
4
Bài tốn 18 Tính lim
n!C1
2n
X
kDnC1
sin
k.
Lời giải VớinD1; 2; : : :, đặtun D
2n
X
kDnC1
sin
k; vnD
2n
X
kDnC1
k
Ta tính lim
n!C1vn Ta có
vnD
2n
X
kDnC1
k D
n
X
iD1
iCn thaykbởiiCn/
D
n n
X
iD1 i
n C1
D
n n
X
iD1 f i n ;
với f hàm số f x/ D
xC1 Chia đoạn Œ0; 1 điểm chia xi D i
n; với i D
0; 1; : : : ; n;tức chia đoạnŒ0; 1thànhnđoạn
x0D
n D0 < x1 D
1
n < x2 D
2
n < < xn 1D
n
n < xn D
n
n D1:
Chọn˛i D i
n 2Œxi 1; xi;8i D1; 2; : : : ; n Khi tổng tích phân hàm sốf x/trên
Œ0; 1ứng với phép phân hoạch nói
n n
X
iD1 f
i n
Như
lim
n!C1vn Dn!C1lim
"
1 n
n
X
iD1 f i n # D Z
f x/dx D
1
Z
0 dx
xC1
(83)6
6i
D1 iCn
6
6 i
D1 n
D n
3
6n3 D
3
6n2; nD1; 2; : : :
Từ sử dụng nguyên lí kẹp ta lim
n!C1jun vnj D
Ta có jun ln2j jun vnj C jvn ln2j; 8n D 1; 2; : : : Từ sử dụng nguyên
lí kẹp ta lim
n!C1unDln2 Vậyln2là kết cần tìm
Bài tốn 19 Tính lim
n!C1
enC11 Ce
1
nC2 C Ce nCn n
.
Lời giải Trước hết, ta chứng minh
06ex x x
2
2 e
x
; 8x > 0: (3)
Xét hàm số liên tụch.x/Dex 1Cx/;8x 2R Ta có
h0.x/Dex 1; 8x 2R:
h0.x/D0,ex D1,x D0:
Lập bảng biến thiên
x C1
h0.x/ C
h.x/ &0%
ta suy rah.x/>0; 8x2 Rhayex >1Cx;8x 2R Tiếp theo, ta chứng minh
ex x6 x
2
2 e
x;
(84)Xét hàm số liên tụcf x/Dex x x
2 e
x
; 8x > Vớix > 0, ta có
f0.x/Dex
xex C x
2
2 e
x
;
f00.x/Dex
exCxex Cxex C x
2 e x D
2xex C x
2
2 e
x
< 0:
Suy hàm sốf0.x/nghịch biến trên.0;C1/, nên
f0.x/ < lim
x!0Cf
0.x/ D0; 8x2 .0;C1/ :
Như hàm sốf x/nghịch biến trên.0;C1/ Ta có lim
x!0Cf x/D0
Từ bảng biến thiên
x C1
f0.x/
f x/ &
ta thấy rằngf x/ < 0; 8x 2.0;C1/ Như (4) đúng, ta có điều phải chứng minh
Vớin2N, đặtunD
n
X
kD1 enC1k
!
n Do (3) nên
ˇ ˇ ˇ ˇ ˇ un n X
kD1
nCk
ˇ ˇ ˇ ˇ ˇ D n X
kD1
enC1k 1
nCk
6
n
X
kD1
enC1k
2.nCk/2
nenC11
2.nC1/2; 8n2N
:
(5)
Ta có lim
n!C1
nenC11
2.nC1/2 D0và
lim
n!C1
n
X
kD1
nCk Dn!C1lim
1 n
n
X
kD1
1C k
n D Z dx
1Cx D ln.1Cx/j
1
0Dln2:
Như từ (5), sử dụng nguyên lí kẹp suy lim
n!C1unDln2
Bài tốn 20 Cho trước˛ > 0:Hãy tính
lim
n!C1
n
X
kD1
(85)1
nC1˛ >
1
nC2˛ > >
1
nCn˛;
) n
nCk˛ <
1
nC1˛ C
1
nC2˛ C C
1
nCn˛ <
n
nC1˛;
) n
nCk˛ <
n
X
kD1
nCk˛ <
n
nC1; 8nD1; 2; : : : (6)
Do
lim
n!C1
n
nCk˛ D1Dn!C1lim
n
nC1;
nên từ (6), sử dụng nguyên lí kẹp, ta lim
n!C1
n
X
kD1
nCk˛ D1:
Nếu˛ > Khi đónCk˛ >2pnk˛ D2pnk˛2;cho nên
0 < n
X
kD1
nCk˛
1
2pn
1
1˛2 C
1
2˛2 C C
1 n˛2
: (7)
Mà dãy số pn
ntăng nghiêm ngặt đến dương vô cực nên sử dụng định lí Stolz, ta có
lim
n!C1
1
1˛2 C
1
2˛2 C C
1 n˛2
p
n Dn!C1lim
1
.nC1/˛2
p
nC1 pn
D lim
n!C1
p
nC1Cpn
.nC1/˛2 Dn!C1lim
r
1C
n C1
n˛21
1C
n
˛2 D
0:
Như vậy, từ (7), sử dụng nguyên lí kẹp, ta lim
n!C1
n
X
kD1
nCk˛ D0
(86)Nếu˛ D1thì lim n!C1
n
X
kD1
nCk˛ Dln2
Nếu˛ > 1thì lim
n!C1
n
X
kD1
nCk˛ D0
Bài toán giải xong
Nhận xét Định lí Stolz phát biểu sau
Định lý (Định lý Stolz) Cho.xn/và.yn/là hai dãy thỏa mãn
a) Dãy.yn/tăng thực tớiC1.
b) lim
n!C1
xnC1 xn
ynC1 yn
Dc 2R.
Khi đó lim
n!C1
xn
yn Dc.
2.6 Các toán ôn luyện tổng hợp
Bài toán 21 (Học viện kỹ thuật mật mã - 1999) Tìm lim
n!C1Snbiết
SnD n
n2C12 C
n
n2C22 C C n
n2Cn2:
Hướng dẫn lim
n!C1SnD
Z
0
f x/dxD
1
Z
0 dx
1Cx2 D
4:
Bài toán 22 ChoSnD p1
n3
1Cp2C Cpn Tìm lim
n!C1Sn.
Hướng dẫn lim
n!C1SnD
1
Z
0
f x/dxD
1
Z
0
p
x D
3:
Bài tốn 23 Tính giới hạn sau
a/ lim
n!C1
1
p
4n2 12 C
1
p
4n2 22 C C
1
p
4n2 n2
(87)
n n n
f / lim
n!C1
"
1C
n2
1
1C
n2
2
1C n 1/
2
n2
n
2n
#n21
: g/ lim n!C1 n2 sin n
1Ccos2
n
C
2sin2
n
1Ccos22
n
C C
nsinn
n
1Ccos2n
n
3 5.
Đáp số.a/
6; b/ 0; c/
1
12ln3; d / 2ln2 1; e/
1
2; f /
2
p
e; g/
2
4
Bài tốn 24 Tính lim
n!C1n
1k
C2kC Cnk
nkC1
1
kC1
; k 2N.
Lời giải Dễ thấy hàm sốf x/Dxk có đạo hàm khả tích bị chặn trênŒ0; 1 Khi
SnD
n n
X
iD1 f
0Ci1
n D n n X
iD1 f i n Z
f x/dxD
1
Z
0
xkdx D x
kC1
kC1
ˇ ˇ ˇ ˇ D
kC1:
Áp dụng tốn15, ta có
lim
n!C1n
1kC2k C Cnk
nkC1
1
kC1
D lim
n!C1n
0 @Sn Z f x/dx A D
2Œf 1/ f 0/D
1
2:
(88)nk C C C kC1 D .kC1/ nk
Xét hai dãy số.xn/; yn/như sau
xn D.kC1/1k C2kC Cnk nkC1; ynD.kC1/ nk:
Khi dãy.yn/tăng thực lim
n!C1yn D C1 Ta có
lim
n!C1
1C
n
k
D lim
n!C1
Ck0CC
1 k:
1
n CC
2 k:
1
n2 C CC
k k: nk D lim n!C1
1C k
n : lim n!C1 "
1C
n
kC1
1
#
D lim
n!C1
Ck1C1
n CC
2 kC1
1
n2 C CC
kC1 kC1
1
nkC1
D lim
n!C1
kC1
n C
k.k C1/
2n2 : Vì lim n!C1
xnC1 xn
ynC1 yn
D lim
n!C1
.kC1/ nC1/k nC1/kC1CnkC1
.kC1/
.nC1/k nk
D lim
n!C1
.kC1/ nk
1C
n
k
nkC1
"
1C
n
kC1
1
#
.kC1/ nk
"
1C
n k # D lim n!C1
.kC1/ nk
1C k
n
nkC1
k
C1
n C
k.kC1/
2n2
.kC1/ nk k
n
D lim
n!C1
.kC1/ n
1C k
n
n2
kC1
n C
k.kC1/
2n2
.kC1/ k
D lim
n!C1
.kC1/k k C1/k
2
.k C1/ k Dn!C1lim
1
2 D
1
2:
(89)Suy
.kC1/ nC1/k h.nC1/kC1 nkC1i D kC1/k
2 n
k
C (8)
Mặt khác
.kC1/h.nC1/k nki D.kC1/
k:nk 1C k.k 1/
2 n
k
C C1
(9) Từ (8) (9) suy
lim
n!C1
xnC1 xn
ynC1 yn
D lim
n!C1
.kC1/k
2 n
k 1C
.kC1/
k:nk 1Ck.k 1/
2 n
k 2C C1
D
1
2:
Bài toán 25 Tính giới hạn sau
a/ lim
n!C1
1
nC1C
1
nC2 C C
1 2n . b/ lim n!C1
2nC1 C
2
2nC3C C
2
4n
.
c/ lim
n!C1n
ln2
nC1
1
nC2
1 2n
.
d / lim
n!C1n
2
ln2
2nC1
2
2nC3
2
4n
.
Lời giải a/Ta có n
X
kD1
nCk D
1 n
n
X
kD1
1C k
n
D
n n
X
kD1 f
k n
, vớif hàm số
f x/D
(90)x0D
n D0 < x1 D
1
n < x2 D
2
n < < xn 1D
n
n < xn D
n
n D1:
Chọn˛i D i
n 2Œxi 1; xi;8i D1; 2; : : : ; n Khi tổng tích phân hàm sốf x/trênŒ0; 1
ứng với phép phân hoạch nói
n n
X
kD1 f
k
n
Như
lim
n!C1
n
X
kD1
nCk D
1
Z
0 dx
1Cx D ln.1Cx//j
1
0 Dln2:
b/VớinD1; 2; : : :, ta có
n
X
iD1
2
2nC2i D
1 n
n
X
iD1
1C 2i 12n
D
n n
X
iD1 f
2i 1
2n
:
Do 2i
2n D i 1 n C i n
là trung điểm đoạn
i 1
n ;
i n
vớii D1; 2; : : : ; ncho nên
1 n
n
X
iD1 f
2i
2n
;là tổng tích phân hàm sốf x/trên đoạnŒ0; 1ứng với phép phân hoạch
đều đoạnŒ0; 1bởi điểm chiaxi D i
n i D0; 1; : : : ; n/và chọn˛i D
2i
2n trung điểm
của đoạnŒxi 1; xi ;8i D1; 2; : : : ; n:Như
lim
n!C1
n
X
iD1
2
2nC2i Dn!C1lim
1 n
n
X
iD1 f 2i 1 2n ! D Z
f x/dxDln2:
c/Ta có
lim
n!C1n ln2
n
X
iD1
nCi
!
D lim
n!C1n
0 B @ Z
f x/dx
n n
X
iD1
1C i
n
1 C A
D f 1/ f 0/
2 D 1 D 4:
d /Ta có
lim
n!C1n
2 ln
n
X
iD1
2
2nC2i
!
D lim
n!C1n
2 @ Z
f x/dx
n n
X
iD1 f 2i 2n A Df
0.1/ f0.0/
24 D
1 24
1
4C1
D
24:4 D
1
32:
(91)ln2nC1
n < xn<ln
2n
n 1;8nD2; 3; : : : (10)
Vì hàm sốf x/Dlnxliên tục khoảng.0;C1/nên
lim
n!C1
ln2nC1
n Dln lim n!C1
2nC1
n
Dln2; lim
n!C1
ln 2n
n
Dln2:
Vậy từ (10) sử dụng nguyên lý kẹp suy lim
n!C1xn Dln2
Bài toán 26 Cho hai số nguyên dương˛; ˇ Tính lim n!C1
n
Y
kD1
1C k
˛
nˇ
.
Lời giải Ta có lim
n!C1
k˛
nˇ D0, mà limx!0
ln.1Cx/
x D1nên limn!C1 ln
1C k
˛
nˇ
k˛
nˇ
D1 Do
lim n!C1 B B @
kD1;2;:::;n
ln
1C k
˛ nˇ k˛ nˇ C C A
D1; lim
n!C1 B B @ max
kD1;2;:::;n
ln
1C k
˛ nˇ k˛ nˇ C C A
D1: (11)
VớinD1; 2; : : :, đặtxn D
n
Y
kD1
1C k
˛
nˇ
Khi
lnxnD
n
X
kD1
ln
1C k
˛ nˇ D n X
kD1
ln
1C k
˛ nˇ k˛ nˇ k ˛
nˇ:
Suy
min
kD1;2;:::;n
ln
1C k
˛ nˇ k˛ nˇ n X
kD1
k˛
nˇ 6lnxn 6kDmax1;2;:::;n
ln
1C k
˛ nˇ k˛ nˇ n X
kD1
k˛
(92)lim
n!C1
kD1n
ˇ Dn!C1lim n
kD1 n
D
0
x dxD
˛C1ˇˇ0 D ˛C1
> 0: (13)
Từ (11), (12), (13) sử dụng nguyên lí kẹp ta lim
n!C1.lnxn/D
1
˛C1 )n!C1lim xnDn!C1lim e
lnxn Den!C1lim lnxn/De˛C11:
Nếu˛C1 < ˇ Khi
lim
n!C1
n
X
kD1
k˛
nˇ Dn!C1lim
1
nˇ ˛C1/
1 n
n
X
kD1
k n ˛ D0 Z
x˛dx D0: (14)
Từ (11), (12), (14), sử dụng nguyên lí kẹp ta lim
n!C1.lnxn/D0)n!C1lim xnDe
0
D1:
Nếu˛C1 > ˇ Khi
lim
n!C1
n
X
kD1
k˛
nˇ Dn!C1lim n
˛C1 ˇ n
n
X
kD1
k
n
˛!
D C1: (15)
Từ (11), (12), (15), ta lim
n!C1.lnxn/D C1 )n!C1lim xn D C1:
Kết luận lim
n!C1
n
Y
kD1
1C k
˛ nˇ D < :
e˛C11 ˛C1Dˇ;
1 ˛C1 < ˇ;
C1 ˛C1 > ˇ:
Nhận xét Một trường hợp đặc biệt toán đề đề nghị thi Olympic 30/04/2006
Cho dãy số.xn/xác định bởi
xn D
1C
n2 1C
2
n2
: : :1C n
n2
;8nD1; 2; : : :
Hãy tìm lim
n!C1.lnxn/.
Đối với trường hợp riêng giải nhẹ nhàng sau Trước hết ta chứng minh bất đẳng thức
x x
2
(93)Bởi (16) chứng minh Bây ta tìm lim
n!C1.lnxn/ Với mọin2 N lnxnDln
1C
n2
Cln
1C
n2
C Cln1C n
n2
:
Theo (16) ta có
i
n2
i2
2n4 6ln
1C i
n2
6 i
n2;8nD1; 2; : : : ; 8i D1; 2; : : : ; n:
Suy với mọinD1; 2; : : : ;ta có
n2 1C2C Cn/
2n4
2
C22C Cn2
6lnxn6
n2.1C2C Cn/: (17)
Mà1C2C CnD n.nC1/
2 ;
2
C22C Cn2 D n.nC1/.2nC1/
6 nên từ (17) ta có
n.nC1/
2n2
n.nC1/.2nC1/
12n4 6lnxn6
n.nC1/
2n2 ;8nD1; 2; : : : (18)
Vì lim n!C1 n.n C1/ 2n2
n.nC1/.2nC1/
12n4 D lim n!C1 n.n C1/ 2n2 D 2;
nên từ (18) sử dụng nguyên lý kẹp ta suy lim
n!C1.lnxn/D
1
2
Bài toán 27 Cho số thực˛và số nguyênp > 1 Tính giới hạn sau
lim
n!C1
n
Y
kD1
np C.˛ 1/kp
np kp :
Lời giải VớinD1; 2; : : :, đặtxnD
n
Y
kD1
npC.˛ 1/kp
np kp Khi
lnxnD
n
X
kD1
ln
np C.˛ 1/kp
np kp
D
n
X
kD1
ln
1C ˛k
p
np kp
(94)
min
kD1;2;:::;nB
@ ˛k
p
np kp
C AkD1n
p kp 6lnxn
6 max
kD1;2;:::;n
0 B B B @ ln
1C ˛k
p
np kp
˛kp
np kp
1 C C C A n X
kD1
˛kp
np kp 1;8nD1; 2; : : :
(19)
Ta có lim
n!C1
˛kp
np kp D0, mà limx!0
ln.1Cx/
x D1nên
lim
n!C1 ln
1C ˛k
p
np kp
˛kp
np kp
D1:
Tiếp theo ta tính giới hạn lim
n!C1
˛kp
np kp Dok2 f1; 2; : : : ; ngnên
np np 6np kp 6np:
Vì
n
X
kD1
kp
np
n
X
kD1
kp
np kp
n
X
kD1
kp
np np D
n
X
kD1
kp
np 1.n 1/
)1
n n
X
kD1
k n p n X
kD1
kp
np kp
1 n
n
X
kD1
k n
p 1!
n n Mà lim n!C1 n n X
kD1
k n p D Z
xp 1dx D x
p p ˇ ˇ ˇ ˇ D p; lim n!C1 " n n X
kD1
k
n
p 1!
n n # D p; (20)
nên từ (20) chon! C1, ta
lim
n!C1
n
X
kD1
kp
np kp D
1
p )n!C1lim
n
X
kD1
˛kp
np kp D
˛
p:
Như vậy, từ (19) sử dụng nguyên lí kẹp ta lim
n!C1.lnxn/D
˛
p )n!C1lim xnDn!C1lim e
lnxn
Den!C1lim lnxn/ De˛p:
(95)MàP x/Da0xmC1Ca1xmCa2xm 1C Camx; a0 Ô0nờn P
1
nCj
Da0
1
nCj
mC1
Ca1
1
nCj
m
C Cam
1
nCj
:
Do
vnDP
n CP
nC1
CP
1
nC2
C CP
1
nCk n
D " a0 1 n
mC1
Ca1
1
n
m
C Cam
1 n # C " a0
nC1
mC1
Ca1
1
nC1
m
C Cam
1
nC1
#
C: : :
C
"
a0
1
nCk n
mC1
Ca1
1
nCk n
m
C Cam
1
nCk n
#
Da0
k n
X
jD0
1
nCj
mC1
Ca1
k n
X
jD0
1
nCj
m
C Cam
k n
X
jD0
1
nCj
:
(21)
Vớii >2ta có
06
k n
X
jD0
1
nCj
i
D
ni C
1
.nC1/i C C
.nCk n/i
k nC1
ni :
Mà lim
n!C1
k nC1
ni D0nên theo nguyên lý kẹp suy
lim
n!C1
k n
X
jD0
1
nCj
i
D0: (22)
Vớii D1ta có k n
X
jD0
1
nCj
i
D
k n
X
jD0
1
nCj
D
nC
k n
X
jD1
1
nCj
(96)Dễ thấy
k n
X
jD1 nf j n D n k n X
jD1 f
j
n
là tổng tích phân hàm sốf x/ đoạnŒ0; k
với điểm chiaxi D i
n.i D0; 1; 2; : : : ; k n/và cách chọn điểm˛g D
g
n g D1; 2; : : : ; k n/
Thành thử lim n!C1 n k n X
jD1 f j n D k Z
f x/dx D
k
Z
0
1Cxdx Dln.1Cx/j
k
0 Dln.1Ck/:
Do
lim
n!C1
k n
X
jD0
1
nCj
D lim n!C1 nC k n X
jD1
1
nCj
D lim n!C1
n Cn!C1lim
k n
X
jD1
1
nCj
Dln.1Ck/:
(23)
Từ (21), (22), (23) suy lim
n!C1vnDamln.1Ck/
Bài toán 29 Cho˛; ˇ là số dương Hãy tính giới hạn sau
lim
n!C1
n
X
kD1
kC˛
n2Ck nCˇ:
Lời giải ĐặtxnD
n
X
kD1
kC˛
n2Ck nCˇ D
1 n
n
X
kD1
kC˛
nCkCˇ
n
Khi
xn D
n n
X
kD1 k
n C
˛ n
1C k
n C ˇ n2 D n n X
kD1
k n C ˇ n2 C˛ n ˇ n2 1C k n C ˇ n2 D n n X
kD1 k
n C
ˇ
n2
1C k
n C
ˇ
n2
C ˛ n ˇ
n n
X
kD1
1
n2Ck nCˇ:
(24)
Vớin >
ˇ ˛
C2,k f1; 2; : : : ; ng, ta có
1
2n2Cˇ
1
n2Ck nCˇ
1
(97)Vớin > ŒˇC1 > ˇ,k f0; 1; : : : ; n 1g, ta có k n k n C ˇ
n2
kC1
n :
Do
n n
X
kD1 k
n C
ˇ
n2
1C k
n C
ˇ
n2
là tổng tích phân hàm sốf x/D x
1Cx ứng với phép phân hoạch
đều đoạnŒ0; 1bởi điểm chia0;1n;n2; : : : ;n 1n ; 1và điểm trung gian˛k
k
n;
kC1
n
là˛k D k
n C
ˇ
n2 k f0; 1; : : : ; n 1g/:
Do lim n!C1 n n X
kD1 k
n C
ˇ
n2
1Ck
n C ˇ n2 D Z xdx
1Cx D
1
Z
0
1
1Cx
dx
D x ln.1Cx//j10D1 ln2:
(26)
Từ (24), (25), (26) suy lim
n!C1
n
X
kD1
kC˛
n2Ck nCˇ D1 ln2
Bài tốn 30 Choa; blà số khơng âm Hãy tính giới hạn sau
lim
n!C1
n
X
kD1
1
nCkCbCpn2Ck nCa:
Lời giải Doa>0; b >0nên
nCkCbCpn2Ck nCa>nCkCpn2Ck n
)xnD
n
X
kD1
1
nCkCbCpn2Ck nCa
1 n
n
X
kD1
1
1Ck
n C
r
1C k
(98)n n
Đặtp DŒb Khi vớin> a
pC1 b; k D1; 2; : : : ; n p 1, ta có
kCp
n k n C b n C a
n2
kCpC1
n :
Ta biến đổi
xnD
n p
X
kD1
1
nCkCbCpn2Ck nCaC
n
X
kDn p
1
nCkCbCpn2Ck nCa
D
n p
X
kD1
1
nCkCbCpn2Ck nCa C
pC1
X
kD1
1
nCkCpn2Ck n
!
pC1
X
kD1
1
nCkCpn2Ck nC
n
X
kDn p
1
nCkCbCpn2Ck nCa
DS1 S2CS3:
Chú ý
S2 D
pC1
X
kD1
1
nCkCpn2Ck n
pC1
n ;
S3 D
n
X
kDn p
1
nCkCbCpn2Ck nCa
pC1
n :
Suy lim
n!C1S2 Dn!C1lim S3 D0 Do
nCkCbCpn2Ck nCa6nCkCbCa
n C
p
n2Ck nCbnCa
nên
S1 D
n p
X
kD1
1
nCkCbCpn2Ck nCa C
pC1
X
kD1
1
nCkCpn2Ck n
>
n n p
X
kD1
1
1C k
n C
b
n C
a
n2 C
r
1C k
n C b n C a n2 C n pC1
X
kD1
1
1C k
n C
r
1C k
n D n B B @
n p
X
kD1
1
1C k
n C
b
n C
a
n2 C
r
1C k
n C b n C a n2 C
pC1
X
kD1
1
1C k
n C
r
1C k
n
(99)lim
n!C1S1D
1
Z
0
dx
1CxCp1Cx D2ln
1Cp2
2 )n!C1lim xnD2ln
1Cp2
2 :
Vậy2ln1C
p
2
2 kết cần tìm
Nhận xét Để có bất đẳng thức
kCp
n k n C b n C a
n2
kCpC1
n ;
ta cần có
p
n
b
n C
a
n2
pC1
n ,pn6aCbn6.pC1/n
,
.b p/n> a
.b p 1/n6 a ,
8 ˆ < ˆ :
n> a
b p
n> a
b p
,n> a
pC1 b:
Bài toán 31 Choa; b; c là số dương, Hãy tính giới hạn sau
lim
n!C1
n
X
kD1
kCa
p
n2Ck nCbpn2Ck nCc:
Lời giải Theo tốn 29, ta có
lim
n!C1
n
X
kD1
kC˛
n2Ck nCˇ D1 ln2 ˛ > 0; ˇ > 0/ : (27)
Giả sử0 < b 6c(trường hợp0 < c < blập luận tương tự),
kCa
n2Ck nCc
kCa
p
n2Ck nCbpn2Ck nCc
kCa
n2Ck nCb:
Suy
n
X
kD1
kCa
n2Ck nCc
n
X
kD1
kCa
p
n2Ck nCbpn2Ck nCc
n
X
kD1
kCa
(100)n!C1
kD1 n
2Ck nCb n2Ck nCc
Từ thu kết toán
Bài toán 32 Choa; blà số dương,˛là số nguyên dương Hãy tính giới hạn sau
lim
n!C1
1
n2˛C1
n
X
kD1
k2CakCb˛
!
:
Lời giải Đặtun D
n2˛C1
n
X
kD1
k2CakCb˛; D
n2˛C1
n
X
kD1
k2˛:Khi
vnD
n n
X
kD1
k n
2˛
) lim
n!C1vn D
1
Z
0
x2˛dxD x
2˛C1
2˛C1
ˇ ˇ ˇ ˇ D
2˛C1:
Do hàm sốt2˛ đồng biến trên.0;C1/nênvn un; 8n D1; 2; : : : Choc Ncố định, với
mọik2N, ta có
k2CakCb 6.kCc/2 ,2kc Cc2>akCb(
2c >a
c2 >b:
Như vậy, đặtc Dhmaxna
2;
p
boiC1thì
k2CakCb6.k Cc/2; 8k D1; 2; : : :
Đặt
wnD
n2˛C1
n
X
kD1
.k Cc/2˛ D
n n
X
kD1
kCc
n
2˛
D
n cCn
X
iDcC1
i n
2˛
:
Khi hàm sốt2˛ đồng biến trên.0;C1/nênun6wn; 8n2N:Ta có
wn vnD
n n
X
kD1
kCc
n 2˛ k n 2˛! :
Do hàm sốg.x/Dx2˛liên tục đoạnŒ0; 1nên hàm sốg.x/Dx2˛ liên tục trênŒ0; 1.
Cho" > 0, tồn tạiı > 0sao cho
8x0; x00 2Œ0; 1 ; ˇˇx0 x00
ˇ
ˇ< ı )
ˇ
ˇg.x0/ g.x00/ ˇ ˇ< ":
Chọn số tự nhiênn0 Dhc
ı
i
C2 Khi đón0> c
ı Với số tự nhiênn > n0, vớik2 f1; 2; : : : ; ng,
ta có ˇ
ˇ ˇ ˇ
kCc
n k n ˇ ˇ ˇ ˇD c n < c
(101)vn6un6wn; 8nD1; 2; : : : ; lim
n!C1.wn vn/D0; n!C1lim D
1
2˛C1:
Ta có ˇ
ˇ ˇ ˇ
wn
2˛C1
ˇ ˇ ˇ ˇ6j
wn vnj C
ˇ ˇ ˇ ˇ
vn
2˛C1
ˇ ˇ ˇ ˇ
; 8nD1; 2; : : :
Từ sử dụng nguyên lí kẹp suy lim
n!C1wnD
1
2˛C1 )n!C1lim unD
1
2˛C1:
Bài tốn 33 Tính giới hạn lim
n!C1
n2 12233: : : nn n24
.
Lời giải VớinD1; 2; : : :, đặtun Dn2 n
Y
kD1
kk
! n2
Khi
lnun D2lnn
n2
n
X
kD1
klnk:
Do hàm sốf x/Dxlnxđồng biến trênŒ1;C1/nên vớikD2; 3; : : :, ta có
n n
X
iD1 f
k 1C i
n
6f k/6
n n
X
iD1 f
kC i
n ) k Z k
f x/dx6f k/6
kC1
Z k f x/dx ) n Z
f x/dx
n
X
kD2
f k/6
nC1
Z f x/dx ) n Z
xlnxdx
n
X
kD2
klnk6
nC1
Z
2
xlnxdx; 8nD2; 3; : : :
(102)n
2 lnn
n
4 C
1
4
X
kD2
klnk nC1/
2 ln.nC1/
.nC1/
4 2ln2C1:
Do
2.nC1/2
n2 ln.nC1/C
.nC1/2
n2 C
8ln2
n2
4
n2
n
X
kD2
klnk 2lnnC1
n2:
Suy
2lnn 2.nC1/
2
n2 ln.nC1/C
.nC1/2
n2 C
4.2ln2 1/
n2 6lnun61
1
n2:
Ta có lim
n!C1
1
n2
D 1; lim
n!C1
"
.nC1/2
n2 C
4.2ln2 1/
n2
#
D Theo quy tắc Lôpitan,
ta có lim
n!C1 lnn
.nC1/2
n2 ln.nC1/
!
D lim
n!C1
n2lnn nC1/2ln.nC1/
n2
D lim
n!C1
2nlnn Œ2.nC1/ln.nC1/C1
2n
D lim
n!C1
2lnnC2 Œ2 ln.nC1/C2
2
D lim
n!C1ln
n
nC1 D2ln1D0:
Như lim
n!C1lnun D1)n!C1lim unDn!C1lim e
lnun Den!C1lim lnun/De1 De:
Bài toán 34 Cho hàm số liên tụcf; gWŒ0; 1!Œ0;C1/ Chứng minh rằng
lim n!C1 " n n X
kD0
s f k n Cg
kC1
n # D Z p
f x/Cg.x/dx:
Lời giải Đặt
un D
n n
X
kD0
s f k n Cg k C1 n
; vnD
n n
X
kD0
s f k n Cg k n :
VớinD1; 2; : : :, ta có
jun vnj D
n ˇ ˇ ˇ ˇ ˇ n X
kD0
(103)8x0; x00 2Œ0; 1 ; ˇˇx0 x00 ˇ
ˇ< ı)
ˇ
ˇg.x0/ g.x00/ ˇ ˇ< "2:
Chọn số tự nhiên n0 D
1
ı
C Khi n0 >
ı Với số tự nhiên n > n0, với k
f0; 1; : : : ; n 1g, ta có
ˇ ˇ ˇ ˇ
kC1
n k n ˇ ˇ ˇ ˇD n <
n0 < ı:
Như vậy, với số tự nhiênn > n0, vớik2 f0; 1; : : : ; n 1g, ta có
ˇ ˇ ˇ ˇ g k C1 n g k n ˇ ˇ ˇ ˇ
< "2:
Suy ra06 n
n
X
kD0
s ˇ ˇ ˇ ˇ g
kC1
n g k n ˇ ˇ ˇ ˇ <
n n"D";8n > n0:Do
lim n!C1 n n X
kD0
s ˇ ˇ ˇ ˇ g k C1 n g k n ˇ ˇ ˇ ˇ !
D0) lim
n!C1jun vnj D0: Mà
lim
n!C1vnDn!C1lim
1 n
n
X
kD0
s f k n Cg k n ! D Z p
f x/Cg.x/dx;
cho nên lim
n!C1unD
1
Z
0
p
f x/Cg.x/dx:
Bài toán 35 Tính lim
n!C1
1 n
n
X
iD1 n
X
jD1
i Cj
i2Cj2.
Lời giải Theo định lí Stolz, ta có
lim n!C1 n n X
iD1 n
X
jD1
i Cj
i2Cj2 Dn!C1lim
0 @
nC1
X
iD1 nC1
X
jD1
iCj
i2Cj2
n
X
iD1 n
X
jD1
iCj
i2Cj2
(104)iD1jD1i
2Cj2 D iD1
@
jD1i
2Cj2 Ci2C.nC1/2A
D
n
X
iD1
0 @
n
X
jD1
iCj
i2Cj2 C
iC.nC1/
i2C.nC1/2
1 A
C
n
X
jD1
.nC1/2Cj
.nC1/2Cj2 C
.nC1/C.nC1/
.nC1/2C.nC1/2
D
n
X
iD1 n
X
jD1
iCj
i2Cj2 C
n
X
iD1
iC.nC1/
i2C.nC1/2
C
n
X
jD1
.nC1/2Cj
.nC1/2Cj2 C
2.nC1/
2.nC1/2
D
n
X
iD1 n
X
jD1
iCj
i2Cj2 C2
n
X
iD1
iC.nC1/
i2C.nC1/2 C
1
nC1:
Như lim n!C1 n n X
iD1 n
X
jD1
iCj
i2Cj2 Dn!C1lim
"
2 n
X
iD1
iC.nC1/
i2C.nC1/2 C
1
nC1
# D lim n!C1 6
nC1
n
X
iD1 i
nC1 C1
i2
.nC1/2 C1
C
nC1
3 7 D lim n!C1 "
nC1
n
X
iD1 f
i
nC1
C
nC1
#
;
với f hàm số f x/ D xC1
x2C1 Chia đoạnŒ0; 1 điểm chia xi D i
nC1 i D
0; 1; : : : ; nC1/và chọn˛i D i
nC1 2Œxi 1; xi;8i D1; 2; : : : ; n:
Khi tổng tích phân hàm sốf x/trênŒ0; 1ứng với phép phân nói
nC1
n
X
iD1 f
i
nC1
:
Do hàm sốf khả tích nên
lim
n!C1
1
nC1
n
X
iD1 f
i
nC1
D
1
Z
0
xC1
x2C1dx D
4 C
1
(105)nghiệp
[3] W.J Kaczkor, M.T Nowak, 2003,Bài tập giải tích 1,NXB Đại Học Sư Phạm
[4] W.J Kaczkor, M.T Nowak, 2003, Problems in Mathematical Analysis III Integration,
American Mathematical Society
(106)HÀM PHẦN NGUYÊN
Nguyễn Tất Thu
(Trường THPT Chuyên Lương Thế Vinh - Đồng Nai)
GIỚI THIỆU
Phần nguyên khái niệm xây dựng đơn giản số học, lại có nhiều ứng dụng khơng Tốn học mà cịn có nhiều ứng dụng tốn ứng dụng công nghệ thông tin Trong viết này, trình bày số định lí phần ngun số toán liên quan đến phần nguyên
1 Khái niệm hàm phần nguyên
1.1 Định nghĩa
Định nghĩa Với số thựcx, có số nguyênnthoản6x < nC1:Chúng ta nói nlà số tự nhiên lớn nhỏ bằngxlà phần nguyên củax.
Ký hiệun D Œx(hoặcn D bxc) Hiệux Œxđược gọi phần lẻ (phần thập phân) củaxvà
kí hiệufxg.
1.2 Tính chất bản
Sử dụng định nghĩa phần nguyên, ta dễ dàng chứng minh tính chất sau
Định lý Ta có
1 Nếux > y thìŒx>Œy :
2 Vớix; y là số thực, ta cóŒxCŒy6ŒxCy 6ŒxCŒyC1.
3 Nếuavàb là số nguyên,b > 0, vàq là thương phép chiaachob thìq Da b
. 4 Với số thựcxvà số nguyênn, ta cóŒxCnDŒxCn.
5 Với số thực dươngxvà số nguyên dươngnthì số bội ngun dương củankhơng
vượt qxlàx n
(107)Mặt khác 6f g C f g nên ta có điều phải chứng minh
5: Ta ý bội n là1n; 2n; : : : ; k n: Ta cók n x < k C1/n: Đó k xn < kC1và kết thúc chứng minh
6: Ta đặtbxc Dm; fxg D ˛ Từ thuật tốn phép chia tính chất ta có mDnjm
n
k
Cr; 06r 6n 1:
Chúng ta có được06r C˛ 6n 1C˛ < n, đórCn˛˘D0và
jx
n
k
D
mC˛ n
D
jm
n
k
Cr C˛
n
Djm
n
k
C
r C˛ n
Djm
n
k
D
bxc
n
:
7: Ta cóxDŒxC fxg Suy
Œ2x ŒxDŒ2ŒxC2fxg ŒŒxC fxgDŒ2fxg ŒfxgDŒ2fxg :
Do
ı Nếu 12 6fxg < 1thì
162fxg< 2)Œ2fxgD1)Œ2x ŒxD1:
ı Nếu06fxg< 12
062fxg< 1)Œ2fxgD0)Œ2x ŒxD0:
1.3 Công thức Polignac
Định lý Choplà số nguyên tố vànlà số nguyên dương, số mũ số nguyên
tốptrong phân tích tiêu chuẩn củanŠđược tính theo cơng thức
vp.nŠ/D
X
iD1
n pi
(108)
WZC !R
là hàm thoả i) f k/
p không số nguyên vớikD1; 2; : : : ; p 1;
ii) f k/Cf p k/là số nguyên chia hết chop với mọik D1; 2; : : : ; p 1.
Khi đó
p
X
kD1
f k/q p D p q p X
kD1
f k/ p
2 : (1)
Lời giải Từ ii) ta có
qf k/ p C
qf p k/
p Z: (2)
Và từ (1), ta có qf k/
p … Zvà
qf p k/
p … Z,kD1; 2; : : : ; p Do
0 < qf k/ p C
qf p k/ p
< 2:
Nhưng từ (2),
qf k/ p C
qf p k/ p Z nên qf k/ p C
qf p k/ p
D1; k D1; 2; : : : ; p 1:
Lấy tổng hai lần ta có
p
X
kD1
qf k/
p
D p
2 :
Nên
p
X
kD1
p qf k/
p
X
kD1
f k/q p
D p
2 :
Từ ta có điều phải chứng minh
Định lý Choa; clà số thực không âm vàf WŒaIb !ŒcId là song ánh hàm
tăng Khi đó
X
a6k6b
bf k/c C X c6k6d
f 1.k/˘
n Gf D bbc bdc ˛.a/˛.c/; (3)
với k là số nguyên, n Gf
là số điểm có toạ độ số tự nhiên đồ thị hàm f và ˛ WR!Zđược xác định bởi
˛.x/D
8 <
:
bxc nếu x 2RnZ
(109)n M1/D a6k6b
bf k/c; n M2/D c6k6d
f k/ ;
n M3/D bbc bdc; n M4/D˛.a/˛.c/:
Ta có
n M1/Cn M2/ n M1\M2/Dn M1[M2/ :
Vì
n M1/Cn M2/ n GfDn M3/ n M4/ :
Từ kết thúc chứng minh
Định lý Chom; n; slà số nguyên dương,m6n Khi đó
s
X
kD1
km n
C X
16k6ms n
k n m
Ds jms
n
k
C
gcd
.m; n/s n
: (4)
Lời giải Trước hết ta chứng minh bổ đề sau Bổ đề Trong dãy 1m
n ; 2m
n ; : : : ; sm
n ;chứa đúng
gcd
.m; n/s n
(110)
Vìn1 D dn D gcd.m;n/n nên có
j
pgcd.m;n/ n
k
số nguyên dãy Bổ đề chứng minh Để chứng minh kết định lí ta xét hàmf WŒ1Is!m
nI ms
n
,f x/D mnxthoả định lí4:
Theo bổ đề ta cón GfD
jgcd.m;n/
s n
k
ta có điều phải chứng minh
Định lý Choa; clà số thực không âm vàf WŒaIb !ŒcId là song ánh hàm
giảm Khi đó
X
a6k6b
bf k/c X c6k6d
f 1.k/˘
D bbc˛.c/ bdc˛.a/;
vớiklà số nguyên và˛là hàm nói định lí 4.
Lời giải Hàmf song ánh tăng nên liên tục Xét tập N1 D
˚
.xIy/2 R2ja6x 6b; c 6y 6f x/ ; N2 D
˚
.xIy/2 R2ˇˇc 6y 6d; a 6x6f 1.y/ ;
N3 D
˚
.xIy/2 R2ja6x 6b; 06y 6c ; N4 D
˚
.xIy/2 R2j06x 6a; c6y 6d :
Khi
X
a6k6b
bf k/c Dn N1/Cn N3/ ;
X
c6k6d
f 1.k/˘
Dn N2/Cn N4/ ; n N1/Dn N2/ ;
và
n N3/D.bbc ˛.a// ˛.c/; n N4/D.bdc ˛.c// ˛.a/:
Vì
X
a6k6b
bf k/c X c6k6d
f 1.k/˘
(111)f
xC
n
D
xC
n
C
xC
n C n
C C
xC
n C n
n
n
xC
n
D
xC
n
C
xC
n
C CŒxC1 ŒnxC1 :
Áp dụngŒxCkDŒxCkvớik2Z, ta
f
xC
n
Df x/ 8x2R:
Do đóf hàm tuần hồn với chu kì 1n Vì ta cần nghiên cứuf x/trong0 x < 1n
Nhưngf x/D0với mọix
0I1n
, nênf x/D0; 8x2 Rvà chứng minh hồn tất
2 Một số ví dụ áp dụng
Ví dụ Tìm tất số nguyên dươngnsao chojpn 111kchia hết111.
Lời giải Các ước nguyên dương của111là1; 3; 37; 111:Ta có trường hợp sau
1 jpn
111kD1hay16111 < 2n, ta cón>7
2 jpn
111kD3hay3n6111 < 4n, ta cónD4
3 jpn
111kD37hay37n6111 < 38n, điều xảy
4 jpn
111kD111haynD1
Bởi vậynD1; nD4hoặcn>7
Ví dụ Giải phương trình tập số thực
(112)b c6 b c
x D 1) bxc D Thìxbxc D1nênx D 1là lời giải
x 1I0/ Chúng ta cóbxc D 1nênxbxc D x < 1, (loại)
x Œ0I1/, thìbxc D 0và xbxc D < 1, nên khơng có lời giải trường
hợp
x 2Œ1I2/chúng ta cóbxc D1vàxbxc Dxthoả tốn
x >2thìbxc>2vàxbxc>4mâu thuẫn với yêu cầu đề
Vì vậyx2 f 1g [Œ1I2/
Ví dụ Tìm tất số thựcx > 1sao cho pn
bxnc.n> 2/là số nguyên dương với số
nguyênn.
(Romanian Regional Mathematical Contest 2004)
Lời giải Đặt pn
bxnc Da
n, đóbxnc Dannvàann6xn< annC1
Do đóan6x < n
p
an
nC1nênbxc Dan Chúng ta thấy, toán thoả mãn với số nguyên
x; x >2/
Giả sử có giá trị củaxthoả tốn vàxkhơng phải số ngun Đặtx DaC˛; a2 Z; a>1; < ˛ < Khi đóan< aC˛/n< anC1,
1 < 1C ˛
a
n
< 1C
an 62: (5)
Mà theo bất đẳng thức Bernoulii, ta có 1C˛an
> 1Cn˛a > 2vớinđủ lớn Dẫn đến mâu
thuẫn với (5) Vậy toán giải
Ví dụ (THTT T10/466) Chứng minh rằng
.3n/Š
nŠ nC1/Š nC2/Š;
là số nguyên với số nguyênn>3:
Lời giải Để chứng minh toán, ta cần chứng minh Với số ngun tốpta ln có
1
X
kD1
3n pk
>
1
X
kD1
n pk
C
nC1 pk
C
nC2 pk
(113)
n a C
2 >
nC2 a :
Từ đó, suy
3n a
>hn
a
i
C
nC1 a
C
nC2 a
: (7)
Do đó, nếup >3thì sử dụng (7) ta có (6)
Xétp D2, (6) trở thành
1
X
kD1
3n 2k > X
kD1
h n 2k i C
nC1 2k
C
nC2 2k
: (8)
Vớin>4ta có
log2.3n/ log2.nC2/Dlog2
3n
nC2 >1;
Nên tồn số nguyên dươngqsao cho
log2.nC2/6q 6log2.3n/ )nC2 <
q < 3n: (9)
Và với n D ta chọn q D Do với n > tồn số nguyên q thỏa (9) Vi k >2; kÔq , ỏp dng (7) ta có
3n 2k
>h n
2k
i
C
nC1 2k
C
nC2 2k
;
Do để chứng minh (8) ta cần chứng minh
3n C 3n 2q
>hn
2 i C n C1 C n C2
Ch n
2q i C n C1 2q C n C2 2q : (10)
Bẳng quy nạp ta chứng minh
3n
2
Dhn
2 i C n C1 C n C2 1; h n 2q i D
nC1 2q
D
nC2 2q D0; 3n 2q >1:
(114)D
2 C C C n
Lời giải Ta cóS > n Mặt khác với x > 0và k > 2là số nguyên dương, ta có bất
đẳng thức
.1Cx/k > 1Ckx )1Cx > pk 1Ckx:
ChoxD k12 ta
k
r
1C
k 61C k2;
Do
S D n
X
kD2 k
r
1C
k
n
X
kD2
1C
k2
<
n
X
kD2
1C
k.k 1/
D n
X
kD2
1C
k 1 k
Dn
n < n:
Suy raŒS Dn 1:
Ví dụ Chop là số nguyên tố lẻ Chứng minh rằng
p
X
kD1
k3 p
D p 1/.p 2/.pC1/
4 :
(German Mathematical Olympiad 2002)
Lời giải Hàmf x/Dx3thoả mãn điều kiện i) ii) định lí nên ta có p
X
kD1
k3q p
D q
p
.p 1/2p2
p D
.p 1/.p2q pq 2/ :
Choq D1ta có điều phải chứng minh
Ví dụ Tính
SnD n.nC1/
2
X
kD1
$
1Cp1C8k
%
:
Lời giải Xét hàm số
f WŒ1In!
1In.nC1/
2
; f x/D x.xC1/
2
Hàmf tăng song ánh Ta thấy n Gf
Dnvàf 1.x/D 1C p
(115)D
2 Cn 12 D :
Từ dẫn đến kết luận tốn
Ví dụ (VN TST 2005) Cho số nguyên tốp.p > 3/ Tính
a) S D p
2
P
kD1
h
2k2 p
i
2hkp2inếup1 mod 4/:
b) P D p
2
P
kD1
h
k2 p
i
nếup mod 8/.
Lời giải Trước hết ta có bổ đề sau
Bổ đề Vớip là số nguyên tố thỏap mod4/thì số tự nhiênavới16 a6 p 12 sẽ
tồn số tự nhiênb thỏa pC21 6b 6p 1vàa2Cb20 mod p/.
Thật vậy, theo định lí Wilson.p 1/Š mod p/ Với mỗik D1; 2; 3; : : : ;p 12 , ta thấy p k k mod p/)k.p k/ k2 mod p/:
Kết hợp với giả thiếtp mod 4/) p 12 ::: 2, ta
1.p 1/Š 1/p21
p 1
2
Š
2
p 1
2
Š
2
.mod p/:
Đặt' D p 12
Š )'2 1 mod p/ Với mỗi16a6 p
2 , ta chọn pC1
2 6b6p 1thỏa
b2 a2:'2 mod p/, dễ thấybtồn Khi
a2Cb2 a2.1C'2/0 mod p/:
Bổ đề chứng minh a) Ta thấy tổng cho
S D p
2
X
kD1
2k2
p
2
k2
p
(116)2 tồn số tự nhiênb thỏa 6b 6p 1sao cho
a2Cb20 mod p/)a2C.p b/2 mod p/
do đó, tồn số tự nhiêna0thỏa16a06 p 12 cho a2Ca02 mod p/:
Gọix; y số số dư phép chiak2chop (16 k6 p 12 ) có giá trị lớn p 12
và nhỏ p
2 Theo nhận xét thìx Dy,
xCy D p
2 )x Dy D p
4 :
Từ ta cóS Dx1Cy0D p 14 Do đó, tổng cần tìm p
4
b) Dop mod8/nên tồn tạiasao choa22 mod p/ Ta thấy p mod 8/)p mod4/:
Ta có
P D p
2
X
kD1
k2 p D p X
kD1
2k2 p k2 p p X
kD1
2k2 p k2 p D p X
kD1
2k2 p k2 p S :
Ta cần tính
p
X
kD1
2k2 p k2 p D p X
kD1
2k2 p k2 p p X
kD1
2k2 p k2 p D p X
kD1
k2 p p X
kD1
2k2 p k2 p ;
trong đóp mod8/
Theo nhận xét tập hợp số dư chiak2; 16k p
2 choptrùng với tập hợp
số dư chia2k2; 16k6 p
2 chop, tức p
2
X
kD1
2k2 p k2 p D0; suy p X
kD1
2k2 p k2 p D p X
kD1
k2 p D p 1 24 :
VậyP D p2241 p
4 D
(117)nên
.i C1/2 2014
DxC1:
Từ suy dãy
12
2014
;
22
2014
; : : : ;
10072
2014
gồm504số tự nhiên khác từ0đến503 Khii > 1007các số hạngh2014i2 ităng thực nên
dãy
10082
2014
;
10092
2014
; : : : ;
20142
2014
có đúng1007số tự nhiên khác Vậy có tất là504C1007D1511số tự nhiên khác
trong dãy số ban đầu
Ví dụ 10 Tìm số nguyên dươngx; y thỏa mãn x Dypx:
Lời giải Ta có
p
x
>1vàp
x
6px <p
x
C1,
nên
p
x2
6x < p
x
C12
:
Do
p
x2
6yp
x
< p
x
C12
;
Hay
p
x6y <pxC2C p1
x <
p
xC3:
Nên ta có trường hợp sau
y Dp
x
, đóx Dp
x2
Từ suy
x Dk2
(118)y Dp
x
C2ta cóx Dp
x2
C2p
x
Từ ta có x Dk C2k
y DkC2 ; k 2N:
Từ dẫn đến kết luận tốn
Ví dụ 11 (APMO 2001) Tìm số nguyên dươngN lớn nhất, cho tập hợpf1; 2; : : : ; Ng
số số chia hết cho3bằng số số chia hết cho5hoặc chia hết cho7:
Lời giải Theo đề ta có
N D N C N N 35 :
ĐặtN D35kCr với06r < 35và ta xétk >1 Ta có
35k Cr D 35k Cr C 35k Cr 35k Cr 35
D7kChr
5
i
C5kChr
7
i
k h r 35
i
D11kChr
5
i
Chr
7
i
:
Mặt khác, ta cóN
> N
3 , nên suy
35kCr
3 611kC
hr
5
i
Chr
7
i
611kC r
5C r 7;
hay 2k 35r C1 < 3nênk ) N 69 Bằng cách xét vớiN D 69; 68; 67; 66; 65ta
thấy cóN D65thỏa VậyN D65là số cần tìm
Ví dụ 12 Tìm số nguyên tốp nhỏ cho
h
3Cpp2ni
C1;
chia hết cho2nC1 với số tự nhiênn:
Lời giải Vớip D2ta có
3Cp2
4
C1D378 6::: 23:
Vớip D3ta có
3Cp3
2
C1D23 6::: 22:
Vớip D5 Đặt
x1 D
3Cp5
2
; x2D
3 p5
2
vàanDx1nCx n
(119)Ví dụ 13 Cho dãy số n W n D Chứng minh rằng, dãy số cho chứa vơ hạng số là
số phương.
Lời giải Ta có
p
2C1
2mC1 D
2mC1
X
iD0
C2mi C1.p2/i D m
X
iD0
C2m2iC12i C m
X
iD0
C2m2iCC112ip2Dx2mC1Cy2mC1 p
2
p
2
2mC1 D
m
X
iD0
C2m2iC12i
m
X
iD0
C2m2iCC112ip2Dy2mC1 p
2 x2mC1:
Trong
x2mC1 D m
X
iD0
C2m2iC12i; y2mC1 D m
X
iD0
C2m2iCC112i
vớimD0; 1; 2; : : :Ta cóx2mC1; y2mC1là số nguyên dương
Từ cách xác địnhx2mC1; y2mC1,ta có
1Dp2C1
2mC1p
2
2mC1
Dp2y2mC1Cx2mC1 p
2y2mC1 x2mC1
D2y2m2 C1 x2m2 C1:
Suy ra1Cx2m2 C1 D2y2m2 C1 Do
x2m4 C1Cx2m2 C1 D2.x2mC1y2mC1/2:
Mà
x2m4 C1 < x2m4 C1Cx2m2 C1 < x2m2 C1C12
)x2m2 C1 <
q
x2m4 C1Cx2m2 C1 < x2m2 C1C1:
Suy
q
x2m4 C1Cx2m2 C1
Dx2m2 C1 )hx2mC1y2mC1 p
2i Dx22mC1:
Đặtbm Dx2mC1y2mC1, ta cóbmlà dãy số dương dãy tăng thực nênfbmgnhân vô
số giá trị nguyên dương vàubm Dx
(120)AD fb˛c;b2˛c;b3˛c; : : :g vàB D fbˇc;b2ˇc;b3ˇc; : : :g
lập thành hai "phân hoạch" tập số nguyên dươngN Tức là
A\B D ;vàA[B DN:
Lời giải Trước tiên ta chứng minh tách rời A \B D ;: Thật vậy, giả sử 9i; j
chobi ˛c D bjˇc D k Khi đó, vìi ˛ jˇ số vơ tỷ nên k < i ˛ < k C1và k < jˇ < kC1hay
i kC1 <
1 ˛ <
i k
j kC1 <
1 ˇ <
j k:
Cộng bất đẳng thức lại theo vế, ta
iCj kC1 <
1 ˛ C
1
ˇ D1 < iCj
k )k < i Cj < kC1) vơ lý!
Vậy ta cóA\B D ;:
Tiếp theo, ta chứng minh số tự nhiênnbất kỳ phải có mặt trongAhoặc trongB Thật
vậy, phương pháp phản chứng, ta giả sửnkhông xuất hai dãy trên,
đó9i; j cho
(
i ˛ < n
.i C1/˛ > nC1
(
jˇ < n
.j C1/ˇ > nC1
hay
i n <
1 ˛ <
i C1 nC1
j n <
1 ˇ <
j C1 nC1:
Cộng bất đẳng thức lại theo vế, ta
iCj n <
1 ˛ C
1
ˇ D1 <
i Cj C2
nC1 )i Cj < n < i Cj C1) vô lý!
Như ta cóA[B DN:
Sau chúng tơi giới thiệu số tập để bạn đọc nghiên cứu
3 Bài tập
Bài tập Tìm số hạng phân biệt dãy số hữu hạnj1998k2 k, vớikD1; 2; : : : ; 1997
(121)Bài tập Cho dãy số nguyên dương.un/thỏa
(
u0 D1; u1D3
unC2D1C
hu2 nC1 un
i
; n>0
Chứng minh rằngunC2un u2nC1 D2
nvới số tự nhiênn. Bài tập Chonlà số ngun dương Tìm cơng thức tổng
n
C1
C
n
C2 22
C C
n
C2k 2kC1
C Bài tập Tình tổng
X
06i <j6n
x
Ci j
;
vớixlà số thực
Bài tập So sánh giá trị tổng sau 2000
X
kD0
3k C2000 3kC1
và
2000
X
kD0
3k 2000 3kC1
:
Bài tập a) Chứng minh có vơ số số hữu tỉ dươngxthoả
˚
x2 C fxg D 0; 99:
b) Chứng minh không tồn số hữu tỉ dươngxthoả˚x2 C fxg D1:
(2004 Romanian Mathematical Olympiad)
Bài tập Cho số hữu tỉ˛1; ˛2; : : : ; ˛nthoả n
X
iD1
fk˛ig <
n
(122)iD1
nguyên
Bài tập 10 Chứng minh n
X
kD1
n2 k2 D n2 X
kD1
n
p
k
;
với số nguyênn>1
Bài tập 11 Cho số vơ tỉ dương Khi với số nguyên dươngmta có
m
X
kD1
bkc C bmc
X
kD1
k
Dmbmc:
Bài tập 12 Cho p q số nguyên dương nguyên tố m số thực thoả 16m < p
1 Nếus Djmqp kthì b mc
P
kD1
jkq p k C s P
kD1
jkp
q
k
D bmcs:
2 Nếup vàqlà số lẻ
p
X
kD1
kq p C q X
kD1
kp q
D p 1/.q 1/
4
Bài tập 13 Chop số nguyên tố lẻ,q số nguyên không chia hết choq Chứng minh
p
X
kD1
1/k k2q p
D p 1/.q 1/
2 :
Bài tập 14 Chop số nguyên tố lẻ Chứng minh
p
X
kD1
kp k p
pC1
2 mod p/:
Bài tập 15 Chứng minh với số nguyên dươngnthì
p
2Cp3
2n
không chia hết cho5:
Bài tập 16 Chứng minh phương trình
j
np2kC2nDjmp2k
(123)D
Tài liệu
[1] Hà Huy Khoái,Các chuyên đề bồi dưỡng số học, NXB Giáo dục, năm2006:
[2] Titu Andreescu and Dorin Andrica,2009:Number Theory: Structures, Examples, and
Prob-lems (179-188)
(124)TỪ ĐA THỨC CHEBYSHEV ĐẾN BẤT ĐẲNG THỨC
BERSTEIN - MARKOV
Trịnh Đào Chiến
(Trường Cao đẳng Sư phạm Gia Lai) GIỚI THIỆU
Đa thức Chebyshev liên kết đẹp đẽ Đại số với Lượng giác Định lí Berstein - Markov mơ tả mối quan hệ đa thức với đạo hàm Đã có nhiều cơng trình nghiên cứu vấn đề Từ Đa thức Chebyshev đến Định lí Berstein - Markov chặng hành trình mà viết làm công việc người lữ khách, ngồi tỉ mẩn kể lại vài kỷ niệm sau chuyến xa
Thời sinh viên, lần đầu tiếp xúc với khái niệm Đa thức Chebyshev, người viết thắc mắc: lại định nghĩa cách “áp đặt” rằngTn(x) = cosnα, khix=cosα? Phải
đến thời gian dài, “ngấm” tinh tế
Một vài kiến thức nội dung viết vượt kiến thức phổ thơng, trang bị chương trình đào tạo cho sinh viên năm thứ nhất, ngành toán học
Bây giờ, giả sửTn(x)là đa thức biến sốx ∈ R, bậcn Ta biết rằng, với mỗix ∈ [−1,1]luôn
tồn nhấtα∈[0, π]sao chox=cosα, đóx=−1khi khiα=π,x= 1khi
và khiα= Khi , hàm sốy= cosxlà hàm giảm nghiêm ngặt đoạn[−1,1]nên α =arcosxvàTn(x) =Tn(cosα)là đa thức bậcntheo cosα
Ngồi ra, theo Cơng thức Moivre,cosnαcũng biểu diễn đa thức bậcntheo cosα
Lưu ý rằng, vớiTn(x)là đa thức xác định trước đa thức bậcntheo cosαnày thường
là khác
Một cách tự nhiên, người ta nghĩ đến việc bổ sung thêm điều kiệnTn(x) = cosnα,
x=cosα, cho đa thứcTn(x) Khi đó, khái niệm Đa thức Chebyshev hình thành, đặt
(125)T2(x) = T2(cosα) = cos 2α= 2cos2α−1 = 2x2−1, T3(x) = T3(cosα) = cos 3α= 4cos3α−3 cosα= 4x3 −3x,
T4(x) = T4(cosα) = cos 4α= 8cos4α−8cos2α+ = 8x4−8x2+ 1,
Hơn nữa, vớin ≥2, vìcosnα+ cos (n−2)α = cosαcos (n−1)αnên ta có Tn(x) =Tn(cosα) = cosnα= cosαcos (n−1)α−cos (n−2)α
= 2xTn−1(x)−Tn−2(x)
VậyTn(x)được xác định theo hệ thức truy hồi
T0(x) = 1, T1(x) =x, Tn(x) = 2xTn−1(x)−Tn−2(x)
Dưới số tính chất Đa thức Chebyshev loại
Tính chất 1.a)Tn(x) = cos (n.arcosx)với mọix∈[−1,1].
b)|Tn(x)| ≤1với mọix∈[−1,1].
Chứng minh. Thật vậy, với x ∈ [−1,1], ta có Tn(x) = cosnα = cos (n.arcosx) Suy
|Tn(x)| ≤1với mọix∈[−1,1]
Tính chất 2. Tn(x)là đa thức với hệ số nguyên, bậcn, hệ số củaxn bằng2n−1 và hàm số
chẵn khinchẵn, hàm số lẻ khinlẻ.
Chứng minh. Từ hệ thức truy hồi xác địnhTn(x)nêu trên, dễ dàng suy Tn(x)là đa
(126)là hàm số lẻ Khi đó, với mọin≥2, ta có
+ Nếunlà số chẵn, giả sử rằngTn−2(x)là hàm số chẵn vàTn−1(x)là hàm số lẻ, ta có Tn(−x) = (−x)Tn−1(−x)−Tn−2(−x) = (−x) (−Tn−1(x))−Tn−2(x)
= 2xTn−1(x)−Tn−2(x) = Tn(x)
Do đóTn(x)là hàm số chẵn
+ Nếunlà số lẻ, giả sử rằngTn−2(x)là hàm số lẻ vàTn−1(x)là hàm số chẵn, ta có Tn(−x) = (−x)Tn−1(−x)−Tn−2(−x)
= (−x) (Tn−1(x))−(−Tn−2(x)) =−2xTn−1(x) +Tn−2(x) = −Tn(x)
Do đóTn(x)là hàm số lẻ
Tính chất 3.Tn(x)có đúngnnghiệm phân biệt đoạn[−1,1], là
xk =cos
2k−1 2n π
, k = 1,2, , n
Hơn nữa, Tn(x)đạt cực trị điểm x∗k = cos
kπ n
, gọi điểm luân phiên (hoặc luân điểm) của Tn(x), với giá trị cực trị tương ứng là Tn(x∗k) = (−1)k,
k = 1,2, , n−1.
Chứng minh.Với mỗik = 1,2, , n, ta có Tn(xk) = cos (n.arcos(xk)) = cos
n.arcos
cos
2k−1 2n π
= cos
2k−1 π
=
Hơn nữa, với mỗik = 1,2, , n−1, ta có Tn
0
(x) = d
dx(cos (n.arccosx)) =
nsin(n.arccosx)
√
1−x2
Do
Tn
0
(x∗k) =
nsin(n.arccosx)
√
1−x2 =
nsin n.arccos cos kπ n
q
1−cos2 kπ n
= nsinsin (kπ)kπ n
(127)√
1−x2 −
Chứng minh.Ta cần chứng minh rằng, đa thức Tn(x)xác định khoảng mở (−1,1),
tương ứng với hàm trọng hàm mật độω(x) = √
1−x2, thỏa mãn tính chất sau
I =
Z
−1
Tn(x)Tm(x)
√
1−x2 dx=
0, nếun 6=m, π
2, nếun=m6= 0, π, nếun =m =
Thật vậy, vớin ≥0,m ≥0, ta có I =
1
Z
−1
Tn(x)Tm(x)
√
1−x2 dx=
Z
−1
cos (n.arccosx) cos (m.arccosx)√ dx
1−x2
Vớiα=arcosx, ta códα=−√ dx
1−x2 Vậy I =−
0
Z
π
cos (nα) cos (mα)dα=
π
Z
0
cos (nα) cos (mα)dα
=
π
Z
0
cos (n+m)α+ cos (n−m)α dα
- Nếun6=m, I =
1
2 (n+m)sin(n+m)α+
2 (n−m)sin(n−m)α
π
0 =
- Nếun=m6= 0,
I =
1
2 (n+m)sin(n+m)α+ x
π
0 = π
(128)0
Ta có điều phải chứng minh
Một cách tự nhiên, người ta nghĩ đến việc tương tự hóa Định nghĩa hàm sin
Định nghĩa 2. Đa thứcUn(x), x ∈ R, bậcn, gọi Đa thức Chebyshev loại thỏa
mãn điều kiện
Un(x) =
sin(n+ 1)α
sinα , khix=cosα
Miền xác định củaxvàαtương tự đa thứcTn(x)
Nhận xét
U0(x) = U0(cosα) = sinα
sinα = 1, U1(x) = U1(cosα) =
sin2α
sinα =
2sinαcosα
sinα =2cosα= 2x, T2(x) = T2(cosα) =
sin3α
sinα =
sinα(4cos2α−1)
sinα = 4cos
2α−1 = 4x2−1,
T3(x) = T3(cosα) =
sin4α
sinα =
sinα(8cos3α−4 cosα)
sinα = 8cos
3α−4 cosα= 8x3−4x,
Hơn nữa, vớin ≥2, sin(n+ 1)α+sin(n−1)α= cosα.sinnαnên ta có Un(x) =Un(cosα) = sin
(n+ 1)α
sinα =
2 cosα.sinnα−sin(n−1)α
sinα = cosαsinnα
sinα −
sin(n−1)α
sinα = 2xUn−1(x)−Un−2(x)
VậyUn(x)được xác định theo hệ thức truy hồi
U0(x) = 1, U1(x) = 2x,
(129)Tính chất 8. Un(x)có đúngnnghiệm phân biệt đoạn[−1,1], là
xk =cos
k n+ 1π
, k = 1,2, , n
Hơn nữa,Un(x)đạt cực trị điểmx∗k = cos
kπ n
, với giá trị cực trị tương ứng là
Tn(x∗k) = (−1)k,k = 1,2, , n−1.
Tính chất 9.Dãy đa thức{Un(x)}là dãy đa thức trực giao, tương ứng với hàm trọng hàm
mật độω(x) = √1−x2xác định khoảng đóng[−1,1].
Các nội dung phần cho thấy góc liên kết đẹp đẽ Đại số với Lượng giác Đa thức Chebyshev Định lí Berstein - Markov xem sản phẩm liên kết Sau đường dẫn đến định lí này, với số áp dụng toán phổ thơng
Định lí (Berstein).Nếut(x) = 2a0+
n
P
k=1
(akcoskx+bksinkx),thì
t0(x)≤n.sup|t(x)|
Chứng minh. Giả sử, ngược lại, supt0(x) = nl, l > sup|t(x)| Vì t0(x)
là hàm liên tục nên tồn giá trị c cho t0(c) = ±nl Giả sử t0(c) = nl Vì nl
giá trị lớn t0(x), nên t00(c) = Xét hàm số S(x) = lsinn(x−c)−t(x) Ta có r(x) = S0(x) = nlcosn(x−c)−t0(x).S(x)vàr(x)đều có bậcn Xét điểm
u0 =c+ π
2n, uk =u0+ kπ
2n, k = 1,2, ,2n
(130)y2n =y0+ 2π Thế thìS(y2n) =S(y0) =
Bởi Định lí Rolle, tồn khơng-điểm xi r(x) thuộc khoảng (yi, yi+1), i =
0,1, ,2n −1 Dễ thấy x2n−1 < x0 + 2π Ta có r(c) = nl− t
0
(c) = Vì đa thức r(x)có bậcn, có2nkhơng-điểm, nên suy rằng, với mỗik, ta cóc≡xk(mod2π) Nhưng
r0(c) = −t00(c) = Do đóc(và với xk) không - điểm bội (và điểm cực
tiểu) củar(x) Do đór(x)≡ 0vàS(x)là số Tuy nhiênS(u0)>0vàS(u1)<0, mâu
thuẫn Ta có điều phải chứng minh
Hệ 1.Nếu đa thứcP(x)có bậcnvà|P (x)| ≤M, với mọix∈(−1,1), thì
P0(x)≤nM√1−x2.
Chứng minh.Đặtt(α) =P (cosα) Ta cót0(α) =−sinα.P0(cosα) Áp dụng Định lí 1, ta có
điều phải chứng minh
Hệ 2. Giả sử đa thức lượng giácP (x) =
n
P
j=0
(ajcosjx+bjsinjx) thỏa mãn điều kiện
|P (x)| ≤1, với mọix∈R Thế thì, ta cóP0(x)≤n, với mọix∈R.
Ta có kết sau
Bổ đề 1.Giả sử
xk=cos
2k−1 2n π
, k= 1,2, , n
là không - điểm Đa thức ChebyschevTn(x) Nếu đa thứcP (x)có bậcn−1, thì
P (x) = n
n
X
k=1
(−1)k−1p1−xk2P(xk)
Tn(x)
x−xk
Chứng minh.Vìxk =cos
(2k−1)π
2n , nênn.arccosxk =
(2k−1)π
2 Vậy
sin (n.arccosxk) = sin
(2k−1)π
2 = (−1)
k−1
Suy
Tn
0
(x) = (cos (n.arccosx))0 = n.sin (n.√ arccosx)
(131)Suy
nP(x) =n
n
X
k=1
P(xk) n
Y
i=1,i6=k
x−xi
xk−xi
!
=(−1)k−1.p1−xk2.Tn
0
(xk)
n
X
k=1
P (xk) n
Y
i=1,i6=k
x−xi
xk−xi
!
=
n
X
k=1
(−1)k−1.p1−xk2.P(xk)
Tn(x)
x−xk
hay
P(x) = n
n
X
k=1
(−1)k−1.p1−xk2.P (xk)
Tn(x)
x−xk
Ta có điều phải chứng minh
Hệ 3. Giả sử đa thứcPn−1(x)có bậc khơng vượt qn−1, có hệ số cao làa0, thỏa
mãn điều kiện
√
1−x2|P
n−1(x)| ≤1, ∀x∈[−1,1]
Thế thì, ta có|a0| ≤2n−1
Giải.Ta viết đa thức cho dạng Đa thức nội suy Lagrange theo nút nội suy
xk =cos
2k−1 2n π
, k = 1,2, , n,
là nghiệm Đa thức ChebyshevTn(x).Áp dụng Bổ đề 1, ta có
P (x) = n
n
X
k=1
(−1)k−1p1−xk2P(xk)
Tn(x)
x−xk
(132)k=1
Do
|a0| ≤
n−1 n
n
X
k=1
np1−xk2P(xk)
≤
n−1 n n=
n−1.
Bổ đề 2. Giả sử đa thứcQ(x)có bậcn−1và
|Q(x)| ≤ √
1−x2, ∀x∈(−1,1)
Thế thì|Q(x)| ≤n, ∀x∈[−1,1]
Chứng minh.Giả sử rằng−x1 =xn ≤x≤x1 Thế thì, Bổ đề 1, ta có
√
1−x2 ≥p1−x12 = sin π 2n ≥
1 n
Do Bổ đề vớixn≤x≤x1
Nếux1 < x≤1(hoặc−1≤x < xn), Bổ đề 1, ta có
|Q(x)| ≤
n
n
X
k=1
Tn(x)
x−xk
Vậy
Tn(x) = 2n−1 n
Y
k=1
(x−xk)
Suy
Tn
0
(x) Tn(x)
=
n
X
k=1 x−xk
Do
|Q(x)| ≤
n
Tn
0
(x)
Vớix=cosα, ta có
Tn
0
(x) = n.sinnα sinα
Khi đó, ta cóTn
0
(133)sint
Đặtcost =x Khi đó|x| ≤1vàP (t) = sintPn−1(cost) =
√
1−x2P
n−1(x).Nhận xét P (x)thỏa mãn điều kiện Bổ đề 2, nên|Pn−1(x)| ≤n,∀x∈[−1,1] Do
Psin(t)t
≤n, ∀t∈R\ { ,−2π,−π,0, π,2π, }
Từ kết trên, ta có định lí kinh điển sau
Định lí (Berstein - Markov). Giả sử đa thứcP (x)có bậcn, thỏa mãn điều kiện |Pn(x)| ≤
1,∀x∈[−1,1].Khi đó
Pn
0
(x)≤n2, ∀x∈[−1,1]
Bất đẳng thức cuối định lí gọi Bất đẳng thức Berstein - Markov
Chứng minh.Đặtx=cosα Khi đó, giả thiết, ta có|P (cosα)| ≤1
Hơn nữaP (cosα)có dạngP(cosα) = Pn
k=0
(akcoskx+bksinkx), nên áp dụng Định lí 1, ta có
sinα.P0(cosα)≤1 Suy
√
1−x2.
P
0
(x) n
≤1
Dó đó, áp dụng Bổ đề 3, ta có
P
0
(x) n
≤n
(134)nên người ta thường tìm kiếm lời giải phổ thơng cho tốn Sau số minh họa cho phương pháp
Xét đa thứcP(x) =ax+b, thỏa mãn điều kiện|P (x)|=|ax+b| ≤1,∀x∈[−1,1].Áp dụng
Bất đẳng thức Berstein - Markov, ta cóP0(x)≤n2, hay|a| ≤1, ∀x∈[−1,1].
Ta thiết lập toán sau
Bài toán (Olympic sinh viên VN 2016 - Khối học sinh phổ thông).Giả sửavàblà hai số
thực cho|ax+b| ≤1khi|x| ≤1 Chứng minh rằng
i)|a| ≤1.
ii)|bx+a| ≤1khi|x| ≤1.
Giải.Bởi giả thiết|ax+b| ≤1khi|x| ≤1, ta có|a+b| ≤1(khix= 1) và|−a+b| ≤1(khi x=−1)
i) Từ Bất đẳng thức tam giác, ta có|2a|=|(a+b)−(−a+b)| ≤ |a+b|+|−a+b| ≤2 Suy
ra|a| ≤1 Ta có điều phải chứng minh
ii) Để chứng minh|bx+a| ≤1khi|x| ≤1, ta cần kiểm tra tạix =±1 Nói cách khác, ta
cần kiểm tra rằng|a+b| ≤1và|a−b| ≤1 Điều chứng minh câu i)
Tương tự, xét đa thứcP(x) = ax2+bx+c, thỏa mãn điều kiện|P (x)| = |ax2+bx+c| ≤ 1,∀x∈[−1,1].Áp dụng Bất đẳng thức Berstein - Markov, ta cóP0(x)≤n2, hay|2ax+b| ≤ 4, ∀x∈[−1,1]
Ta thiết lập toán sau
Bài toán (Olympic sinh viên VN 2016 - Khối học sinh phổ thông).Giả sửa, b, clà b số
thực cho giá trị đa thứcax2 +bx+ctại 1, 0,−1đều thuộc đoạn[−1,1] Chứng
minh rằng
i)|2ax+b| ≤4khi|x| ≤1.
ii)|cx2+bx+a| ≤2khi|x| ≤1.
Giải.i) Đặtd=a+b+cvàe=a−b+c Khi đó, theo giả thiết, ta có a =
2(d+e)−c, b=
(135)ii) Ta có
cx2+bx+a=c x2−1+d
2(1 +x) + e
2(1−x)
Vậy, theo bất đẳng thức tam giác, khi|x| ≤1, ta có
cx2+bx+a=
c x2−1+ d2(1 +x) + e2(1−x) ≤
x2−1+1
2|1 +x|+ e
2|1−x| = 1−x2 +1
2(1 +x+ 1−x) = 2−x ≤2.
Tương tự, xét đa thứcP(x) = ax3+bx2+cx+d, thỏa mãn điều kiện
|P(x)|=ax3+bx2+cx+d≤1,∀x∈[−1,1]
Áp dụng Bất đẳng thức Berstein - Markov, ta cóP0(x)≤n2, hay
3ax2+ 2bx+c≤9, ∀x∈[−1,1]
Ta thiết lập tốn sau giải kiến thức phổ thơng
Bài tốn (Olympic sinh viên VN 2016 - Khối học sinh phổ thông). Giả sửa, b,c,dlà bốn
số thực cho giá trịα,β,γ,δcủa đa thứcax3+bx2+cx+dtương ứng tại−1,−1 2,
1 2, 1đều thuộc đoạn[−1,1] Chứng minh rằng
i) Với số thựcA,B, ta có đẳng thức|A+B|+|A−B|= max{|A|,|B|}.
ii) Bằng cách biểu diễn3ax2+2bx+ctheoα,β,γ,δvàx, chứng minh rằng|3ax2+ 2bx+c| ≤ 9khi|x| ≤1.
(136)Bài toán 4. Cho a, b,c là ba số thực và n là số nguyên dương Giả sử đa thức f(x) = ax2n+bx+ccó giá trị1,0,−1đều thuộc đoạn[−1,1] Chứng minh rằng
i)|f(x)| ≤ 2n2n−√1 −1
4nn2n + 1khi|x| ≤1.
ii) Với mỗi1≤M <∞, ta có|f(x)| ≤2M2n−1khi1≤ |x| ≤M. Giải.i) Ta cóa=
2(d+e)−c,b =
2(d−e) Như f(x) = d
2 x
2n+x+ e
2 x 2n
−x+c 1−x2n
Theo giả thiết,max{|c|,|d|,|e|} ≤1, nên dựa vào kết luận Bài 3i) trên, khi|x| ≤1, ta
có
|f(x)| ≤
x2n+x+x2n−x+1−x2n =maxx2n,|x| + 1−x2n= +x−x2n
-Cách 1.Mặt khác, áp dụng Bất đẳng thức AM - GM cho2nsố không âm x2n, 2n−√11
4nn2n,
1
2n−√1
4nn2n, ,
1
2n−√1
4nn2n
| {z }
2n−1so
,
ta có
x2n+ 2n2n−√1 −1
4nn2n ≥2n
2n
r
x2n
4nn2n =|x|
Do
|f(x)| ≤ 2n2n−√1 −1
4nn2n +
-Cách 2.Mặt khác, ta khảo sát hàm sốy =g(t) = +t−t2nvới0≤t≤1 Dễ thấy
g0(t) = 1−2nt2n−1 >0,khi0≤t < t0 := 2n−√11
2n, g0(t)<0,khit0 < t≤1
Vậy
maxg(t) 0≤t≤1
(137)Bài toán 5. Cho đa thức với hệ số thực f(x) = ax3+bx2 +cx+d và số α > 0, thỏa mãn
|f(x)| ≤M,∀x∈[−1,1] Tìm giá trị lớn của|a|, giá trị lớn của|b|, giá trị lớn nhất của|c|, giá trị lớn của|d|
Giải.Đặt
A=f(−1) =−a+b−c+d B =f
−1
2
=−a
8+ b 4−
c +d C =f
= a + b + c +d D=f(1) =a+b+c+d E =f(0) =d
Giải hệ theo ẩna, b, c, d, ta
a=−2 3A+
4 3B−
4 3C+
2 3D b=
2A+
2D−E c=
6A− 6B+
8 6C−
1 6D d=E
Bởi giả thiết |a| ≤ 4M, |b| ≤ 2M, |c| ≤ 3M, |d| ≤ M, ta xét f(x) = M(4x3−3x) và g(x) = M(2x2−1), nên dấu đẳng thức xảy ra.
Vậy max|a|= 4M, max|b|= 2M, max|c|= 3M, max|d|=M
Bài toán 6. i) Chof(x) = 2x2+bx+c Tìmb, c∈Rsao cho|f(x)| ≤1,∀x∈[−1,1].
(138)Vậy đẳng thức phải đồng thời phải xảy đẳng thức Do đób = 0,c=−1
Ngược lại, vớib = 0,c=−1, ta dễ dàng chứng minh đượcf(x) = 2x2−1thỏa mãn
|f(x)| ≤1, ∀x∈[−1,1]
ii) Theo câu trên, kết thu đượcf(x)là Đa thức Chebyschev bậc hai điểm
chọn luân điểm f(x) Một cách tự nhiên, ta nghĩ đến kết câu
một Đa thức Chebyschev bậc ba luân điểm coskπ
3 , k= 0,1,2,3
Thật vậy, từ giả thiết, ta có
|f(−1)|,
f
−12,
f
1
, |f(1)| ≤1
Do
|4 +a+b+c| ≤1, |−4 +a−b+c| ≤1,
12 +14a+ 12b+c ≤1,
−12 +14a− 12b+c ≤1
Nhận xét rằng, bất đẳng thức|a1+a2+ +an| ≤ |a1|+|a2|+ +|an|xảy dấu đẳng thức
khi cácai dấu Với việc dự đoána= 0,b=−3,c= 0, ta có
|8 + 2b| ≤ |4 +a+b+c|+|4−a+b−c| ≤2
Suy ra|4 +b| ≤1và
|−1−b| ≤
−12− 14a− 12b−c +
−12 +14a− 12b+c ≤2
Cuối cùng, ta cần khửb Ta có3≤ |4 +b|+|−1−b| ≤1 + =
Vậy dấu tất bất đẳng thức xảy ra, haya= 0,b=−3,c=
Khi đóf(x) = 4x3−3x, với mọix∈[−1,1]bất kì Vậy tồn tạiα ∈[0, π]đểx=cosα.
(139)f
2 =8 +4 + 2+c, f −2 =8− + 2−c
Suy
f(1) +f(−1)≥ |2 + 2b|, f
1
+f
−1
2
≥
14+b
Do
f(1) +f(−1) +
f
1
+f
−1
2
≥
2
Vậy6M ≥
2 hayM ≥
Suy max |x|≤1 |x
3+ax2+bx+c|đạt giá trị nhỏ bằng
4, khia=c= 0, b=−
Bài toán 8.Chox1, x2, , xn,n ≥2, số thực phân biệt đoạn[−1,1] Chứng minh
rằng
1 t1 +
1 t2 +
1 tn ≥
2n−2,
với
tk=
n
Y
i=1,i6=k
(xi−xk)
, k = 1,2, , n
Giải. Áp dụng Công thức nội suy Lagrange cho Đa thức ChebyschevTn−1(x)bậcn−1tạin
điểmx1, x2, , xn, ta có
Tn−1(x) =
n
X
k=1
Tn−1(xk)
(x−x1) (x−xk−1) (x−xk+1) (x−xn)
(xk−x1) (xk−xk−1) (xk−xk+1) (xk−xn)
(140)
Ngoài ra, vì|Tn−1(x)| ≤1,∀ |x| ≤1vàxk∈[−1,1],∀k = 1,2, , n, nên
2n−2 ≤
n
X
k=1
Tn−1(xk) |
Tn−1(xk)|
|(xk−x1) (xk−xk−1) (xk−xk+1) (xk−xn)| ≤ n
X
k=1 tk
Ta có điều phải chứng minh
Tài liệu tham khảo
[1] Trịnh Đào Chiến, Huỳnh Minh Thuận,Một số ứng dụng Đa thức nội suy Lagrange, Tạp chí Khoa học Trường Đại học Quy Nhơn, tập II, số 3, năm 2008
[2] Trần Nam Dũng,Đa thức, file pdf, nguồn Internet
[3] Nguyễn Văn Mậu,Đa thức đại số phân thức hữu tỉ, Nhà xuất Giáo dục 2002 [4] Đề thi đáp án Olympic sinh viên Việt Nam 2016 (Khối học sinh phổ thông)
(141)GIỚI THIỆU
Bên cạnh bất đẳng thức đối xứng, phổ biến bất đẳng thức đoạn, khoảng đề tài vơ hấp dẫn Cái khó đến từ việc khai thác triệt để giả thiết cho Chẳng hạn, với giả thiếta, b, c ∈[0,1], ta thường bắt gặp việc sử dụng
các đánh giáabc>0,(a−1)(b−1)(c−1)60 Thế nhưng, việc liên tục sử dụng
đánh giá trung gian giảm độ chặt bất đẳng thức, khiến ta khơng thể giải tốn ban đầu, khơng đủ khéo léo Qua viết này, xin đưa cách “phiên dịch” giả thiết dạng thành đẳng thức để bảo toàn độ chặt bất đẳng thức cần chứng minh
1 Cơ sở
Bổ đề Cho số thựcx∈(a, b] Khi ta có đánh giá x−a∈(0;b−a]→
x−a ∈
1
b−a,+∞
→
x−a −
b−a ∈[0,+∞)
Đặtm= x−a −
1
b−a thìm >0,ta có mối quan hệx=
b−a
m(b−a) + +a.
Chú ý
• Nếub →+∞thì đặtm =x−a
• Nếu xét biến trên[a, b]thì xét trước trường hợp tồn biến bằnga 2 Bài tập
Bài tập Choa, b∈[0,1] Chứng minh rằng
(142)•
chứng minh trở thànhb61, điều hiển nhiên theo giả thiết
• Nếu hai biến khác0 Đặta=
x+ 1, b=
y+ thìx, y >0 Khi a+b = x+y+
(x+ 1)(y+ 1), ab=
1 (x+ 1)(y+ 1)
⇒ab+ > xy+x+y+ (x+ 1)(y+ 1) >
x+y+
(x+ 1)(y+ 1) =a+b
Bài toán chứng minh xong
Nhận xét Bài toán bản, cách làm quen thuộc để ý
(ab+ 1)−(a+b) = (a−1)(b−1)>0
Cách làm dài hơn, cho ta thấy rõ độ yếu bất đẳng thức (chỉ cầnxy>0)
và quan trọng hơn, tảng tư cho sau đây.
Bài tập Choa, b, c∈[0; 1] Chứng minh rằng a+b+c6abc+
Lời giải.
Cách 1.Theo đề ta có
(a−1)(b−1)(c−1)60⇒a+b+c6ab+bc+ca−abc+
Lại để ý
(a−1)(b−1)>0⇒ab+ >a+b ⇒abc+c>bc+ca
⇒a+b+c6abc+c+ab−abc+ =c+ab+ (1)
Hơn
(ab−1)(c−1)>0⇒c+ab6abc+ ⇒c+ab+ 6abc+ (2)
Từ(1)và(2)ta có đpcm
Cách 2.
• Giả sử tồn biến bằng0 Khơng tính tổng quát, giả sửa= BĐT cần chứng
(143)⇔xy+yz+zx+ 2xyz >0,
một điều hiển nhiên Bài toán chứng minh xong
Nhận xét Về mức độ khó Cách ngắn gọn cần chút
khéo léo Và lần phương pháp đặt ẩn phụ giải gọn gàng toán mặt ý tưởng.
Bài tập Choa, b, c∈[0,1] Chứng minh rằng
a+b+c−(ab+bc+ca)61
(Đề chọn đội tuyển VMO trường Phổ thơng Năng khiếu) Lời giải.
• Nếu ba sốa, b, ccó số bằng0, giả sửa= 0, BĐT cần chứng minh trở thành b+c−bc61,
chính Bài
• Nếu ba biếna, b, cđều khác0, lần đặta =
x+ 1, b =
y+ 1, c = z+ x, y, z >0 Khi
a+b+c= 2(x+y+z) +xy+yz+zx+ (1 +x)(1 +y)(1 +z) , ab+bc+ca= x+y+z+
(1 +x)(1 +y)(1 +z)
BĐT cần chứng minh trở thành
x+y+z+xy+yz+zx (1 +x)(1 +y)(1 +z) 614
⇔x+y+z+xy+yz+zx6x+y+z+xy+yz+zx+xyz+ 1⇔xyz+ >0,
(144)léo tinh tế không nhỏ Trong đó, phương pháp ta đề cập đến giải tốt ý tưởng, khó khăn có khối lượng tính tốn mà thơi Một ví dụ biến thể đề Olympic 30 - năm 2001
Bài tập Choa, b, c∈[1,2] Chứng minh rằng (a+b+c)
a + b + c <10 Lời giải.
Cách 1.BĐT cần chứng minh tương đương với
a b + b a + b c+ c b + c a + a c <7
Khơng tính tổng qt, giả sử16c6b6a62 Khi (a−b)(a−c)>0⇔ab+bc>b2+ca⇔
a
c + 1> a b +
b c c
a + 1> c b +
b a
Lại có
2> a c >
1
2 ⇒(2c−a)(a−2c)<0⇒2(a
2+c2)<5ca⇒5>2a c + c a Như a c + c
a + > a b +
b c+
a c + c a , tức a b + b a + b c+ c b + c a + a c <7
Bài toán chứng minh xong
Cách 2.Đặta = x+ x+ 1, b =
y+ y+ 1, c =
z+
z+ thìx, y, z >0
Khi
a+b+c= 4(xy+yz+zx) + 5(x+y+z) + 3xyz+ (x+ 1)(y+ 1)(z+ 1) , a + b + c =
5(xy+yz+zx) + 8(x+y+z) + 3xyz + 12 (x+ 2)(y+ 2)(z+ 2)
Đặt
p=x+y+z, q =xy+yz+zx,
(145)lại điều hiển nhiên nữa, toán chứng minh xong
Nhận xét Đến này, rõ ràng ý tưởng chứng minh cách 1, ta thấy, hay đẹp mắt
nhưng việc nghĩ không dễ Trong cách 2, với dòng tư trên, ta thấy đem lại một lời giải tự nhiên dù tính tốn có chút cồng kềnh.
Bài tập Chox, y, z ∈[0,1] Chứng minh rằng
(x−x2)(y−y2)(z−z2)>(x−yz)(y−zx)(z−xy)
(Đề chọn đội tuyển chuyên Bảo Lộc - Lâm Đồng 2016) Lời giải.
Cách 1.Ta có đẳng thức sau
(x−x2)(y−y2)(z−z2) = (xyz−x2y2z2)−xyz(x+y+z−xy−yz−zx), (x−yz)(y−xz)(z−xy) = (xyz−x2y2z2)−(x2y2+y2z2+z2x2) +xyz(x2+y2+z2)
BĐT cần chứng minh tương đương với
xyz(x+y+z−xy−yz−zx)6(x2y2+y2z2+z2x2)−xyz(x2+y2+z2)
⇔3xyz(x+y+z−xy−yz−zx)leq(xy+yz+zx)2−xyz(x+y+z)2
Đặtx=
1 +a, y =
1 +b, z=
1 +c thìa, b, c >0 Khi
x+y+z = 2(a+b+c) +ab+bc+ca+ (1 +a)(1 +b)(1 +c) ,
xy+yz+zx= a+b+c+ (1 +a)(1 +b)(1 +c), xyz =
(1 +a)(1 +b)(1 +c)
Thế
(146)(a+b+c+ 3)2(a+ 1)(b+ 1)(c+ 1)−[2(a+b+c) + (ab+bc+ca) + 3]2
>3(a+b+c+ab+bc+ca)(a+ 1)(b+ 1)(c+ 1) (3)
Đặt
p=a+b+c, q =ab+bc+ca,
r =abc
thì
(a+ 1)(b+c)(c+ 1) =p+q+r+
Do(a−b)2+ (b−c)2+ (c−a)2 >0nênp2 >3q Khi đó(3)trở thành (p+q+r+ 1)[(p+ 3)2−3(p+q)]>(2p+q+ 3)2
⇒(p+q+r+ 1)(p2+ 3p−3q+ 9)>(2p+q+ 3)2
⇔p3+p2q−3q2+r(p2 + 3p−3q+ 9)>q2+ 4pq
Theo BĐT Schur cho bậc thìp3+ 9r>4pq, nên ta cần chứng minh
p2q+r(p2+ 3p−3q)>4q2 (4)
Xét trường hợp:
• 4q6p2 thì vìp2q >4q2, r(p2+ 3p−3q)>r(p2−4q)>0nên(4)đúng.
• 4q > p2, theo BĐT Schur bậc để ý rằngp2 + 3p−3q>3p, ta được r(p2+ 3p−3q)> p(4q−p
2) 3p=
p2(4q−p2)
nên toán hoàn tất ta chứng minh
3p2q+p2(4q−p2)>12q2 ⇔(4q−p2)(p2−3q)>0,
một điều chắn Bài toán chứng minh xong
Cách (Michael Rozenberg): Vớia ∈ [0,1]thìa−a2 > 0nên vế trái ln khơng âm, đó
nếu vế phải khơng dương BĐT ln Vì vậy, ta cần chứng minh BĐT cho trường hợp vế phải khơng âm
Vớix, y, z ∈[0,1]thì
(147)Rõ ràng phương pháp có điểm yếu, biến đổi tương đương dài dòng phức tạp Tuy nhiên, kết hợp với biến đổi ban đầu khéo léo ta lời giải khơng q dài dịng.
Bài tập Choa, b, c∈[0,1] Chứng minh rằng a
b+c+ + b
c+a+ + c
a+b+ + (1−a)(1−b)(1−c)61
(China TST 2003) Lời giải.
• Nếua+b+c= 0thì theo đề dẫn đếna=b=c= 0, BĐT xảy dấu
• Nếua+b+c0, BĐT cần chứng minh tương đương với 1−
a b+c+ +
b
c+a+ + c a+b+
>(1−a)(1−b)(1−c)
⇔X
cyc
a a+b+c−
a
b+c+ >(1−a)(1−b)(1−c)
⇔
a+b+c
X
cyc
a(1−a)
b+c+ >(1−a)(1−b)(1−c)
⇔X
cyc
a(1−a)
b+c+ >(1−a)(1−b)(1−c)(a+b+c)
⇔X
cyc
a(1−a)
1
b+c+ −(1−b)(1−c)
>0
Vậy ta cần vớim, n∈[0,1]thì
(148)n+
hiển nhiên
- Nếu hai sốm, nđều khác 0, đặtm =
1 +x, n =
1 +y thìx, y >0 Khi
m+n+ =
(1 +x)(1 +y)
(1 +x)(1 +y) +x+y+ 2,
(1−m)(1−n) = xy
(1 +x)(1 +y)
Ta cần chứng minh
(1 +x)2(1 +y)2 >xy[(1 +x)(1 +y) +x+y+ 2]
Bằng biến đổi tương đương, rút gọn điều cần chứng minh trở thành
(x+y+ 1)(x+y+ 1) >xy,
hiển nhiên Bài toán chứng minh xong
Bài tập Choa, b, c∈[0,1]thoả
1 a −1
1 b −1
1 c −1
=
Tìm GTNN biểu thức
S =a2+b2+c2
(Thi thử THPT QG 2016 THPT Thanh Hoa, Bình Phước) Lời giải. Trước hết xin phát biểu không chứng minh bổ đề đơn giản quan trọng
Bổ đề Vớia, blà số thực dương thoả mãnab>1, ta có BĐT
a2 + 1 + b2+ 1 >
2 +ab
Chứng minh bổ đề đơn giản, xin dành lai cho bạn đọc.
Quay lại toán Đặt a =
x+ 1.b =
y+ 1, c =
z+ x, y, z > Giả thiết trở thành xyz = 1và S trở thành
S = (x+ 1)2 +
1 (y+ 1)2 +
(149)3 Một số bất đẳng thức không mẫu mực
Ở ta làm quen với phương pháp đơn giản ý tưởng để giải bất đẳng thức đoạn Như trình bày, đơn giản đơi với tính tốn vơ phức tạp Những tốn cho thấy phức tạp khiến phương pháp khó khả thi, đó, nhạy cảm quan sát tinh tế chìa khóa giải
Bài tập Choa, b, c∈
1
√
6,
√
6
Chứng minh rắng
4 a+ 3b +
4 b+ 3c +
4 c+ 3a >
3 a+ 2b +
3 b+ 2c+
3 c+ 2a
(Mihacla Berindeanu)
Nhận xét Rõ ràng việc áp dụng phương pháp đổi biến đem lại khối lượng tính tốn nặng,
chưa kể thức xuất điều kiện làm chùn tay Đây lúc nhạy cảm toán học lên tiếng!
Lời giải. BĐT cần chứng minh tương đương với
4 a+ 3b −
3 a+ 2b
+
4 b+ 3c −
3 b+ 2c
+
4 c+ 3a −
3 c+ 2a
>0
Xét số hạng đại diện
4 a+ 3b −
3 a+ 2b
= a−b (a+ 3b)(a+ 2b)
Số hạng luôn không âm, ta thêm hạng tử có dạng
ka− kb
vào số hạng đại diện, chứng minh tổng có sau thêm, cụ thể
4 a+ 3b −
3 a+ 2b
= a−b
(a+ 3b)(a+ 2b) + ka −
(150)4 a+ 3b −
3
a+ 2b =
a−b
(a+ 3b)(a+ 2b)+ ka−
1
kb >0∀a, b
Bằng phép biến đổi tương đương kĩ thuật chia đa thức hai biến, ta tìm đượck= 12là số
cần tìm (việc chứng minh xin dành cho bạn đọc) Kiểm tra lại, ta thấy
4 a+ 3b −
3 a+ 2b
= a−b
(a+ 3b)(a+ 2b) + 12a −
1 12b =
(a−b)2(6b−a)
12ab(a+ 3b)(a+ 2b) >0
(chú ý6b−a>6.√1
6 −
√
6>0) Chứng minh tương tự với trường hợp lại cộng vế
theo vế, ta có đpcm
Bài tập Choa, b, c∈[0,1] Chứng minh rằng
a+b+c6ab2+bc2+ca2+
Lời giải. Trước hết xin phát biểu chứng minh bổ đề quan trọng khác
Bổ đề Chom, n, p >0 Khi ta có
mn2+np2+pm2
27(m+n+p) 3.
Chứng minh. Khơng tính tổng quát, chuẩn hoám+n+p= 3và giả sửmnằm giữan, p
Khi
p(m−n)(m−p)60⇒np2+pm2 6m(p2 +pn)
⇒mn2+np2+pm2 6m(n2+p2+pn)6m(n+p)2 =m(3−m)2 64
và điều chứng minh bổ đề ta
Quay lại toán BĐT cần chứng minh tương đương với
a(1−b2) +b(1−c2) +c(1−a2)6
Áp dụng bổ đề, ta có
S 62(x+y+z)−(x+y+z)2+
27(x+y+z) 3.
Đặtu=x+y+z ∈[0,3] Khảo sát hàmg(u)trên[0,3], ta tìm GTLN củag(u)là đạt
tạiu=
(151)Bài tập 12 (Nguyễn Anh Cường) Chox, y, z ∈[1,2] Chứng minh rằng xy
yz+zx + yz zx+xy +
zx xy+yz
19 12
Bài tập 13. 1 (Algebraic Inequalities, Old and New methods) Cho số thựca, b, cthuộc
√
2,
√
2
Chứng minh
3 a+ 2b +
3 b+ 2c +
3 c+ 2a >
2 a+b +
2 b+c+
2 c+a
2 (Bài toán tổng quát) Cho số dươngkvàa, b, c∈
"
1
p
k(k+ 1),
p
k(k+ 1)
#
Chứng minh
k+ a+kb +
k+ b+kc +
k+ c+ka >
k
a+ (k−1)b +
k
b+ (k−1)c+
k c+ (k−1)a
Bài tập 14 Choa, b, cthoả06a616b 6cvàa+b+c= 3 Chứng minh rằng a2b+b2c+c2a>abc+
Bài tập 15 Choa, b, cthoả06a616b 6cvàab+bc+ca= 3 Chứng minh rằng a2b+b2c+c2a>abc+
Tài liệu
[1] Nguyễn Anh Cường,ABC Method Abstract Concreteness
[2] Võ Quốc Bá Cẩn,Phân loại phương pháp chứng minh bất đẳng thức
(152)BÀI TOÁN HAY LỜI GIẢI ĐẸP
Ban biên tập
GIỚI THIỆU
Chuyên mục lấy cảm hứng từ viết thầy Nguyễn Duy Liên số báo thứ
3về toán số6trong kỳ thi IMO2001với5cách giải khác Mục để dành
viết toán hay, lời giải đẹp câu chuyện thú vị xung quanh tốn lời giải
Tên chuyên mục mượn từ tên nhóm người u tốn Facebook anh Nguyễn Văn Lợi sáng lập “Bài toán hay – Lời giải đẹp – Đam mê toán học” Chuyên mục ghi nhận đề cử bạn đọc chọn đăng kỳ1; 2bài tốn
Số chúng tơi giới thiệu toán số5của IMO2014;bài toán đồng xu
ngân hàng Cape Town Bài toán Luxembourg đề nghị Ban tuyển chọn đề xếp vào phần Số học
Bài toán (Bài toán 5; IMO 2014) Với số nguyên dương n, Ngân hàng Cape Town phát
hành đồng xu mệnh giá
n:Cho hữu hạn đồng xu (khơng thiết
phải có mệnh giá khác nhau) có tổng giá trị khơng q99C12:Chứng minh ta chia
các đồng xu thành100nhóm hơn, cho nhóm có tổng giá trị không quá1:
Chúng ta thấy qua lời giải toán thật toán tổ hợp Lời giải toán sử dụng hướng tiếp cận xây dựng thuật toán với3bước
1 Tổng qt hóa tốn để thực bước “rút gọn” sau
2 “Rút gọn” toán bước ghép xu loại bỏ đồng xu mệnh giá1:
3 Phân chia nhóm lớn thực thuật tốn phân phối đồng xu nhỏ
Lời giải Ta chứng minh với số nguyên dươngN;mọi hữu hạn đồng xu có
tổng mệnh giá khơng quáN 12 chia thànhN nhóm nhóm có tổng giá trị không 1: Khẳng định toán trường hợp riêng khiN D 100:Ta bắt đầu với vài chuẩn
bị Nếu có số đồng xu đồng xu cho có tổng mệnh giá có dạng k vớik
(153)N 12
N D
1
2N;do ta bỏ thêm đồng xu nhỏ vào nhóm Bằng cách vậy, ta có
thể phân phối tất đồng xu vào nhóm
Một lời giải đẹp! Có thể thấy lời giải không vận dụng kiến thức hay kết cao siêu nào, phép cộng, phép nhân, phép chia tư thuật tốn Bài tốn đóng gói tin học xếp vào lớp toán NP với điều kiện ràng buộc khối lượng (phải có dạng
k), tốn có thuật giải đẹp trình bày
Bước chuẩn bị (ghép đồng xu) bước tự nhiên quan trọng, giúp làm gọn liệu, loại “bùng nổ tổ hợp” không chất Ta thấy, phép biến đổi vậy, chiều (theo nghĩa sau ghép làm trước ghép làm được) thực tương đương, sau ghép, ta có cấu hình tốn
Dưới chúng tơi xin đưa số toán tương tự, giải thuật toán thành tố quan trọng bước “sắp xếp, điều chỉnh liệu”
Bài toán Có số hịn đá, hịn đá có khối lượng không quá 0; 5kg Tổng khối lượng
các hịn đá khơng vượt q3kg Chứng minh ta chia hịn đá thành4nhóm, mỗi
nhóm có tổng khối lượng khơng vượt q1:
Bài tốn Cho 50 số nguyên dương có tổng bằng99: Chứng minh với số nguyên
dươngS < 100tồn số số số có tổng bằngS:
Bài toán Một số tự nhiênnthỏa mãn điều kiện với số thực bất kỳa1; a2; : : : ; ad thỏa
mãna1Ca2C Cad D 2013và06ai 61vớii D1; 2; : : : ; d thì ln có cách chia
tập hợp số thực thànhntập đơi phân biệt (trong cho phép tập rỗng), sao
cho tổng số tập hợp ln khơng vượt q1:Hãy tìm sốnnhỏ thỏa mãn tính
chất trên.
Bài tốn Tại hội nghị quốc tế, đại biểu tham dự biết trong 3thứ tiếng
Anh, Pháp, Đức Biết có đúng50đại biểu biết tiếng Anh, đúng50đại biểu biết tiếng Pháp
và đúng 50đại biểu biết tiếng Đức Chứng minh chia đại biểu thành5 nhóm
sao cho nhóm có đúng10đại biểu biết tiếng Anh, đúng10đại biểu biết tiếng Pháp